649 336 21MB
English Pages [250] Year 2023
Table of contents :
Front Cover
Thoracic Imaging: Case Review Series, 3ed
Copyright Page
Foreword
Preface
Contents
I. Opening Round
Case 1
Case 2
Case 3
Case 4
Case 5
Case 6
Case 7
Case 8
Case 9
Case 10
Case 11
Case 12
Case 13
Case 14
Case 15
Case 16
Case 17
Case 18
Case 19
Case 20
Case 21
Case 22
Case 23
Case 24
Case 25
Case 26
Case 27
Case 28
Case 29
Case 30
Case 31
Case 32
Case 33
Case 34
II. Fair Game
Case 35
Case 36
Case 37
Case 38
Case 39
Case 40
Case 41
Case 42
Case 43
Case 44
Case 45
Case 46
Case 47
Case 48
Case 49
Case 50
Case 51
Case 52
Case 53
Case 54
Case 55
Case 56
Case 57
Case 58
Case 59
Case 60
Case 61
Case 62
Case 63
Case 64
Case 65
Case 66
Case 67
Case 68
Case 69
Case 70
Case 71
Case 72
Case 73
Case 74
Case 75
Case 76
Case 77
Case 78
Case 79
Case 80
Case 81
Case 82
Case 83
Case 84
Case 85
Case 86
Case 87
Case 88
Case 89
Case 90
Case 91
Case 92
Case 93
Case 94
Case 95
Case 96
Case 97
Case 98
Case 99
Case 100
Case 101
Case 102
III. Challenge
Case 103
Case 104
Case 105
Case 106
Case 107
Case 108
Case 109
Case 110
Case 111
Case 112
Case 113
Case 114
Case 115
Case 116
Case 117
Case 118
Case 119
Case 120
Case 121
Case 122
Case 123
Case 124
Case 125
Case 126
Case 127
Case 128
Case 129
Case 130
Case 131
Case 132
Case 133
Case 134
Case 135
Case 136
Case 137
Case 138
Case 139
Case 140
Case 141
Case 142
Case 143
Case 144
Case 145
Case 146
IV. Answers
Opening Round
Case 1
Squamous Cell Carcinoma of the Lung
Comment
Differential Diagnosis
Lung Cancer
Imaging Findings
REFERENCES
Case 2
Spontaneous Pneumothorax
Comment
Differential Diagnosis
Discussion
Case 3
Pleural Plaques
Comment
Differential Diagnosis
Case 4
Emphysema
Comment
Case 5
Sarcoidosis (Stage I)
Comment
Differential Diagnosis
Discussion
Case 6
Mesothelioma
Comment
Differential Diagnosis
Discussion
Case 7
Junction Lines
Comment
Differential Diagnosis
Discussion
Case 8
Hamartoma
Comment
Differential Diagnosis
Discussion
Case 9
Cystic Fibrosis
Comment
Differential Diagnosis
Discussion
Case 10
Mediastinal Mass (Vascular)
Differential Diagnosis
Discussion
Comment
Differential Diagnosis
Discussion
Comment
Optimal Location of Central Venous Catheter
Azygos Line Placement
Comment
Comment
Differential Diagnosis
Imaging Findings
Comment
Differential Diagnosis
Imaging Findings
Comment
Differential Diagnosis
Imaging Findings
Comment
Differential Diagnosis
Imaging Findings
Comment
Differential Diagnosis
Discussion
Comment
Differential Diagnosis
Discussion
Comment
Comment
Comment
Comment
Comment
Differential Diagnosis
Solitary Pulmonary Nodule
Comment
Differential Diagnosis
Discussion
Comment
Differential Diagnosis
Discussion
Comment
Differential Diagnosis
Discussion
Comment
Differential Diagnosis
Discussion
Comment
Differential Diagnosis
Pericardiophrenic Mass
Imaging Findings
Comment
Differential Diagnosis
Lobar Consolidation
Imaging Findings
Comment
Differential Diagnosis
Discussion
Case 11
Lung Contusion and Laceration
Case 12
Malpositioned Catheter
Case 13
Paraspinal Hematoma
Case 14
Miliary Tuberculosis
Case 15
Neurofibromatosis
Case 16
Left Lower Lobe Atelectasis
Case 17
Pulmonary Hypertension
Case 18
Lung Abscess
Case 19
Pneumomediastinum
Case 20
Subpulmonic Pleural Effusion
Case 21
Pneumothorax on Supine Radiograph
Case 22
Loculated Pleural Fluid in the Major Fissure (Pseudotumor)
Case 23
Esophageal Cancer
Case 24
Benign Calcified Granuloma
Case 25
Radiation Pneumonitis
Case 26
Acute Respiratory Distress Syndrome With Barotrauma
Case 27
Cavity From Postprimary Tuberculosis
Case 28
Mycetoma
Case 29
Pericardial Cyst
Case 30
Right Lower Lobe Pneumonia
Case 31
Empyema
Case 32
Neurogenic Tumor (Schwannoma)
Comment
Differential diagnosis
Discussion
Comment
Differential Diagnosis
Hemothorax
Imaging Findings
Comment
Differential Diagnosis
Neurofibromatosis Type 1 (NF-1)
Imaging Findings
Comment
Differential Diagnosis
Discussion
Comment
Differential Diagnosis
Discussion
Comment
Differential Diagnosis
Discussion
Comment
Differential Diagnosis
Discussion
Comment
Differential Diagnosis
Discussion
Comment
Differential Diagnosis
Discussion
Comment
Differential Diagnosis
Discussion
Comment
Differential Diagnosis
Discussion
Comment
Differential Diagnosis
Discussion
Comment
Differential Diagnosis
Discussion
Comment
Differential Diagnosis
Discussion
Comment
Differential Diagnosis
Discussion
Comment
Differential Diagnosis
Discussion
Comment
Differential Diagnosis
Discussion
Comment
Differential Diagnosis
Discussion
Comment
Differential Diagnosis
Discussion
Comment
Differential Diagnosis
Discussion
Comment
Differential Diagnosis
Discussion
Comment
Differential Diagnosis
Discussion
Comment
Differential Diagnosis
Discussion
Comment
Differential Diagnosis
Discussion
Comment
Differential Diagnosis
Discussion
Comment
Differential Diagnosis
Discussion
Comment
Differential Diagnosis
Discussion
Comment
Comment
Comment
Comment
Comment
Comment
Comment
Comment
Comment
Comment
Comment
Comment
Comment
Comment
Comment
Comment
Comment
Comment
Comment
Comment
Comment
Comment
Comment
Comment
Differential Diagnosis
Discussion
Comment
Differential Diagnosis
Discussion
Comment
Differential Diagnosis
Discussion
Comment
Differential Diagnosis
Discussion
Comment
Differential Diagnosis
Discussion
Comment
Differential Diagnosis
Discussion
Comment
Differential Diagnosis
Discussion
Comment
Differential Diagnosis
Discussion
Comment
Differential Diagnosis
Discussion
Comment
Differential Diagnosis
Discussion
Differential Diagnosis
Discussion
Comment
Differential Diagnosis
Discussion
Comment
Differential Diagnosis
Discussion
Comment
Differential Diagnosis
Discussion
Comment
Comment
Differential Diagnosis
Discussion
Comment
Differential Diagnosis
Discussion
Comment
Differential Diagnosis
Discussion
Comment
Differential Diagnosis
Discussion
Comment
Differential Diagnosis
Discussion
Comment
Differential Diagnosis
Comment
Differential Diagnosis
Discussion
Comment
Differential Diagnosis
Discussion
Comment
Differential Diagnosis
Discussion
Comment
Differential Diagnosis
Discussion
Comment
Differential Diagnosis
Discussion
Comment
Differential Diagnosis
Discussion
Comment
Differential Diagnosis
Discussion
Comment
Differential Diagnosis
Discussion
Comment
Differential Diagnosis
Discussion
Comment
Differential Diagnosis
Discussion
Comment
Differential Diagnosis
Discussion
Comment
Differential Diagnosis
Discussion
Comment
Discussion
Comment
Differential Diagnosis
Discussion
Comment
Differential Diagnosis
Discussion
Comment
Differential Diagnosis
Discussion
Comment
Differential Diagnosis
Discussion
Comment
Differential Diagnosis
Discussion
Comment
Comment
Comment
Comment
Comment
Comment
Comment
Comment
Comment
Comment
Comment
Comment
Comment
Comment
Differential Diagnosis
Comment
Comment
Differential Diagnosis
Discussion
Comment
Differential Diagnosis
Discussion
Comment
Differential Diagnosis
Discussion
Comment
Differential Diagnosis
Comment
Comment
Differential Diagnosis
Discussion
Comment
Differential Diagnosis
Discussion
Comment
Differential Diagnosis
Comment
Comment
Differential Diagnosis
Comment
Comment
Differential Diagnosis
Discussion
Comment
Differential Diagnosis
Comment
Comment
Differential Diagnosis
Discussion
Comment
Differential Diagnosis
Discussion
Case 33
Hemothorax
Case 34
Rib Notching
Fair Game
Case 35
Left Upper Lobe Atelectasis (Lung Cancer)
Case 36
Interstitial Pulmonary Edema
Case 37
Thyroid Goiter
Case 38
Community Acquired Pneumonia
Case 39
Pulmonary Langerhans Cell Histiocytosis
Case 40
Lymphangitic Carcinomatosis
Case 41
Rounded Atelectasis
Case 42
Sarcoidosis (Stage IV)
Case 43
Emphysema (Centrilobular)
Case 44
Anterior Mediastinal Mass
Case 45
Subcarinal Lymph Node Enlargement Secondary to Metastatic Disease
Case 46
Silicosis
Case 47
Connective Tissue Disease-Associated Interstitial Lung Disease (Systemic Sclerosis [Scleroderma] With Nonspecific Interstit ...
Case 48
Ranke Complex
Case 49
Bronchiectasis
Case 50
Arteriovenous Malformation
Case 51
Aortic Aneurysm
Case 52
Pulmonary Infarct
Case 53
Lymphoma
Case 54
Traumatic Aortic Injury (Transection)
Case 55
Lipoid Pneumonia
Case 56
Kartagener Syndrome (Primary Ciliary Dyskinesia)
Case 57
Invasive Aspergillus
Case 58
Granulomatosis With Polyangiitis
Case 59
Lymphangioleiomyomatosis
Case 60
Tuberculosis
Case 61
Pneumocystis jirovecii Pneumonia
Case 62
Metastatic Osteosarcoma
Case 63
Septic Infarcts
Case 64
Bronchopleural Fistula
Case 65
Superior Sulcus Tumor
Case 66
Tuberculosis (Post-Primary Type)
Case 67
Lymphadenopathy From Lung Cancer
Case 68
Subsolid Nodule (Adenocarcinoma)
Case 69
Azygos Continuation of the Inferior Vena Cava
Case 70
Right Upper Lobe Atelectasis (Lung Cancer)
Case 71
Traumatic Rupture of the Left Hemidiaphragm
Case 72
Persistent Left Superior Vena Cava
Case 73
Aspiration
Case 74
Nonspecific Interstitial Pneumonitis (NSIP)
Case 75
Ascending Aortic Aneurysm
Case 76
Carcinoid Tumor
Case 77
Boerhaave Syndrome
Case 78
Bronchiectasis
Case 79
Giant Bulla
Case 80
RML and RLL atelectasis
Case 81
Lipoma
Case 82
Apical Cap Secondary to Extrapleural Hematoma
Case 83
Bleomycin Lung Toxicity
Case 84
Primary Lung Adenocarcinoma With N2 Nodal Disease
Case 85
Superior Vena Cava Syndrome from Lung Cancer
Case 86
Diffuse Alveolar Hemorrhage From Vasculitis
Case 87
Usual Interstitial Pneumonia (UIP) Caused by Idiopathic Pulmonary Fibrosis (IPF)
Case 88
Intramuscular Hematoma From Supratherapeutic Warfarin Therapy
Case 89
Scimitar Syndrome
Case 90
Pericardial Effusion
Case 91
Idiopathic Bronchiectasis
Case 92
Achalasia
Case 93
Cavity Caused by Coccidioidomycosis
Case 94
Hydrostatic Pulmonary Edema
Case 95
Internal Mammary Lymph Node Enlargement
Case 96
CT-Guided Transthoracic Needle Biopsy Procedure
Case 97
Bronchial Atresia
Case 98
Hypersensitivity Pneumonitis
Case 99
Solitary Pulmonary Nodule: Lung Cancer Screening
Case 100
Post-Intubation Tracheal Stenosis
Case 101
Thymic Hyperplasia
Case 102
Part-Solid Nodule (Lepidic Predominant Lung Adenocarcinoma)
Challenge
Case 103
Pulmonary Alveolar Proteinosis
Case 104
Allergic Bronchopulmonary Aspergillosis (ABPA)
Case 105
Mucoid Impaction (Endobronchial Hamartoma)
Case 106
Solitary Fibrous Tumor of the Pleura
Case 107
Complete Lung Collapse Secondary to an Endobronchial Lesion (Lung Cancer)
Case 108
Mycobacterium avium complex (MAC) infection
Case 109
Sarcoidosis (Perilymphatic Nodules)
Case 110
Small Airways Disease (Infectious Bronchiolitis)
Case 111
Mosaic Attenuation (Small Airway Disease)
Case 112
Chronic Beryllium Disease
Case 113
Chronic Eosinophilic Pneumonia
Case 114
Tracheal Diverticulum
Case 115
Sternal Dehiscence After Median Sternotomy
Case 116
Mediastinitis
Case 117
Cryptogenic Organizing Pneumonia
Case 118
Tracheobronchial Amyloidosis
Case 119
Constrictive Bronchiolitis (Swyer-James-MacLeod Syndrome)
Case 120
Metastasizing Leiomyomatosis
Case 121
Asbestosis
Case 122
Mounier-Kuhn Syndrome
Case 123
Tracheomalacia
Case 124
Adenoid Cystic Carcinoma
Case 125
Nocardia
Case 126
Chronic Thromboembolic Disease
Case 127
Metastatic Thyroid Cancer
Case 128
Castleman Disease (Benign Lymph Node Hyperplasia)
Case 129
Aortic dissection
Case 130
Relapsing Polychondritis
Case 131
Swyer-James-Macleod Syndrome
Case 132
Accessory Cardiac Bronchus
Case 133
Intralobar Sequestration
Case 134
Acute Pulmonary Emboli With Right Heart Strain
Case 135
Panlobular Emphysema (Alpha-1 Antitrypsin Deficiency)
Case 136
Thymic Cyst
Case 137
Tracheal Bronchus
Case 138
Amiodarone Toxicity
Case 139
Saphenous Vein Graft Aneurysm Following CABG Surgery
Case 140
Infectious Bronchiolitis
Case 141
Cytomegalovirus Pneumonia in a Lung Transplant Recipient
Case 142
Pulmonary Veno-Occlusive Disease
Case 143
Tracheobronchial Papillomatosis
Case 144
Echinococcal Cyst
Case 145
Congenital Pulmonary Airway Malformation (CPAM)
Case 146
Lymphoid Interstitial Pneumonia (LIP)
V. Supplementary Figures
Index
i w
HHMHIIE
MBIMII
>
fii
w in Sil i
ora i
/ *
«J
Q CASE REVIEW SERIES ELSEVIER
r
3rd EDITION
Any screen. Any time. Anywhere. Activate the eBook version of this title at no additional charge.
Elsevier eBooks+ gives you the power to browse, search, and customize your content, make notes and highlights, and have content read aloud.
Unlock your eBook today. 1. Visit http://ebooks.health.elsevier.com/ 2. Log in or Sign up 3. Scratch box below to reveal your code 4. Type your access code into the “Redeem Access Code” box 5. Click “Redeem”
It’s that easy!
Place Peel Off Sticker Here
For technical assistance: email [email protected] call 1-800-545-2522 (inside the US) call +44 1 865 844 640 (outside the US) Use of the current edition of the electronic version of this book (eBook) is subject to the terms of the nontransferable, limited license granted on http://ebooks.health.elsevier.com/. Access to the eBook is limited to the first individual who redeems the PIN, located on the inside cover of this book, at http://ebooks.health.elsevier.com/ and may not be transferred to another party by resale, lending, or other means. 2022v1.0
Thoracic Imaging Case Review Series
Thoracic Imaging Case Review Series THIRD EDITION
JUSTIN T. STOWELL, MD Assistant Professor of Radiology Consultant Division of Cardiothoracic Imaging, Department of Radiology, Mayo Clinic Jacksonville, Florida
JONATHAN H. CHUNG, MD Professor of Radiology Chief Quality Ofcer, Department of Radiology Section Chief, Thoracic Radiology The University of Chicago Medicine Chicago, Illinois
JEFFREY P. KANNE, MD, FACR, FCCP Professor of Radiology Chief of Thoracic Imaging University of Wisconsin School of Medicine and Public Health Madison, Wisconsin
THERESA C. MCLOUD, MD Professor of Radiology Harvard Medical School Senior Advisor for Faculty Affairs and Thoracic Radiologist Massachusetts General Hospital Boston, Massachusetts
GERALD F. ABBOTT, MD, FACR Associate Professor Harvard Medical School Division of Thoracic Imaging and Intervention Massachusetts General Hospital Boston, Massachusetts
Elsevier 1600 John F. Kennedy Blvd. Ste 1800 Philadelphia, PA 19103-2899 THORACIC IMAGING: CASE REVIEW SERIES, THIRD EDITION
ISBN: 978-0-323-42879-8
Copyright © 2024 by Elsevier Inc. All rights reserved No part of this publication may be reproduced or transmitted in any form or by any means, electronic or mechanical, including photocopying, recording, or any information storage and retrieval system, without permission in writing from the publisher. Details on how to seek permission, further information about the Publisher’s permissions policies, and our arrangements with organizations such as the Copyright Clearance Center and the Copyright Licensing Agency, can be found at our website: www.elsevier.com/permissions. This book and the individual contributions contained in it are protected under copyright by the Publisher (other than as may be noted herein).
Notice Practitioners and researchers must always rely on their own experience and knowledge in evaluating and using any information, methods, compounds, or experiments described herein.Because of rapid advances in the medical sciences in particular, independent verication of diagnoses and drug dosages should be made. To the fullest extent of the law, no responsibility is assumed by Elsevier, authors, editors, or contributors for any injury and/or damage to persons or property as a matter of products liability, negligence, or otherwise, or from any use or operation of any methods, products, instructions, or ideas contained in the material herein.
Previous editions copyrighted 2011, 2001
Senior Content Development Manager: Somodatta Roy Choudhury Senior Content Strategist: Melanie Tucker Senior Content Development Specialist: Priyadarshini Pandey Publishing Services Manager: Shereen Jameel Project Manager: Vishnu T. Jiji Senior Designer: Amy L. Buxton Printed in India Last digit is the print number:
9
8
7
6
5
4
3
2
1
This book is dedicated to radiology residents and other trainees and practitioners in related elds with the hope that the provided cases and discussions will expose them to the rich variety of imaging ndings in thoracic imaging and their relevant clinical features. Justin T. Stowell, MD Jonathan H. Chung, MD Jeffrey P. Kanne, MD, FACR, FCCP Theresa C. McLoud, MD Gerald F. Abbott, MD, FACR
Foreword What is more fundamental to radiology than thoracic imaging? Radiology imaging no doubt starts here for most residents, and yet the depth of investigation into cardiopulmonary pathology stretches into the nal year of residency (or fellowship) training. The lungs and heart yield ever greater challenges to the reader, even as our imaging modalities get more and more sophisticated. So it is that Thoracic Imaging: Case Review Series, Third Edition, serves the needs of all resident levels, fellows, and practicing radiologists. This is a book for the ages. There are new COVID-like pearls and all kinds of new autoimmune and infectious agents that are explored in the latest edition. Congratulations to Drs. Gerald F. Abbott, Jonathan H. Chung, Jeffrey P. Kanne, Theresa C. McLoud (goddess of thoracic radiology), and Justin T. Stowell for their latest
contributions to the ever-vibrant Case Review Series. These books are perfect for the current generation of learners who want the short, pertinent hits that they can enjoy during breaks from clinical cases or video games or TikTok submissions. The series keeps your attention by stimulating you, like with ongoing longitudinal assessments, for a few images, quick questions, short blurb, and relevant reference. Boom. Move on to next case. Thank you to the authors for their hard work. To our readers, I hope you enjoy Thoracic Imaging: Case Review Series, Third Edition, whether you are currently reading hard copy or Soft Copy … or both. Live, love, LEARN, and leave a legacy. David M. Yousem, MD, MBA
vii
Preface The publication of the third edition of Thoracic Imaging: The Requisites in 2019 by our close friend and colleague, Jo-Anne O. Shepard, MD, was a major undertaking and involved the collaboration of numerous other experts in thoracic imaging. The many co-editors and contributors to that work all share close ties to the Massachusetts General Hospital (MGH) Department of Thoracic Imaging and Intervention (Boston, MA). We are pleased to present the long-awaited accompanying Thoracic Imaging: Case Review Series, Third Edition. Similar to the third edition of Thoracic Imaging: The Requisites, this Case Review book has been extensively edited to include the latest updates in the expanding subspecialty of thoracic imaging. This edition has been completely rewritten and supplemented with hundreds of new images. Firstly, it maintains the two primary goals of the previous editions. These include a case format presentation to illustrate and review the imaging features of disorders that span the spectrum of thoracic diseases. Secondly, it is our goal to help the reader develop a sound framework with which to approach imaging interpretation in thoracic radiology. The evolution of complex thoracic diseases and treatments, including lung cancer, interstitial lung disease, and heart and lung transplant, must be emphasized in both residency and fellowship curricula. Lung cancer screening continues to expand to more eligible patients with risk factors for lung cancer. The critical role of radiology has been emphasized, and the art and
science of radiologic staging is illustrated in several cases. Pulmonary nodule management strategies and interstitial lung disease classications have been updated by multidisciplinary societies and are included in this edition. This update is composed of a diverse group of more than 140 cases using multimodality imaging from radiography to magnetic resonance imaging. As is standard for the Case Review Series, the book is organized by the level of difculty: Opening Round, Fair Game, and Challenge cases. Each case is accompanied by a series of multiple-choice questions with answers on the opposite page. Answers are followed by a brief discussion to highlight imaging features, differential diagnostic considerations, and key points from each case. Additional references are provided for those who desire more information about each topic, and cross-references to Thoracic Imaging: The Requisites are also provided. We hope that this latest edition of Thoracic Imaging will continue to serve as a valuable tool for lifelong learning of thoracic imaging. Justin T. Stowell, MD Jonathan H. Chung, MD Jeffrey P. Kanne, MD, FACR, FCCP Theresa C. McLoud, MD Gerald F. Abbott, MD, FACR
ix
Contents Section I
Opening Round, 1
Case 1
Lung Cancer, 1
Case 2
Spontaneous Pneumothorax, 2
Case 3
Pleural Plaques, 3
Case 4
Emphysema: Chest Radiography, 4
Case 5
Case 33
Hemothorax, 33
Case 34
Rib Notching, 34
Section II
Fair Game, 35
Case 35
Left Upper Lobe Atelectasis (Lung Cancer), 35
Sarcoidosis, 5
Case 36
Interstitial Edema, 36
Case 6
Mesothelioma, 6
Case 37
Thyroid Goiter, 37
Case 7
Junction Lines, 7
Case 38
Community-Acquired Bacterial Pneumonia, 38
Case 8
Hamartoma, 8
Case 39
Pulmonary Langerhans Cell Histiocytosis, 39
Case 9
Cystic Fibrosis, 9
Case 40
Lymphangitic Carcinomatosis, 40
Case 10
Mediastinal Mass (Aneurysm or Other Vascular Lesion), 10
Case 41
Rounded Atelectasis, 41
Case 11
Pulmonary Contusion and Laceration, 11
Case 42
Sarcoidosis (Stage IV), 42
Case 12
Malpositioned Catheter, 12
Case 43
Emphysema on CT (Centrilobular), 43
Case 13
Abnormal Paraspinal Line on Radiography, 13
Case 44
Anterior Mediastinal Mass (Thymoma), 44
Case 14
Miliary Tuberculosis, 14
Case 45
Subcarinal Lymph Node Enlargement Secondary to Metastatic Disease, 45
Case 15
Neurobromas, 15
Case 46
Silicosis, 46
Case 16
Left Lower Lobe Atelectasis, 16
Case 47
Case 17
Pulmonary Hypertension, 17
Progressive Systemic Sclerosis (Scleroderma), 47
Case 18
Lung Abscess, 18
Case 48
Ranke Complex, 48
Case 19
Pneumomediastinum, 19
Case 49
Bronchiectasis, 49
Case 20
Subpulmonic Pleural Effusion, 20
Case 50
Arteriovenous Malformation, 50
Case 21
Pneumothorax on Supine Radiograph, 21
Case 51
Aortic Aneurysm, 51
Case 22
Loculated Pleural Fluid in the Major Fissure, 22
Case 52
Pulmonary Infarction, 52
Case 53
Lymphoma, 53
Case 23
Esophageal Cancer, 23
Case 54
Traumatic Aortic Injury, 54
Case 24
Calcied Granuloma, 24
Case 55
Lipoid Pneumonia, 55
Case 25
Radiation Pneumonitis, 25
Case 56
Kartagener Syndrome, 56
Case 26
Acute Respiratory Distress Syndrome (ARDS), 26
Case 57
Invasive Aspergillosis, 57
Case 27
Cavitary Tuberculosis, 27
Case 58
Granulomatosis With Polyangiitis, 58
Case 28
Mycetoma, 28
Case 59
Lymphangioleiomyomatosis, 59
Case 29
Pericardial Cyst, 29
Case 60
Tuberculosis (Lymphadenopathy), 60
Case 30
Pneumonia, 30
Case 61
Pneumocystis jirovecii Pneumonia, 61
Case 31
Empyema, 31
Case 62
Ossied Lymph Nodes Secondary toMetastatic Osteosarcoma, 62
Case 32
Neurogenic Tumor (Schwannoma), 32
Case 63
Septic Infarcts, 63 xi
xii
Contents
Case 64
Bronchopleural Fistula, 64
Case 99
Case 65
Superior Sulcus Tumor, 65
Case 66
Post-Primary “Reactivation” Tuberculosis (TB), 66
Case 67
Mediastinal and Hilar Lymphadenopathy (Lung Cancer), 67
Case 68
Subsolid Nodule (Adenocarcinoma), 68
Case 69
Azygos Continuation of the Inferior Vena Cava, 69
Case 70
Right Upper Lobe Atelectasis, 71
Case 104 Allergic Bronchopulmonary Aspergillosis, 105
Case 71
Traumatic Rupture of the Left Hemidiaphragm, 72
Case 106 Solitary Fibrous Tumor of the Pleura, 107
Case 72
Persistent Left Superior Vena Cava, 73
Case 107 Complete Lung Collapse (Lung Cancer), 108
Case 73
Aspiration, 74
Case 74
Nonspecic Interstitial Pneumonia(NSIP), 75
Case 108 Mycobacterium Avium Complex (MAC) Infection, 109
Case 75
Ascending Aortic Aneurysm, 76
Case 76
Carcinoid, 77
Case 77
Boerhaave Syndrome, 78
Case 78
Bronchiectasis, 79
Case 111 Mosaic Attenuation (Small Airways Disease), 112
Case 79
Bulla (Giant), 80
Case 112 Chronic Beryllium Disease, 113
Case 80
Combined RML and RLL Atelectasis (Endobronchial Metastases), 81
Case 113 Chronic Eosinophilic Pneumonia, 114
Case 81
Lipoma, 82
Case 82
Apical Cap, 83
Case 115 Sternal Dehiscence Following MedianSternotomy, 116
Case 83
Bleomycin Drug Toxicity, 84
Case 116 Mediastinitis, 117
Case 84
Lung Cancer With N2 Nodal Disease, 85
Case 117 Organizing Pneumonia, 118
Case 85
Superior Vena Cava (SVC) Syndrome, 86
Case 118 Tracheobronchial Amyloidosis, 119
Case 86
Pulmonary Hemorrhage, 87
Case 119 Constrictive Bronchiolitis, 120
Case 87
Idiopathic Pulmonary Fibrosis (IPF), 88
Case 120 Metastasizing Leiomyoma, 121
Case 88
Chest Wall Mass: Hematoma, 89
Case 121 Asbestosis, 122
Case 89
Scimitar Sign (Hypogenetic Lung Syndrome), 90
Case 122 Mounier-Kuhn Syndrome, 123
Case 90
Pericardial Effusion, 91
Case 91
Bronchiectasis, 92
Case 92
Achalasia, 93
Case 93
Coccidioidomycosis, 94
Case 94
Hydrostatic Pulmonary Edema, 95
Case 95
Internal Mammary Lymph Node Enlargement, 96
Case 96
Computed Tomography (CT)-Guided Transthoracic Needle Biopsy (TTNB), 97
Case 97
Bronchial Atresia, 98
Case 130 Relapsing Polychondritis, 131
Case 98
Hypersensitivity Pneumonitis, 99
Case 131 Swyer-James-Macleod Syndrome, 132
Solitary Pulmonary Nodule: Lung Cancer Screening, 100
Case 100 Tracheal Stenosis, 101 Case 101 Thymic Hyperplasia, 102 Case 102 Part Solid Nodule, 103
Section III Challenge, 104 Case 103 Pulmonary Alveolar Proteinosis, 104
Case 105 Mucoid Impaction (Obstructing Tumor), 106
Case 109 Sarcoidosis (Perilymphatic Nodules), 110 Case 110 Small Airways Disease (InfectiousBronchiolitis), 111
Case 114 Tracheal Diverticulum, 115
Case 123 Tracheomalacia, 124 Case 124 Adenoid Cystic Carcinoma of the Trachea, 125 Case 125 Nocardia Infection (Heart TransplantPatient), 126 Case 126 Chronic Pulmonary Embolism, 127 Case 127 Metastatic Thyroid Carcinoma, 128 Case 128 Castleman Disease (Benign Lymph Node Hyperplasia), 129 Case 129 Aortic Dissection, 130
Contents
xiii
Case 132 Accessory Cardiac Bronchus, 133
Case 142 Pulmonary Venoocclusive Disease, 143
Case 133 Intralobar Sequestration, 134
Case 143 Tracheobronchial Papillomatosis, 144
Case 134 Acute Pulmonary Embolism, 135
Case 144 Echinococcal Cysts, 145
Case 135 Panlobular Emphysema (Alpha1-Antitrypsin Deciency), 136
Case 145 Congenital Pulmonary Airway Malformation (CPAM), 146
Case 136 Thymic Cyst, 137
Case 146 Lymphoid Interstitial Pneumonia (LIP), 147
Case 137 Tracheal Bronchus, 138 Case 138 Amiodarone Drug Toxicity, 139 Case 139 Saphenous Vein Graft Aneurysm Following Coronary Artery Bypass Grafting (CABG) Surgery, 140
Section IV Answers, 148 Section V
Supplementary Figures, e1
Index, 232
Case 140 Infectious Bronchiolitis, 141 Case 141 Viral Pneumonia (Organ Transplant Patient), 142
Supplemental gures can be accessed at Elsevier eBooks+ (eBooks.Health.Elsevier.com)
SECTION I
Opening Round
Case 1 History: Hemoptysis. 1. Which of the following should be included in the differential diagnosis? (Figs. 1.1, 1.2, 1.3 and 1.4) (Choose all that apply.) A. Lung carcinoma B. Infection C. Silicosis D. Pulmonary sequestration 2. What is the most common histologic type of lung carcinoma? A. Adenocarcinoma B. Squamous cell carcinoma C. Small cell carcinoma D. Carcinoid 3. What is the greatest risk factor for developing lung carcinoma? A. Coal B. Asbestos
C. Smoking D. Obesity 4. Which factor best predicts 5-year survival for patients with lung carcinoma? A. Smoking history B. Histologic type C. Grade of neoplasm D. Stage at diagnosis
Fig. 1.1
Fig. 1.2
Fig. 1.3
Fig. 1.4
1
Case 2 History: A 28-year old man has acute chest pain while playing basketball. 1. What is the most likely diagnosis (Figs. 2.1 and 2.2)? A. Traumatic pneumatocele B. Pneumomediastinum C. Spontaneous tension pneumothorax D. Swyer-James-McLeod syndrome 2. What is the likely mechanism involved in producing a traumatic pneumatocele? A. Penetrating trauma to the lung B. Compression-decompression trauma causing rupture of small airways C. Extension of pneumomediastinum into lung parenchyma D. Expansion of a preexisting bulla 3. What is the causative pathologic process in Swyer-JamesMcLeod syndrome? A. Blunt force trauma B. Postinfectious bronchiolitis C. Congenital bronchial atresia D. Incomplete interlobar ssures 4. Which radiographic nding occurs in tension pneumothorax? (Choose all that apply.) A. Contralateral shift of the mediastinum B. Downward displacement of the ipsilateral hemidiaphragm C. Ipsilateral widening of intercostal space (between ribs) D. All of the above
Fig. 2.1
2
Fig. 2.2
Case 3 History: Asymptomatic 63-year-old man. 1. Based on the radiographic and computed tomography CT ndings (Fig.3.1), what is the most likely diagnosis? A. Fibrothorax B. Previous pleurodesis C. Asbestos-related pleural plaques D. Metastases 2. What percentage of patients diagnosed with the pleural malignancy caused by mesothelioma will have concomitant pleural plaques? A. 0% B. 5% C. 25% D. 100%
3. What is the characteristic latency period between a person’s occupational exposure to asbestos and the development of mesothelioma? A. 1–2 years B. 5–10 years C. 10–15 years D. 30–40 years 4. Which of the following occupations has been associated with asbestos exposure in past decades? (Choose all that apply.) A. Construction worker B. Fireghter C. Shipyard worker D. Textile mill worker E. All of the above
Fig. 3.1
3
Case 4 History: 60-year-old man with dyspnea. 1. Which of the following conditions is associated with hyperinated lungs? (Choose all that apply.) A. Centrilobular emphysema B. Fibrotic hypersensitivity pneumonitis C. Lymphangioleiomyomatosis D. Asthma 2. What is the characteristic etiology of Swyer-James-McLeod syndrome? A. Follicular bronchiolitis B. Postinfectious bronchiolitis C. Broncholithiasis with air trapping D. Bronchial atresia
3. Which of the following is not always a smoking-related disease? A. Centrilobular emphysema B. Paraseptal emphysema C. Panlobular emphysema D. Desquamative interstitial pneumonia (DIP) 4. Which of the following are characteristic radiographic ndings of advanced centrilobular emphysema in a patient who smokes (Fig.4.1)? (Choose all that apply.) A. Hyperination B. Hyperlucency of upper lung zones C. Rapid tapering and attenuation of pulmonary vessels in affected lung D. Flattened hemidiaphragms
Fig. 4.1
4
Case 5 History: Asymptomatic 29-year-old woman. 1. Based on the clinical setting and imaging ndings (Fig.5.1), what is the most likely diagnosis? A. Castleman disease B. Lymphoma C. Sarcoidosis D. Silicosis 2. Which of the following malignancies may manifest with metastatic mediastinal and hilar lymphadenopathy? (Choose all that apply.) A. Head and neck malignancies B. Melanoma C. Breast cancer
D. Genitourinary malignancies E. All of the above 3. Which of the following endemic fungal infections is most associated with lymphadenopathy? A. Blastomycosis B. Coccidioidomycosis C. Histoplasmosis 4. Which of the following may manifest with asymmetric bilateral hilar lymphadenopathy? (Choose all that apply.) A. Lymphoma B. Sarcoidosis C. Metastatic disease D. All of the above
Fig. 5.1
5
Case 6 History: 66-year-old man with weight loss. 1. Which of the following imaging features are characteristic of pleural malignancy (Figs.6.1A and 6.1B)? (Choose all that apply.) A. >1 cm in thickness B. Nodularity C. Circumferential growth pattern within the involved hemithorax D. Involvement of the mediastinal pleural 2. Which of the following is most common? A. Mesothelioma B. Pleural lymphoma C. Pleural metastasis D. Solitary brous tumor of the pleura
Fig. 6.1A
6
3. What percentage of patients with pleural mesothelioma also have pleural plaques as a manifestation of prior asbestos exposure? A. 10% B. 25% C. 50% D. 100% 4. Which of the following is the most common histologic subtype of mesothelioma? A. Epithelioid B. Sarcomatoid C. Adenomatoid D. Mixed (biphasic)
Fig. 6.1B
Case 7 History: 28-year-old man with cough. 1. Based on the structures indicated (arrow and arrowhead Fig.7.1), what is the most likely diagnosis? A. Pneumomediastinum B. Pneumothorax C. Esophageal dilatation D. Normal study 2. Which of the following etiologies are associated with pneumomediastinum? (Choose all that apply.) A. Inhalational drug use B. Weightlifting C. Obstructive lung disease D. Pulmonary brosis
3. In Dr. Benjamin Felson’s original system for evaluating mediastinal abnormalities on lateral chest radiography, which of the following courses of a boundary line separates the anterior mediastinum from the middle mediastinum? A. Anterior to the trachea and anterior to the heart B. Anterior to the trachea and posterior to the heart C. Posterior to the trachea and anterior to the heart D. Posterior to the trachea and posterior to the heart 4. Which of the following radiographic ndings typically occur in total atelectasis of the right or left lung? A. Contralateral displacement of the anterior junction line B. Ipsilateral displacement of the anterior junction line C. Downward displacement of the ipsilateral hemidiaphragm D. Downward displacement of the contralateral hemidiaphragm
Fig. 7.1
7
Case 8 History: 70-year-old man with 30-pack year history of cigarette smoking. Finding detected on initial lung cancer screening computed tomography (CT) study (Fig.8.1). 1. What is the most likely diagnosis? A. Lepidic adenocarcinoma B. Carcinoid tumor C. Hamartoma D. Lipoid pneumonia 2. Which of the following may show uorodeoxyglucose (FDG)-avidity on positron emission tomography (PET) imaging? (Choose all that apply.) A. Lepidic adenocarcinoma B. Carcinoid tumor C. Lipoid pneumonia
D. Hamartoma E. All of the above 3. What is the range of fat attenuation on the Hounseld scale of CT numbers? A. -300 to -200 B. -200 to -100 C. -100 to -50 D. -40 to 0 4. What is the percentage of hamartomas detected by thinsection CT exhibit fat attenuation? A. 9% B. 29% C. 59% D. 79%
Fig. 8.1
8
Case 9 History: 22-year-old man with chronic cough. 1. Based on the radiographic ndings (Fig.9.1), what is the most likely diagnosis? A. Lymphangioleiomyomatosis B. Cystic brosis C. Centrilobular emphysema D. Pulmonary Langerhans cell histiocytosis 2. Which of the following computed tomography (CT) techniques is most helpful in the detection of bronchiectasis? A. Maximum-intensity projection images (MIPs) B. Minimum-intensity projection images (MinIPs) C. Multiplanar reformations D. Expiratory images
3. Which of the following conditions are associated with bronchial artery dilatation? (Choose all that apply.) A. Bronchiectasis B. Cavitary infection C. Cavitary sarcoidosis D. Lung cancer E. All of the above 4. Which of the following abnormalities are associated with cystic brosis? (Choose all that apply.) A. Mediastinal/hilar lymphadenopathy B. Pansinusitis C. Biliary cirrhosis D. Infertility E. All of the above
Fig. 9.1
9
Case 10 History: 66-year-old man with substernal chest pain. 1. Dr. Benjamin Felson introduced the concept of mediastinal compartments by using normal anatomic landmarks on the lateral chest radiograph. What is the course of the line Felson used to separate the anterior mediastinum from the middle mediastinum? A. Anterior to the trachea and anterior to the heart B. Anterior to the trachea and posterior to the heart C. Posterior to the trachea and anterior to the heart D. Posterior to the trachea and posterior to the heart 2. Which of the following diagnoses is most consistent with the radiographic nding of diffuse mediastinal widening (Fig.10.1)? A. Thymoma B. Bronchogenic cyst
C. Metastatic lymphadenopathy D. Esophageal varices 3. What percentage of mediastinal abnormalities are vascular lesions? A. 10% B. 20% C. 30% D. 40% 4. Which of the following are most characteristic of patient age encountered in cases of thymoma and teratoma? A. Over and under the age of 20 years, respectively B. Under and over the age of 40 years, respectively C. Over and under the age of 40 years, respectively D. Under and over the age of 20 years, respectively
Fig. 10.1
10
Case 11 History: 46-year-old man was struck by a motorcycle. 1. What is the most likely cause of the ground-glass opacity and consolidation in the left lung (Figs.11.1A and 11.1B)? A. Aspiration B. Atelectasis C. Pulmonary contusion D. Lung cancer 2. What is the most likely cause of the focal lucency within the parenchymal consolidation? A. Bronchiectasis B. Bullous emphysema C. Post-traumatic pneumatocele D. Cystic neoplasm
Fig. 11.1A
3. Pulmonary contusion should begin to resolve within how much time from injury? A. 4 hours B. 12 hours C. 24 hours D. 48 hours 4. Which of the following may manifest as cysts on computed tomography (CT)? A. Desquamative interstitial pneumonia (DIP) B. Lymphocytic interstitial pneumonia (LIP) C. Pulmonary Langerhans cell histiocytosis (PLCH) D. All of the above
Fig. 11.1B
11
Case 12 History: Recent central line placement. 1. Where does the central line terminate (Figs.12.1 and 12.2)? A. Superior vena cava (SVC) B. Azygos vein C. Great cardiac vein D. Brachiocephalic vein 2. What is the ideal location for the termination of a central venous line in most cases? A. Subclavian vein B. Azygos vein C. Brachiocephalic vein D. Cavoatrial junction 3. What is the most likely complication of a central venous line placed in the right atrium? A. Cardiac arrhythmia B. Cardiac thrombus
Fig. 12.1
12
C. Cardiac rupture D. Cardiac tamponade 4. A subclavian central catheter is placed for administration of chemotherapy for lung cancer. Pulsatile, bright, red blood is noted from the catheter immediately after placement. Of the listed choices, what is the next best step in management? A. Use the catheter for drug administration B. Remove the line immediately C. Contact vascular surgery D. Vigorously ush the catheter See Supplemental Figures section for additional gures for this case. [E-book/Inkling: Link to supplemental gures, Case 12]
Fig. 12.2
Case 13 History: History withheld. 1. Which of the following could cause this imaging nding (Figs. 13.1, 13.2 and 13.3)? A. Achalasia B. Spinal fracture C. Metastatic disease D. Aortic aneurysm 2. What classic sign of posterior mediastinal lesions is present in this case? A. Cervicothoracic sign B. Thoracoabdominal sign C. Cervicoabdominal sign D. Cervicothoracoabdominal sign 3. What is the most sensitive modality to detect acute spinal fractures? A. Radiography B. Fluoroscopy C. Computed tomography (CT) D. Magnetic resonance imaging (MRI) 4. An increase in tube potential does what to radiation dose? A. Increase on radiography and CT B. Decrease on radiography and CT C. Increase on radiography but decrease on CT D. Decrease on radiography but increase on CT
Fig. 13.2
Fig. 13.1
Fig. 13.3
13
Case 14 History: 53-year-old man with cough and shortness of breath. 1. Which of the following should be included in the differential diagnosis for this patient (Figs.14.1 and 14.2)? (Choose all that apply.) A. Emphysema B. Pulmonary edema C. Tuberculosis D. Metastases 2. The computed tomography (CT) chest pattern is best described as: A. Tree-in-bud B. Random C. Perilymphatic D. Centrilobular 3. A tree-in-bud pattern on CT is most suggestive of which of the below conditions? A. Aspiration B. Edema
Fig. 14.1
14
C. Metastases D. Hemorrhage 4. What does a random pattern of diffuse nodular lung disease imply? A. Lymphatic spread B. Hematogenous spread C. Aerogenous spread D. Mixed spread See Figure S14.1. See Supplemental Figures section for additional gures for this case. [E-book/Inkling – Link to supplemental gures Case 14]
Fig. 14.2
Case 15 History: History withheld. 1. Which of the following should be included in the differential diagnosis for this patient (Figs.15.1A and 15.1B)? (Choose all that apply.) A. Pulmonary metastases B. Neurobromatosis C. Pleural metastases D. Pulmonary histoplasmosis 2. What imaging sign is exemplied in this case example? A. Incomplete border B. Hilar overlay C. Cervicothoracic D. Monod
Fig. 15.1A
3. Which of the following is least common in neurobro matosis 1? A. Caféaulait spots B. Scoliosis C. Acoustic neuromas D. Optic gliomas 4. Two symmetrically located nodules are noted overlying the lower thorax; both are welldened laterally but poorly dened medially. What is the next step in management? A. Inandout of phase magnetic resonance imaging (MRI) B. Chest computed tomography (CT) with contrast C. Chest radiograph with BB markers D. Positron emission tomography (PET)/CT
Fig. 15.1B
15
Case 16 History: Outpatient radiograph in a 55-year-old man with subacute cough. 1. Based on the radiograph (Fig. 16.1), which of the following should be included in the differential diagnosis? A. Pneumonia B. Atelectasis C. Pleural effusion D. Chest wall mass 2. What is the most likely cause of this condition in this man (Fig. 16.2)? A. Aspiration B. Hypoventilation C. Lung cancer D. Carcinoid tumor
3. What would be the most likely cause of this condition in a child? A. Aspiration B. Hypoventilation C. Lung Cancer D. Carcinoid tumor 4. Which of the following signs is associated with this condition? A. Cervicothoracic sign B. S sign of Golden C. Luftsichel sign D. Flat waist sign
Fig. 16.2
Fig. 16.1
16
Case 17 History: Chronic dyspnea. 1. What is the diagnosis (Fig.17.1)? A. Pulmonary hypertension B. Aortic dissection C. Pulmonary stenosis D. Sarcoidosis 2. Which of the following is not a cause for this condition? A. Left-to-right shunt B. Mitral stenosis C. Calcium channel blockers D. Sleep apnea 3. What is the upper limit of normal for main pulmonary artery size on computed tomography (CT)? A. 2 cm B. 2.5 cm C. 3 cm D. 3.5 cm 4. Which of the following is a secondary cause of pulmonary hypertension? A. Portopulmonary hypertension B. Renopulmonary hypertension C. Adrenopulmonary hypertension D. Splenopulmonary hypertension Fig. 17.1
17
Case 18 History: Middle-aged man with fever and cough. 1. What is the differential diagnosis for cavitary nodule(s) (Fig.18.1)? A. Vasculitis B. Necrobiotic nodules C. Metastatic disease D. Abscess 2. Which primary tumor is least likely to cause cavitary lung metastases? A. Sarcoma B. Transitional cell carcinoma C. Melanoma D. Squamous cell carcinoma
3. What is the most common predisposing cause or factor in formation of a lung abscess (Fig.18.2)? A. Trauma B. Pulmonary embolism C. Aspiration D. Air-trapping 4. In which lung segment is a lung abscess least likely to develop? A. Anterior segment, right upper lobe B. Superior segment, right lower lobe C. Posterior segment, right upper lobe D. Lateral basilar segment, right lower lobe
Fig. 18.1
Fig. 18.2
18
Case 19 History: Patient with long history of asthma with new-onset chest pain. 1. Which of the following could be a cause this imaging nding (Figs.19.1 and 19.2)? A. Asthma B. Esophageal rupture C. Ruptured bleb D. Tracheal rupture 2. Which of the following ectopic gas collections will not shift with patient position? A. Pneumomediastinum B. Pneumoperitoneum C. Pneumothorax D. Pneumopericardium
Fig. 19.1
3. Which anatomic landmark represents the upper limit of visualized gas in a pneumopericardium? A. Tracheal carina B. Left atrial appendage C. Aortic arch D. Coronary sinus 4. Which of the following signs can be seen in either pneumomediastinum or pneumopericardium? A. Naclerio V sign B. Continuous diaphragm sign C. Deep sulcus sign D. Ring around the artery sign
Fig. 19.2
19
Case 20 History: Acute onset shortness of breath. 1. Which of the following is the most likely cause this imaging nding (Fig.20.1)? A. Liver mass B. Pleural effusion C. Diaphragmatic paralysis D. Diaphragmatic eventration 2. What is the most sensitive radiographic technique to detect pleural effusion? A. Oblique B. Posteroanterior (PA) C. Anteroposterior (AP) D. Lateral decubitus 3. What is the most common cause of transudative pleural effusion? A. Congestive heart failure B. Nephrotic syndrome C. Hepatic hydrothorax D. Hypoalbuminemia 4. What is the most common cause of exudative pleural effusion? A. Pneumonia B. Lung cancer C. Pulmonary infarct D. Collagen vascular disease
20
Fig. 20.1
Case 21 History: Decreased left-sided breath sounds on auscultation. 1. Which of the following could cause this imaging nding (Figs.21.1 and 21.2)? A. Pneumothorax B. Pleural effusion C. Pneumoperitoneum D. Basilar emphysema 2. What is the most sensitive radiographic technique to detect pneumothorax? A. Oblique B. Posteroanterior (PA) C. Expiration PA D. Lateral decubitus
Fig. 21.1
3. In a young patient, what is the most common cause of pneumothorax? A. Primary spontaneous B. Secondary spontaneous C. Traumatic D. Iatrogenic 4. Which sign is diagnostic of a pneumothorax on supine radiographs? A. Ring around the artery sign B. Deep sulcus sign C. Westermark sign D. Air crescent sign
Fig. 21.2
21
Case 22 History: History of pulmonary hypertension. 1. What is the location of the abnormality on the right (Figs.22.1, 22.2 and 22.3)? A. Right upper lobe B. Right middle lobe C. Right lower lobe D. None of the above 2. Which of the following is the most likely diagnosis? A. Lung cancer B. Pulmonary hamartoma C. Pseudotumor D. Metastatic disease 3. What is the most common malignancy to involve the pleura? A. Mesothelioma B. Solitary brous tumor C. Metastatic disease D. Sarcoma 4. What is the most common type of accessory pulmonary ssure? A. Azygos ssure B. Superior accessory ssure C. Inferior accessory ssure D. Left minor ssure Fig. 22.2
Fig. 22.1
22
Fig. 22.3
Case 23 History: History of dysphagia and gastroesophageal reux. 1. What diagnoses should be included in the differential diagnosis based on the scout image (Figs.23.1 and 23.2)? A. Esophageal cancer B. Lung cancer C. Bronchogenic cyst D. Thymoma 2. Which of the following is true regarding uorodeoxyglucose (FDG)-positron emission tomography (PET) FDGPET imaging in the setting of esophageal cancer? A. High accuracy for T staging B. High accuracy for N staging C. High accuracy for M staging D. Not useful for TNM staging 3. What is the most common process causing abnormal convexity along the inferior aspect of the azygoesophageal recess? A. Hiatal hernia B. Left atrial dilation C. Esophageal cancer D. Lymphadenopathy 4. Regarding the differences between squamous cell carcinoma (SCC) and adenocarcinoma (AC) of the esophagus, which of the following is true? A. SCC is more common in the lower third of the esophagus than AC. B. AC is more common than SCC world-wide. C. The most common risk factor for AC is gastroesophageal reux. D. SCC has a better prognosis than AC.
Fig. 23.1
Fig. 23.2
Fig. 23.3
23
Case 24 History: Preoperative evaluation. 1. Which of the following should be included in the differential diagnosis for this patient (Fig.24.1)? (Choose all that apply.) A. Hamartoma B. Adenocarcinoma C. Calcied granuloma D. Calcied pleural plaque 2. Which of the following is not a benign pattern of calcication in smoothly marginated lung nodules? A. Diffuse B. Central C. Stippled D. Laminar 3. Which of the following features on computed tomography (CT) is the most suggestive that a solitary lung nodule is benign? A. Eccentric calcication B. Smooth margins C. Mixed solid and ground-glass attenuation D. Fat attenuation 4. Which of the following characteristics is associated with the highest likelihood ratio that a solitary lung nodule is malignant? A. Size >3 cm B. Spiculated margins C. Doubling time less than 30 days D. Upper lobe location
24
Fig. 24.1
Case 25 History: Lung cancer. 1. Which of the following should be included in the differential diagnosis for this patient (Figs.25.1 and 25.2)? (Choose all that apply.) A. Sarcoidosis B. Idiopathic pulmonary brosis C. Radiation pneumonitis D. Infectious pneumonia 2. At what time point following completion of radiation therapy is radiation pneumonitis usually detectable by chest radiography? A. >1 week B. 2–4 weeks C. 6–8 weeks D. >8 weeks
3. Which of the following factors plays the least direct role in contributing to risk for developing radiation pneumonitis? A. Tumor histology B. Concomitant chemotherapy C. Radiation dose D. Size of tumor 4. Which of the following statements regarding radiationinduced lung disease is true? A. Radiation pneumonitis is a prerequisite for developing radiation brosis. B. A normal chest radiograph excludes the diagnosis of radiation pneumonitis. C. Radiation brosis is a progressive condition that often results in respiratory failure. D. Radiation brosis usually develops 6 to 12 months after the completion of therapy.
Fig. 25.1
Fig. 25.2
25
Case 26 History: Hypxia and failure t wean frm ventilatr. 1. Which f the fllwing shuld be included in the differential diagnsis fr this patient (Fig.26.1)? (Chse all that apply.) A. Acute respiratry distress syndrme (ARDS) B. Aspiratin C. Sarcidsis D. Bacterial pneumnia 2. Aside frm diffuse lung pacities, what ther abnrmality is present n this patient’s chest radigraph? (Chse all that apply.) A. Pneummediastinum B. Pneumpericardium C. Pneumperitneum D. Pneumthrax
Fig. 26.1
26
3. What is the mst likely cause f pneumthrax in this patient? A. Central venus catheter placement B. Bartrauma C. Brnchial rupture D. Pulmnary laceratin 4. Which f the fllwing is not a criterin fr the diagnsis f ARDS? A. Pao2/Fio2 ≤300 mmHg B. Bilateral lung pacity n chest radigraphy C. Hypxemic respiratry failure within ne week f insult D. Pleural effusin
Case 27 History: 55-year-old man with worsening cough and weight loss. Query lung carcinoma. 1. Which of the following should be included in the differential diagnosis of this patient (Figs.27.1 and 27.2)? (Choose all that apply.) A. Lung cancer B. Granulomatosis with polyangiitis C. Sarcoidosis D. Tuberculosis 2. Which cell type of lung cancer is most closely associated with the presence of cavitation? A. Adenocarcinoma B. Small cell carcinoma C. Squamous cell carcinoma D. Large cell carcinoma
Fig. 27.1
3. Which one of the following is most suggestive of a malignant cavitary lesion? A. Thick (>4 mm) wall B. Intracavitary uid level C. Development in a preexisting area of consolidation D. Lower lobe location 4. Which of the following is the next best step in the management of this patient? A. Bronchoscopy with bronchoalveolar lavage B. Transthoracic ne needle aspiration C. Respiratory isolation D. Fluorodeoxyglucose (FDG)-positron emission tomography (PET)/computed tomography (CT)
Fig. 27.2
27
Case 28 History: Hemoptysis. 1. Which of the following should be included in the differential diagnosis for this patient (Figs.28.1, 28.2 and 28.3)? (Choose all that apply.) A. Lung carcinoma B. Progressive massive brosis C. Mycetoma D. Pulmonary Langerhans cell histiocytosis 2. Which organism is most commonly associated with mycetoma? A. Aspergillus B. Blastomyces C. Candida D. Coccidioides 3. What one of the following is the most common complication of a mycetoma? A. Spread of infection to others B. Hemoptysis C. Dissemination of infection D. Lung carcinoma 4. Which of the following is preferred rst-line therapy for patients with mycetoma and acute severe hemoptysis? A. Antifungal therapy B. Pneumonectomy C. Bronchial artery embolization D. Radiation therapy Fig. 28.2
Fig. 28.1
28
Fig. 28.3
Case 29 History: 33-year-old man presenting for employment physical examination. 1. Based on the chest radiograph (Fig.29.1), which of the following should be included in the differential diagnosis for this patient? (Choose all that apply.) A. Pericardial cyst B. Lymphoma C. Paraganglioma D. Morgagni hernia 2. Based on the computed tomography (CT) scan image, what one of the following is the best diagnosis for this patient (Fig.29.2)? A. Pericardial cyst B. Lymphoma C. Paraganglioma D. Morgagni hernia
Fig. 29.1
3. Which of the following is true regarding pericardial cysts? A. Pericardial cysts occur more commonly in the left pericardiophrenic space. B. Most pericardial cysts communicate with the pericardium. C. Most pericardial cysts attach to the parietal pericardium. D. Most pericardial cysts cause symptoms. 4. Which of the following MR = magnetic resonance ndings of pericardial cysts is correct? A. Most pericardial cysts have high signal intensity on T1-weighted imaging. B. T2-signal intensity is heterogeneous. C. T1 postcontrast images show thin capsular enhancement. D. T1 postcontrast images show no internal enhancement.
Fig. 29.2
29
Case 30 History: 20-year-old female with fever and cough. 1. Which of the following should be included in the differential diagnosis for this patient? (Choose all that apply.) A. Acute histoplasmosis B. Community-acquired pneumonia C. Pleural effusion D. Septic emboli 2. What is the term used to describe the air-lled, tubular, branching structures that are visible within the right lower lobe (Figs.30.1 and 30.2)? A. Air bronchogram B. Pseudocavitation C. Pulmonary interstitial emphysema D. Black bronchus sign
Fig. 30.1
30
3. Which organism is most often associated with lobar pneumonia in the normal (immunocompetent) host? A. Aspergillus fumigatus B. Legionella pneumophila C. Staphylococcus aureus D. Streptococcus pneumoniae 4. What sign is used to describe obscuration of the right hemidiaphragm on the posteroanterior (PA) view? A. Sail sign B. Spine sign C. Silhouette sign D. Continuous hemidiaphragm sign
Fig. 30.2
Case 31 History: Leukocytosis and cough. 1. Which of the following should be included in the differential diagnosis for this patient (Fig.31.1)? (Choose all that apply.) A. Chylothorax B. Empyema C. Malignant effusion D. Hemothorax 2. Which of the following best suggests empyema on computed tomography (CT)? A. Gravitational layering of uid B. Lenticular shape of pleural collection C. Gas within the pleural collection D. Compressive atelectasis of adjacent lung 3. What is the most likely cause of an air uid level within a pleural uid collection? A. Gas-forming organisms B. Pulmonary gangrene C. Pleurocutaneous stula D. Bronchopleural stula 4. What is the most likely type of organism causing this abnormality? A. Bacterial B. Fungal C. Parasitic D. Viral
Fig. 31.1
31
Case 32 History: Chest pain. 1. Which of the following should be included in the differential diagnosis for this patient (Figs.32.1, 32.2 and 32.3)? (Choose all that apply.) A. Schwannoma B. Thymoma C. Ganglioneuroma D. Germ cell tumor 2. In which mediastinal compartment is the mass? A. Anterior mediastinum B. Middle mediastinum C. Posterior mediastinum D. Inferior mediastinum 3. Which of the following is the next best step in management of this patient? A. Chest computed tomography (CT) B. Thoracic spine magnetic resonance imaging (MRI) C. Fluorodeoxyglucose-positron emission tomography (FDG-PET)/CT D. Biopsy 4. Which of the following regarding neurogenic tumors is true? A. Most neurogenic tumors are malignant. B. Rib spreading and erosions imply malignancy. C. Tumors of the sympathetic chain often widen the neural foramen. D. Calcication is more common in sympathetic chain tumors than peripheral nerve tumors.
Fig. 32.1
Fig. 32.3
Fig. 32.2
32
Case 33 History: 50-year-old male with dyspnea and cough with increasing chest pain three days after slipping on ice. 1. Which of the following should be included in the differential diagnosis for this patient (Fig.33.1)? (Choose all that apply.) A. Hemothorax B. Empyema C. Chylothorax D. Solitary brous tumor 2. Which of the following most likely suggests hemothorax? A. Rapidly enlarging pleural collection following heart surgery B. Slowly enlarging pleural collection over several weeks C. Pleural collection loculated in the major ssure D. Homogeneous, low-attenuation pleural collection on computed tomography (CT)
3. What sign describes layers of different attenuation in a hemothorax? A. Split pleura sign B. Dependent viscera sign C. Collar sign D. Hematocrit sign 4. What is the next best management for this hemodynamically stable patient? A. Thoracotomy B. Tube thoracostomy C. Transcatheter embolization D. Talc pleurodesis
Fig. 33.1
33
Case 34 History: Multiple skin lesions. 1. Which of the following should be included in the differential diagnosis for this patient (Fig.34.1)? (Choose all that apply.) A. Tuberous sclerosis complex B. Metastases C. Neurobromatosis type 1 (NF-1) D. Takayasu arteritis 2. Which of the following is not an osseous manifestation of NF-1? A. Widened neural foramen B. Rib erosion C. Vertebra plana D. Scoliosis 3. What is the most common cause of destruction of a rib in an adult? A. Primary bone sarcoma B. Metastases C. Multiple myeloma D. Metabolic bone disease 4. What congenital aortic anomaly is associated with rib notching? A. Right aortic arch B. Cervical aortic arch C. Aberrant subclavian artery D. Aortic coarctation
34
Fig. 34.1
SECTION II
Fair Game
Case 35 History: 60-year-old man with cough. 1. Which radiographic sign is demonstrated (Fig.35.1)? A. S-sign of Golden B. Rock of Gibraltar C. Luftsichel sign D. Flat waist sign 2. In an adult outpatient, which is the most likely cause for lobar atelectasis? A. Aspirated foreign body B. Endobronchial mucus plug C. Lung cancer D. Bronchomalacia
Fig. 35.1
3. In an adult inpatient, which is the most likely cause for lobar atelectasis? A. Aspirated foreign body B. Endobronchial mucus plug C. Lung cancer D. Bronchomalacia 4. The revised 8th edition TNM Staging Classication for Lung Cancer considers any degree of post-obstructive atelectasis or pneumonitis extending to the hilum to represent at least what T (tumor) stage? A. T1 B. T2 C. T3 D. T4
Fig. 35.2
35
Case 36 History: Shortness of breath in 48-year-old man. 1. Based on the radiographic appearance, what is the most likely diagnosis (Fig.36.1)? A. Cardiogenic pulmonary edema B. Noncardiogenic pulmonary edema C. Lymphangitic carcinomatosis D. Community acquired pneumonia 2. What of the following is not a typical radiologic feature of this entity? A. Architectural distortion B. Perihilar haze C. Septal thickening D. Cardiomegaly 3. What is the most common cause for this radiographic appearance? A. Drug toxicity B. Left heart disease
Fig. 36.1
36
C. Neurologic disease D. Inhalational injury 4. Which of the following is true regarding the imaging features of this entity? (Choose all that apply.) A. Associated with increased hydrostatic pressure in pulmonary capillaries B. Affects the axial, peripheral, and parenchymal interstitium C. Widening of the vascular pedicle to greater than 58 mm is a marker for increased central vascular pressure D. Radiographic improvement may lag behind clinical parameters E. All of the above
Fig. 36.2
Case 37 History: Elderly woman with incidental nding on chest radiography. 1. What radiographic sign is demonstrated (Fig.37.1)? A. Thoracoabdominal sign B. Hilum convergence sign C. Hilum overlay sign D. Cervicothoracic sign 2. In which radiographic space is the abnormality located on the lateral radiograph (Fig.37.2)? A. Retrotracheal space B. Retrosternal space C. Infrahilar window D. Paravertebral space 3. In which anatomic compartment is the abnormality located? A. Prevascular mediastinum B. Visceral mediastinum C. Paravertebral mediastinum D. Pleural space
Fig. 37.1
4. Which of the following imaging features of this entity might prospectively predict the need for sternotomy in the surgical management of this lesion? A. Tracheal deviation B. Visceral mediastinal extension C. Paravertebral mediastinal extension D. Subcarinal extension E. Intrathoracic component larger than thoracic inlet F. Dumbbell or “conical” shape G. B, C, D, E, and F H. All of the above
Fig. 37.2
37
Case 38 History: 68-year-old female with otherwise noncontributory medical history presents with 3 days of fever and cough. 1. In which lobe of the lung is the abnormality located (Figs.38.1 and 38.2)? A. Right upper lobe B. Right lower lobe C. Right middle lobe D. Left upper lobe E. Left lower lobe 2. Which imaging sign is demonstrated on the radiograph (Figs.38.1 and 38.2)? (Choose all that apply.) A. Spine sign B. Halo sign C. Reverse halo sign D. Silhouette sign
3. Which of the following organisms is most likely responsible for these patterns? A. Mycoplasma pnemoniae B. Bacillus anthracis C. Histoplasma capsulatum D. Streptococcus pneumoniae
Fig. 38.1
Fig. 38.2
38
Case 39 History: 33-year-old male smoker with progressive cough. 1. What is the most likely cause for the high-resolution computed tomography (HRCT) ndings (Figs.39.1 and 39.2)? A. Metastatic disease B. Usual interstitial pneumonia (UIP) C. Pulmonary Langerhans cell histiocytosis D. Sarcoidosis 2. Which pair of associations is typical of this entity? A. Skin folliculomas and renal cell carcinoma B. Intracranial subependymal tubers and renal angiomyolipomas C. Perilymphatic micronodules and hilar lymphadenopathy D. Smoking history and younger age
3. Which portion of the lung is typically spared in this disorder? A. Upper lobes B. Costophrenic sulci C. Apices D. Terminal bronchioles 4. Which of the following choices list the two most common complications of the disease? A. Emphysema and lung cancer B. Chylothorax and pneumothorax C. Pneumothorax and pulmonary hypertension D. Recurrent infections and bronchiectasis
Fig. 39.2 Fig. 39.1
39
Case 40 History: 62-year-old former smoker presents with dyspnea. 1. Based on the CT ndings, what is the most likely diagnosis (Fig.40.1)? A. Lymphangitic carcinomatosis B. Sarcoidosis C. Idiopathic pulmonary brosis D. Lymphoma 2. Which of the following is a high-resolution computed tomography (HRCT) feature of this condition? (Choose all that apply.) A. Smooth or nodular axial interstitial thickening B. Smooth or nodular interlobular septal thickening C. Smooth or nodular ssural thickening D. Preservation of normal lung architecture E. All the above
Fig. 40.1
40
3. Which of the following primary malignant neoplasms is associated with lymphangitic carcinomatosis? (Choose all that apply.) A. Colon B. Lung C. Breast D. Stomach E. All of the above 4. Which of the following histologic subtypes of malignancy is most likely to exhibit a lymphangitic pattern? A. Squamous cell carcinoma B. Sarcoma C. Adenocarcinoma D. Neuroendocrine cell
Case 41 History: 57-year-old asymptomatic man. 1. What should be included in the differential diagnosis for the parenchymal abnormality (Fig.41.1)? A. Pneumonia B. Pulmonary infarct C. Lung cancer D. Rounded pneumonia E. All of the above 2. Which computed tomography (CT) sign is demonstrated (Fig.41.2)? A. Reverse halo sign B. Comet tail sign C. Bronchus cut-off sign D. Dark bronchus sign 3. What is the most likely diagnosis for the parenchymal abnormality? A. Pneumonia B. Rounded atelectasis
Fig. 41.1
C. Lung cancer D. Pulmonary infarct 4. Which of the following features must be present to make a denitive imaging diagnosis? A. Rounded, wedge-shaped, or lentiform morphology of mass-like opacity B. Pleural abnormality in contact with the subpleural masslike opacity C. Signs of volume loss in the affected lobe D. Traction and convergence of adjacent bronchovascular structures E. All of the above
Fig. 41.2
41
Case 42 History: 44-year-old female with dyspnea on exertion. 1. Which of the following entities is associated with upper lung zone architectural distortion? A. Sarcoidosis B. Tuberculosis C. Pneumoconiosis (e.g., silicosis, berylliosis) D. Chronic hypersensitivity pneumonitis E. All of the above 2. Which radiographic stage of the diagnosis is depicted (Fig.42.1)? A. Stage 0 B. Stage I C. Stage II D. Stage III E. Stage IV
3. Which term is used to describe the airway abnormalities associated with pulmonary brosis? A. Traction bronchiectasis B. Honeycombing C. Bronchomalacia D. Bronchial stenosis E. Obliterative bronchiolitis 4. Which laboratory value is often elevated in patients with this condition? A. Alpha-1 antitrypsin B. Angiotensin converting enzyme (ACE) C. pCO2 D. Eosinophil count E. Cytoplasmic anti-neutrophil cytoplasmic antibody (c-ANCA)
Fig. 42.1
Fig. 42.2
Fig. 42.3
42
Case 43 History: 66-year-old female smoker. 1. What is the anatomic distribution of the abnormality (Figs.43.1 and 43.2)? A. Paraseptal B. Centrilobular C. Panlobular D. Paracicatricial 2. What is the most common cause of the abnormality? A. Autoimmune disease B. Drug reaction C. Smoking D. Positive airway pressure
3. What distribution is typical for this condition? A. Lower lobes B. Upper lobes C. Asymmetric D. Random 4. True or False? This condition is reversible with smoking cessation. A. True B. False
Fig. 43.1 Fig. 43.2
43
Case 44 History: Asymptomatic 66-year-old female with incidental nding on routine imaging. 1. In what mediastinal compartment is the abnormality located (Figs.44.1, 44.2 and 44.3)? A. Prevascular (anterior) B. Visceral (middle) C. Paravertebral (posterior) D. Superior 2. What is the most common primary neoplasm of the prevascular mediastinum? A. Thymoma B. Thymic carcinoma C. Thymic neuroendocrine tumor D. Lymphoma E. Germ cell neoplasm 3. At which age are thymomas typically encountered? A. 5 to 10 years B. 20 to 30 years C. 30 to 40 years D. >40 years 4. Which paraneoplastic syndrome is most commonly associated with thymoma? A. Myasthenia gravis B. Lambert-Eaton syndrome C. Syndrome of inappropriate antidiuretic hormone D. Serotonin syndrome
Fig. 44.1
44
Fig. 44.2
Fig. 44.3
Case 45 History: 67-year-old male smoker with weight loss. 1. What is the most likely diagnosis (Figs. 45.1A and 45.1B)? A. Bronchogenic cyst B. Hiatal hernia C. Mediastinal hematoma D. Lymphadenopathy 2. In which anatomic space is the abnormality located? A. Paraesophageal B. Subaortic C. Right paratracheal D. Subcarinal 3. In a patient with right upper lobe lung cancer, involvement of this level represents, at a minimum, which tumor, node, metastasis (TNM) nodal classication? A. N0 B. N1 C. N2 D. N3
4. Which interventional staging procedure(s) can be used to access and sample abnormalities in this location? A. Bronchoscopy B. Endobronchial ultrasound (EBUS) C. Esophageal endoscopic ultrasound (EUS) D. Mediastinoscopy E. all of the above
Fig. 45.1A
Fig. 45.1B
45
Case 46 History: Asymptomatic male. 1. What is the predominant imaging abnormality (Figs. 46.1A, 46.1B, 46.1C, 46.1D, and 46.1E)? A. Pleural plaques B. Nodules C. Masses D. Architectural distortion 2. Which of the following is most likely to have been the patient’s occupation? A. Business executive B. Airplane pilot C. Fireman D. Quarry worker
3. In the simple form of this disease, which represents the typical temporal relationship from the exposure to the disease manifestation? A. 1 to 2 hours B. 1 to 2 weeks C. 1 to 2 years D. 10 to 20 years 4. The International Labor Ofce has established guidelines for the objective classication of this entity using which of the following? A. Pulmonary function test B. Whispered pectoriloquy C. Digital chest radiography D. Chest computed tomography
Fig. 46.1C
Fig. 46.1A
Fig. 46.1D
46
Fig. 46.1B
Fig. 46.1E
Case 47 History: 50-year-old woman with progressive decline in lung function. 1. Which of the following descriptors should be excluded as a predominant nding in this case (Fig.47.1)? A. Peripheral B. Nodules C. Basilar predominant D. Ground-glass E. Subpleural sparing 2. Which of the following is the most likely cause for this computed tomography (CT) pattern in this patient? A. Cigarette smoking B. Connective tissue disease C. Hot tub use D. Asbestos exposure
3. Which of the following associated features shown may help narrow the differential diagnosis for the CT pattern in this patient? A. Traction bronchiectasis B. Coronary artery calcication C. Dilated pulmonary artery D. Esophageal dilation 4. Elevation of which serologic markers would be characteristic in this disease? A. Rheumatoid factor, anti-CCP B. Anti-ribonuclear protein C. Anti-centromere antibody, anti-SCL-70 D. Anti-SS-A (Ro), anti-SS-B (La)
Fig. 47.1
47
Case 48 History: 44-year-old asymptomatic female with positive puried protein derivative (PPD) skin test during pre-employment physical. 1. What is the most likely cause of the imaging abnormality (Figs. 48.1A and 48.1B)? A. Asbestosis B. Broncholith C. Calcied metastasis D. Healed tuberculosis 2. Which historical eponym is given to the parenchymal nding? A. Fleischner sign B. Ghon focus C. S-sign of Golden D. Hampton hump
Fig. 48.1A
48
3. Which historical eponym is given to the combination of the parenchymal and hilar nding? A. Ranke complex B. Ghon focus C. Kerley’s line D. Naclerio V-sign 4. Which of the following is included in the differential diagnosis for a calcied pulmonary nodule? (Choose all that apply.) A. Healed granulomatous disease B. Hamartoma C. Pneumoconiosis D. Carcinoid E. Calcic metastases F. Pulmonary ossication G. All of the above
Fig. 48.1B
Case 49 History: 49-year-old woman with chronic cough. 1. What is the most prominent imaging abnormality (Fig.49.1A)? A. Bronchiectasis B. Dilated pulmonary artery C. Middle lobe atelectasis D. Lingular atelectasis 2. Which of the following denes bronchiectasis by imaging criteria? A. Bronchoarterial ratio >0.5 B. Bronchoarterial ratio >1.0 C. Bronchoarterial ratio >1.5 D. Greater than 70% luminal cross-sectional area collapse on expiration E. Greater anteroposterior airway diameter than transverse diameter
Fig. 49.1A
3. Which computed tomography (CT) imaging sign is associated with this condition (Fig.49.1B)? A. Signet ring sign B. Three density sign C. Fallen viscera sign D. Reverse CT halo sign 4. Which of the following belong in the differential diagnosis? (Choose all that apply) A. Cystic brosis B. Allergic bronchopulmonary aspergillosis (ABPA) C. Chronic aspiration D. Chronic or recurrent infection E. Williams-Campbell syndrome F. Primary ciliary dyskinesia G. All of the above
Fig. 49.1B
49
Case 50 History: 29-year-old woman with an incidental nding on chest radiography. 1. Which of the following should be included in the differential diagnosis of the chest radiographic nding (Figs. 50.1 and 50.2)? (Choose all that apply.) A. Lung cancer B. Septic embolus C. Arteriovenous malformation D. Pulmonary artery pseudoaneurysm E. All of the above 2. What percent of patients with this abnormality have the inherited disorder known as hereditary hemorrhagic telangiectasia? A. Less than 1% B. 1% to 20%
C. 20% to 40% D. 40% to 60% E. 60% to 80% F. Greater than 80% 3. Which of the following is the most common presentation for persons with pulmonary arteriovenous malformation (AVMs)? A. Cerebral abscess B. Hemoptysis C. Cough D. Asymptomatic
Fig. 50.2
Fig. 50.1
50
Case 51 History: 52-year-old woman with an abnormal chest radiograph (Figs. 51.1A and 51.1B). 1. What is the most likely cause of the mediastinal mass? A. Mediastinal hematoma B. Lymphadenopathy C. Ascending aortic aneurysm D. Thymic neoplasm 2. What percentage of mediastinal masses are vascular in etiology? A. Less than 1% B. 10%
Fig. 51.1A
C. 20% D. 50% E. 75% 3. In a patient with Marfan syndrome, what is the threshold dimension of luminal dilatation that prompts surgical repair of an aortic aneurysm? A. 4.0 cm B. 4.5 cm C. 5.0 cm D. 5.5 cm E. 6.0 cm
Fig. 51.1B
51
Case 52 History: 32-year-old man with right chest pain and dyspnea. 1. Which of the following should be included in the differential diagnosis of the chest radiographic nding (Fig. 52.1)? (Choose all that apply.) A. Lung cancer B. Pneumonia C. Pulmonary infarct D. Pulmonary contusion E. All of the above 2. Which of the following indicates a radiographic manifestation of pulmonary infarction? A. Wells score B. Hampton hump
Fig. 52.1
52
C. Westermark sign D. Knuckle sign E. Fleishner sign 3. Which of the following is a poor prognostic indicator in patients with acute pulmonary emboli? A. Pulmonary infarct B. Hemoptysis C. RV:LV ratio >1.0 D. Subsegmental DECT perfusion defects
Case 53 History: 25-year-old reghter with fever, fatigue, and night sweats. 1. Which of the following should be included in the differential diagnosis of the chest radiographic nding (Fig. 53.1)? (Choose all that apply) A. Metastasis B. Lymphoma C. Germ cell neoplasm D. Thymic epithelial neoplasm E. All of the above 2. Which of the following is true regarding “bulky” disease for Hodgkin lymphoma (Figs.53.2 and 53.3)? A. Portends a favorable prognosis B. Dened by chest radiograph extent C. Dened by a single nodal mass measuring greater than 3 cm on computed tomography (CT) D. Dened by a single nodal mass occupying greater than 1/3 of thoracic diameter 3. Which of the following is considered the gold standard imaging modality for initial, interim, and end-of-treatment assessment of lymphoma? A. Chest radiograph B. Non-contrast chest CT C. Fluorodeoxyglucose -positron emission tomography (18FDG-PET)-CT D. Contrast-enhanced thoracic magnetic resonance imaging (MRI)
Fig. 53.2
Fig. 53.1
Fig. 53.3
53
Case 54 History: 35-year-old man after high-speed motorcycle accident. 1. Which of the following are the most common sites of traumatic aortic injury (TAI). (Choose all that apply.) A. Aortic root B. Aortic isthmus C. Sinotubular junction D. Branch vessel origins E. Diaphragmatic hiatus 2. Which of the following ndings on the radiograph supports the diagnosis of TAI over other mimics (Fig.54.1)? A. Atherosclerotic plaque B. Sinotubular junction dilation
Fig. 54.1
54
C. Rib notching D. Periaortic hemorrhage 3. What is the typical in-the-eld mortality associated with TAI? A. 10% to 20% B. 20% to 30% C. 40% to 50% D. 50% to 60% E. 80% to 90%
Case 55 History: 88-year-old woman referred to CT for an abnormal chest radiograph (Figs.55.1 and 55.2). 1. Which of the following should be included in the differential diagnosis for chronic airspace opacity? (Choose all that apply.) A. Lung adenocarcinoma B. Lymphoma C. Alveolar proteinosis D. Organizing pneumonia E. Lipoid pneumonia 2. Which of the following ndings supports the diagnosis of lipoid pneumonia in the setting of a chronic consolidation? A. Spiculated margin B. Long-term stability
Fig. 55.1
C. Macroscopic fat attenuation D. Central cavitation 3. Which of the following may predispose a patient to developing lipoid pneumonia? A. Young age B. Smoking C. Positive pressure ventilation D. Zenker diverticulum E. Intravenous (IV) drug abuse
Fig. 55.2
55
Case 56 History: 33-year-old man referred for an abnormal chest radiograph (Fig.56.1) and infertility. 1. What is the most likely diagnosis (Figs.56.2 and 56.3)? A. Allergic bronchopulmonary Aspergillosis B. Kartagener syndrome C. Cystic brosis D. Recurrent aspiration E. Bronchial atresia 2. Which of the following comprise the triad associated with this entity? (Choose three.) A. Bronchiectasis B. Situs solitus C. Situs inversus D. Sinusitis E. Extralobar sequestration 3. Which of the following is the typical pattern of inheritance of this disorder? A. Autosomal dominant B. Autosomal recessive C. X-linked dominant D. X-linked recessive E. Mosaicism Fig. 56.2
Fig. 56.1
56
Fig. 56.3
Case 57 History: 59-year-old woman on chronic steroids for asthma presents with hemoptysis. 1. Which of the following should be included in the differential diagnosis for the radiographic ndings (Fig.57.1)? (Choose all that apply.) A. Pulmonary infarct B. Lung cancer C. Invasive aspergillus D. Rounded atelectasis E. Organizing pneumonia 2. What is the most likely diagnosis in this case? A. Pulmonary infarct B. Lung cancer C. Invasive aspergillus D. Rounded atelectasis E. Organizing pneumonia
Fig. 57.1
3. What clinical information in this case supports the diagnosis? A. Female sex B. History of asthma C. Chronic steroid therapy D. Peripheral location 4. Which computed tomography (CT) sign is demonstrated (Fig.57.2)? A. Signet ring B. Air bronchogram C. Comet tail D. Halo E. Reverse halo
Fig. 57.2
57
Case 58 History: 75-year-old man with epistaxis. 1. What should be included in the differential diagnosis of the imaging ndings (Figs.58.1 and 58.2)? (Choose all that apply.) A. Septic emboli B. Lung abscess C. Multifocal pneumonia D. Metastatic disease E. Vasculitis 2. What is the most likely diagnosis in this case? A. Septic emboli B. Pulmonary abscesses C. Multifocal pneumonia D. Metastatic disease E. Vasculitis
Fig. 58.1
58
3. What clinical information would support the diagnosis? A. Male sex B. Intravenous (IV) drug abuse C. Epistaxis D. Immunosuppression 4. What computed tomography (CT) sign is demonstrated? A. Signet ring B. Air bronchogram C. Comet tail D. CT halo E. Reverse CT halo
Fig. 58.2
Case 59 History: 52-year-old woman with chronic dyspnea (Figs. 59.1 and 59.2). 1. Which of the following should be included in the differential diagnosis for this patient? (Choose all that apply.) A. Langerhans cell histiocytosis B. Lymphangioleiomyomatosis C. Tuberous sclerosis D. Hermansky-Pudlak syndrome 2. What is a characteristic computed tomography (CT) nding in Langerhans cell histiocytosis? A. Ground-glass opacity B. Costophrenic angle sparing C. Consolidation D. Apical sparing
Fig. 59.1
3. Which of the conditions listed is associated with chylous pleural effusions? A. Lymphangioleiomyomatosis B. Langerhans cell histiocytosis C. Amyloidosis D. Lymphocytic interstitial pneumonitis 4. Which of the following ndings are not usually present in the setting of tuberous sclerosis? A. Fatty cardiac lesions B. Multinodular multifocal pneumocyte hyperplasia C. Hepatocellular carcinoma D. Angiomyolipomas
Fig. 59.2
59
Case 60 History: Young man with history of chest pain and cough (Fig.60.1). 1. Which of the following should be included in the differential diagnosis for this patient? (Choose all that apply.) A. Lymphoma B. Tuberculosis C. Metastatic disease D. Sarcoidosis 2. In the setting of tuberculosis, which pulmonary nding is more common in children than adults? A. Fibrocavitary disease B. Ground-glass opacity C. Bronchiectasis D. Consolidation 3. Tuberculous lymphadenopathy is common in which of the following conditions? A. Acquired immune deciency syndrome B. Rheumatoid arthritis C. Common variable immune deciency D. Lymphocytic interstitial pneumonitis 4. In which demographic is tuberculous pleuritis least common? A. Infants B. Adolescents C. Young adults D. Elderly
60
Fig. 60.1
Case 61 History: 49-year-old man with human immunodeciency virus (HIV) and shortness of breath (Fig.61.1). 1. Which of the following should be included in the differential diagnosis for this patient? (Choose all that apply.) A. Pulmonary edema B. Diffuse alveolar hemorrhage C. Pneumocystis jirovecii pneumonia D. Pulmonary alveolar proteinosis 2. Which of the following is another common computed tomography (CT) nding in patients with Pneumocystis jirovecii pneumonia? A. Pleural effusions B. Cysts C. Lymphadenopathy D. Tree-in-bud nodules 3. Which of the conditions listed below is associated with chylous pleural effusions? A. Lymphangioleiomyomatosis B. Langerhans cell histiocytosis C. Amyloidosis D. Lymphocytic interstitial pneumonitis 4. What is rst-line therapy for Pneumocystis jirovecii pneumonia? A. Trimethoprim-sulfamethoxazole B. Amoxicillin/clavulanic acid C. Linezolid D. Azithromycin
Fig. 61.1
61
Case 62 History: 25-year-old man; history withheld (Fig.62.1). 1. Which of the following should be included in the differential diagnosis for the appearance of the mediastinum and hila? (Choose all that apply.) A. Silicosis B. Tuberculosis C. Histoplasmosis D. Sarcoidosis 2. Calcied lymphadenopathy following treatment is seen most frequently in which disease? A. Cat scratch disease B. Acquired immunodeciency syndrome (AIDS) C. Systemic sclerosis D. Lymphoma
3. Which of the following conditions is least likely to cause eggshell calcication in hilar and mediastinal lymphadenopathy? A. Silicosis B. Sarcoidosis C. Histoplasmosis D. Coal workers pneumoconiosis 4. Garland’s triad is highly associated with which of the following diseases? A. Sarcoidosis B. Lymphoma C. Tuberculosis D. Leukemia
Fig. 62.1
62
Case 63 History: 52-year-old man with fever, chills, and pleuritic chest pain (Fig.63.1). 1. Which of the following should be included in the differential diagnosis for this patient? (Choose all that apply.) A. Metastatic disease B. Septic infarcts C. Granulomatosis with polyangiitis D. Acute respiratory distress syndrome 2. What is the likely etiology for chronic cavitary lung nodules in a patient with joint pain and subcutaneous nodules? A. Tuberculosis B. Rheumatoid arthritis C. Granulomatosis with polyangiitis D. Invasive aspergillosis 3. Which the following is not a risk factor for development of septic infarcts? A. Central venous catheter B. Periodontal disease C. Pulmonary hypertension D. Intravenous drug use
4. What is the most common imaging manifestation of fat embolism? A. Ground-glass opacity B. Fatty attenuation pulmonary emboli C. Tree-in-bud nodules D. Consolidation
Fig. 63.1
63
Case 64 History: History of recent pneumonectomy. Two chest radiographs are shown acquired 2 days apart (Figs.64.1 and 64.2). 1. What is the diagnosis? A. Extrapleural hematoma B. Hemothorax C. Bronchopleural stula D. Expected postoperative evolution 2. What is likely underlying reason for a higher incidence of bronchopleural stula after right pneumonectomy as compared to left pneumonectomy? A. Obtuse angle of the bronchial stump B. Shorter length of bronchial stump C. Technically more difcult surgery D. Greater frequency of right pneumonectomies
Fig. 64.1
64
3. Which lobe is most prone to lobar torsion after ipsilateral lobectomy? A. Right middle lobe B. Right lower lobe C. Left upper lobe D. Left lower lobe 4. In the setting of suspected bronchopleural stula after pneumonectomy, what is the next step in management? A. Chest tube drainage B. Watchful waiting C. Surgical exploration D. Bronchoscopy
Fig. 64.2
Case 65 History: 59-year-old man with left shoulder pain and hand weakness. 1. The imaging ndings in this case are most commonly associated with which cell type of lung cancer (Figs.65.1, 65.2 and 65.3)? A. Small cell carcinoma B. Squamous cell carcinoma C. Large cell carcinoma D. Adenocarcinoma 2. If this tumor invaded the chest wall, what staging factor would make him an inoperable candidate? A. T3 disease B. N2 disease C. N3 disease D. Any nodal status
3. In the staging of lung cancer invasion of which of the following is considered T4 (non-operable)? A. Phrenic nerve B. Recurrent laryngeal nerve C. Diaphragm D. Parietal pericardium 4. What is the current standard therapy for most operable superior sulcus tumors? A. Chemotherapy B. Chemotherapy and radiotherapy C. Chemotherapy and radiotherapy with subsequent surgery D. Surgery with subsequent chemotherapy and radiotherapy
Fig. 65.3
Fig. 65.1
Fig. 65.2
65
Case 66 History: 26-year-old with fever, night sweats, and cough (Fig.66.1). 1. Which of the following should be included in the differential diagnosis for this patient? (Choose all that apply.) A. Lung cancer B. Fungal pneumonia C. TB D. Pulmonary infarct 2. Which lung cancer cell type is associated most frequently with cavitation? A. Small cell lung cancer B. Adenocarcinoma C. Large cell carcinoma D. Squamous cell carcinoma 3. In adult patients with tuberculosis, tree-in-bud opacities indicate involvement in which anatomic structures? A. Arterioles B. Lymphatics C. Bronchioles D. Alveoli 4. In the setting of acute pulmonary embolism, which comorbid condition is most frequently associated with the development of pulmonary infarction? A. Congestive heart failure B. Asthma C. Pulmonary brosis D. Tricuspid regurgitation
66
Fig. 66.1
Case 67 History: 63-year-old man with long history of smoking and chronic cough. 1. Which of the following lymph node stations demonstrate uorodeoxyglucose (FDG) avidity in this patient with non-small cell lung cancer (Figs.67.1, 67.2 and 67.3)? (Check all that apply.) A. Subaortic B. Low paratracheal C. High paratracheal D. Para-aortic E. Hilar 2. What nding makes this cancer unresectable? A. T3 disease B. N2 disease C. N3 disease D. M1 disease
3. What is the most common lobe affected by lung cancer? A. Right upper lobe B. Right middle lobe C. Right lower lobe D. Left lower lobe 4. What feature of lung cancer is least likely to be associated with a false negative result on positron emission tomography/ computed tomography (PET/CT)? A. Small size B. Carcinoid histology C. Ground-glass attenuation D. Spiculated margins
Fig. 67.1 Fig. 67.3
Fig. 67.2
67
Case 68 History: 57-year-old man with cough. 1. Which of the following should be included in the differential diagnosis for the nodule shown in (Fig. 68.1)? (Choose all that apply.) A. Pneumonia B. Aspiration C. Adenocarcinoma D. Scar or focal brosis 2. Which attenuation characteristic of a focal pulmonary nodule is most concerning for primary lung cancer? A. Ground-glass B. Part-solid C. Solid D. Solid with cavitation
Fig. 68.1
68
3. A solitary 4 to 5 mm ground-glass nodule is identied incidentally on a chest computed tomography (CT) for evaluation of chronic dyspnea. According to current guidelines, what is the next step in management? A. CT follow-up in 3 months B. CT follow-up in 6 months C. CT follow-up in 12 months D. No CT follow-up 4. Which of the following is least likely to cause diffuse centrilobular ground-glass nodules throughout the lungs? A. Hypersensitivity pneumonitis B. Metastatic disease C. Pulmonary hemorrhage D. Respiratory bronchiolitis
Case 69 History: 45-year-old woman with chronic dyspnea. 1. Which of the following should be included in the differential diagnosis for this patient based solely on the radiograph (Fig.69.1)? (Choose all that apply.) A. Lymphoma B. Azygos continuation of the inferior vena cava (IVC) C. Superior vena cava obstruction D. Partial anomalous pulmonary venous return 2. What is the most likely diagnosis based on all imaging available (Figs.69.1, 69.2 and 69.3)? A. Lymphoma B. Azygos continuation of the inferior vena cava C. Superior vena cava obstruction D. Partial anomalous pulmonary venous return
3. In the setting of chronic superior vena cava obstruction, which collateral veins allow drainage of abdominal wall veins into the anterior aspect of the liver, which may produce enhancing pseudo-hepatic lesions? A. Veins of Sappey B. Veins of Marshall C. Veins of Galen D. Veins of Mayo 4. When compared to asplenia heterotaxy syndrome, polysplenia is more often associated with which of the following? A. Higher prevalence of cyanotic congenital heart disease B. Higher prevalence of midgut malrotation C. Lower prevalence of azygos continuation of IVC D. Lower prevalence of bilateral hyparterial bronchi
Fig. 69.1
69
70
SECTION II
Fair Game
Fig. 69.2
Fig. 69.3
Case 70 History: 75-year-old man with cough and dyspnea (Figs. 70.1 and 70.2). 1. Which of the following should be included in the differential diagnosis for this patient based solely on the radiograph? (Choose all that apply.) A. Pneumonia B. Aspiration C. Lung cancer D. Lymphoma 2. What imaging sign is demonstrated in this case? A. Flat-waist sign B. Hilar-overlay sign C. Ivory-heart sign D. S sign of Golden
Fig. 70.1
3. What tumor is most likely in a young adult with lobar collapse associated with a calcied endobronchial lesion? A. Hamartoma B. Carcinoid tumor C. Adenoid cystic carcinoma D. Mucoepidermoid carcinoma 4. Based solely on the presence of postobstructive atelectasis present, what is the correct T (tumor) stage if the underlying lesion were a non–small cell lung cancer (NSCLC)? A. T1 B. T2 C. T3 D. T4
Fig. 70.2
71
Case 71 History: 45-year-old man with history of falling off a ladder one-year ago. 1. Which of the following should be included in the differential diagnosis for this patient? (Choose all that apply.) A. Large pleural effusion B. Diaphragmatic rupture C. Diaphragmatic paralysis D. No abnormality 2. What imaging sign is present on the axial computed tomography (CT) images (Figs.71.1 and 71.2)? A. Thoracoabdominal sign B. Cobweb sign C. Fleischner sign D. Dependent viscera sign 3. In diaphragmatic rupture from blunt trauma, what location of injury is most common? A. Anterolateral right hemidiaphragm B. Anterolateral left hemidiaphragm C. Posterolateral right hemidiaphragm D. Posterolateral left hemidiaphragm 4. A patient presents with marked elevation of the right hemidiaphragm on radiography after thoracic surgery. What is the most appropriate next imaging modality? A. Magnetic resonance imaging (MRI) B. CT C. Fluoroscopy D. Ultrasound
Fig. 71.2 Fig. 71.1
72
Case 72 History: 38-year-old man with shortness of breath (Figs.72.1 and 72.2). 1. Which of the following should be included in the differential diagnosis for this patient? (Choose all that apply.) A. Left superior vena cava (SVC) B. Azygos continuation of the inferior vena cava (IVC) C. Partial anomalous pulmonary venous return (PAPVR) D. Abnormal left superior intercostal vein 2. Into which vessel do most left-sided SVCs drain? A. Right SVC B. Hemiazygos vein C. Coronary sinus D. Right atrium
Fig. 72.1
3. In the setting of right superior PAPVR, what is the most common associated anomaly? A. Left SVC B. Azygos continuation of the IVC C. Atrial septal defect (ASD) D. Patent ductus arteriosus (PDA) 4. An unroofed coronary sinus is a type of: A. Ventricular septal defect B. PAPVR C. PDA D. ASD
Fig. 72.2
73
Case 73 History: 56-year-old man with chronic cough (Fig.73.1). 1. Which of the following should be included in the differential diagnosis for this patient? (Choose all that apply.) A. Aspiration B. Non-tuberculous mycobacterial infection C. Primary ciliary dyskinesia D. Diffuse panbronchiolitis 2. Which diffuse lung disease is least likely to be associated with aspiration? A. Usual interstitial pneumonitis B. Diffuse alveolar damage C. Lymphocytic interstitial pneumonitis D. Organizing pneumonia 3. Which of the following pulmonary segments is least likely to be affected by aspiration? A. Right upper lobe anterior B. Right upper lobe posterior C. Right lower lobe superior D. Right lower lobe basal posterior 4. Of the listed choices, which is the least common sequelae of foreign body aspiration? A. Pneumothorax B. Pneumonia C. Atelectasis D. Bronchiectasis
74
Fig. 73.1
Case 74 History: 48-year-old man with shortness of breath and crackles on physical examination (Figs.74.1 and 74.2). 1. What is the most likely diagnosis? A. Usual interstitial pneumonitis (UIP) B. Nonspecic interstitial pneumonitis (NSIP) C. Acute interstitial pneumonitis (AIP) D. Desquamative interstitial pneumonitis (DIP) 2. Which nding is inconsistent with UIP? A. Lower lung zone preponderance B. Peripheral lung preponderance C. Honeycombing D. Predominant ground-glass opacity
Fig. 74.1
3. What is the most specic differentiator of NSIP from UIP? A. Upper lung zone distribution B. Mid lung zone distribution C. Subpleural sparing D. Reticulation 4. What is the least common cause of NSIP pattern of lung disease? A. Collagen vascular disease B. Drugs C. Hypersensitivity pneumonitis D. Idiopathic
Fig. 74.2
75
Case 75 History: 42-year-old man with pleuritic chest pain (Fig.75.1). 1. Which of the following are likely causes of the abnormal aorta? (Choose all that apply). A. Atherosclerosis B. Hypertension C. Marfan syndrome D. Ehlers Danlos syndrome 2. What is the size criteria for operative repair in ascending aortic aneurysms? A. >2.5 cm B. >3.5 cm C. >4.5 cm D. >5.5 cm 3. Which of the following is least often associated with bicuspid aortic valves? A. Aortic aneurysm B. Intracranial arterial aneurysm C. Turner syndrome D. Partial anomalous pulmonary venous return 4. There is an enlarged ascending aorta with loss of the normal concave junction between the sinus of Valsalva and tubular ascending aorta. This imaging nding is most specic for which condition? A. Syphilis B. Marfan syndrome C. Bicuspid aortic valve D. Atherosclerosis
76
Fig. 75.1
Case 76 History: 30-year-old woman with chronic cough (Figs. 76.1, 76.2 and 76.3). 1. What is the most likely diagnosis? A. Small cell carcinoma B. Lymphoma C. Squamous cell carcinoma D. Carcinoid tumor 2. What is the most common benign pulmonary tumor? A. Lipoma B. Hamartoma C. Chondroma D. Papilloma 3. What percentage of carcinoid tumors are calcied? A. 10% B. 20% C. 30% D. 40% 4. What is a major nding in diffuse idiopathic pulmonary neuroendocrine cell hyperplasia (DIPNECH)? A. Consolidation B. Fibrosis C. Pneumothorax D. Air-trapping
Fig. 76.2
Fig. 76.1
Fig. 76.3
77
Case 77 History: 58-year-old woman with acute chest pain (Figs. 77.1 and 77.2). 1. Which of the following etiologies should be considered? (Choose all that apply.) A. Childbirth B. Asthma C. Esophageal rupture D. Extension of pneumothorax 2. What condition is thought to be related to the development of asthma-related pneumomediastinum? A. Pneumothorax B. Pulmonary interstitial emphysema C. Honeycombing D. Centrilobular emphysema 3. In which anatomic area of the esophagus does rupture from vomiting usually occur? A. Left mid B. Right mid C. Right lower D. Left lower 4. What is the most serious complication of esophageal rupture? A. Pneumonia B. Pneumothorax C. Mediastinitis D. Empyema
Fig. 77.1
78
Fig. 77.2
Case 78 History: 67-year-old woman with chronic cough and malaise (Fig.78.1). 1. Which of the following should be included in the differential diagnosis for this patient? (Choose all that apply.) A. Nontuberculous mycobacterial infection B. Allergic bronchopulmonary aspergillosis C. Cystic brosis D. Usual interstitial pneumonitis 2. Which computed tomography (CT) postprocessing tool is most helpful in differentiating bronchiectasis from cystic lung disease? A. Maximum intensity projection B. 3D volume rendering C. Surface rendering D. Minimum intensity projection 3. Which of the following is not an imaging subtype of bronchiectasis? A. Spherical B. Varicoid C. Cystic D. Cylindrical 4. Excluding the pancreas, which is the most frequently involved abdominal organ in cystic brosis? A. Kidneys B. Liver C. Spleen D. Stomach
Fig. 78.1
79
Case 79 History: 43-year-old man with dyspnea. 1. Which of the following should be included in the differential diagnosis for this patient based on the chest radiograph (Fig.79.1)? (Choose all that apply.) A. Pneumothorax B. Giant bulla C. Poland syndrome D. Langerhans cell histocytosis 2. What is the size criterion that distinguishes a bulla from a bleb? A. ≥0.5 cm B. ≥1 cm C. ≥1.5 cm D. ≥2 cm 3. Which of the following tests is most sensitive for the detection of early emphysema? A. Tomosynthesis B. Pulmonary function tests C. Hyperpolarized He magnetic resonance imaging (MRI) D. Chest computed tomography (CT) 4. Which illicit drug has been associated with basilar predominant panlobular emphysema? A. Smoked crack cocaine B. Smoked marijuana C. Intravenous (IV) heroin D. IV Ritalin
Fig. 79.1
80
Case 80 History: 61-year-old woman with history of breast cancer (Figs.80.1 and 80.2). 1. Which is collapsed? A. Right upper lobe B. Right middle lobe C. Right lower lobe D. Combined right middle and lower lobes 2. Which bronchus is obstructed? A. Right upper lobe bronchus B. Right middle lobe bronchus C. Bronchus intermedius D. Right lower lobe bronchus
Fig. 80.1
3. In a recently intubated patient, what is the most likely cause of complete right lung atelectasis? A. Foreign body aspiration B. Mucous plugging C. Endobronchial tumor D. Endotracheal tube misplacement 4. Which of the listed primary tumors are least likely to cause central airway metastases? A. Melanoma B. Renal cell carcinoma C. Breast cancer D. Hepatocellular carcinoma
Fig. 80.2
81
Case 81 History: 41-year-old woman with chest asymmetry on examination (Figs.81.1, 81.2 and 81.3). 1. Which of the following should be included in the differential diagnosis for this patient? (Choose all that apply.) A. Lipoma B. Hamartoma C. Liposarcoma D. Fat necrosis 2. Which of the following material is not inherently T1 hyperintense on magnetic resonance imaging (MRI)? A. Fat B. Melanin C. Methemoglobin D. Cerebral spinal uid
Fig. 81.1
82
Fig. 81.2
3. Which of the follow-up ndings is more suggestive of liposarcoma rather than lipoma? A. Larger size B. Amorphous shape C. Greater amount of soft tissue D. Greater T2 signal 4. A bilobed fat attenuation mass is noted in the interatrial septum with high uorodeoxyglucose (FDG) uptake on positron-emission tomography–computed tomography (PET-CT). What is most likely diagnosis? A. Teratoma B. Liposarcoma C. Lipomatous hypertrophy D. Thymolipoma
Fig. 81.3
Case 82 History: Found down and unresponsive (Figs.82.1 and 82.2). 1. Which of the following should be included in the differential diagnosis for this patient? (Choose all that apply.) A. Extrapleural hematoma B. Pulmonary contusion C. Asbestos pleural thickening D. Hemothorax 2. Which of the following ndings suggest primary lung cancer as the most likely cause of a unilateral apical cap? A. Hilar lymphadenopathy B. Pleural effusion C. Bone destruction D. Mediastinal widening
3. Which one of the following is the most likely to cause bilateral apical caps? A. Trauma associated with mediastinal hematoma B. Radiation for breast carcinoma C. Advancing age D. Head and neck carcinoma 4. Which one of the following is the best next step in the management of this patient? A. Computed tomography (CT) B. Magnetic resonance imaging (MRI) C. Ultrasound D. Fluorodeoxyglucose–positron emission tomography (FDGPET) (FDG-PET)
Fig. 82.2
Fig. 82.1
83
Case 83 History: 35-year-old man with testicular cancer treated with chemotherapy. 1. Which of the following should be included in the differential diagnosis for this patient (Figs. 83.1 and 83.2)? (Choose all that apply.) A. Bleomycin lung toxicity B. Congestive heart failure C. Collagen vascular disease D. Fungal pneumonia 2. Approximately what percent of patients receiving bleomycin develop lung toxicity? A. < 5% B. 15% C. 25% D. 35%
Fig. 83.1
84
3. Which one of the following does not increase the risk of developing bleomycin-related lung toxicity? A. Lymphoma B. Concurrent radiation therapy C. Renal insufciency D. Advanced age 4. Which one of the following regarding treatment of bleomycinrelated lung toxicity is not true? A. The drug must be discontinued. B. Corticosteroids are usually administered. C. Most patients improve within a few days. D. Some patients take up to 2 years to recover.
Fig. 83.2
Case 84 History: Hemoptysis. 1. Which of the following should be included in the differential diagnosis for this patient (Figs.84.1, 84.2 and 84.3)? (Choose all that apply.) A. Lung Cancer B. Metastatic disease C. Silicosis D. Tuberculosis 2. Biopsy showed the left upper lobe mass to be primary lung adenocarcinoma. Which one of the following is the likely N staging of this patient based on the most recent tumor, node metastasis (TNM) staging system? A. N0 B. N1 C. N2 D. N3
3. Which one of the following regarding lymph node metastases is true? A. Ipsilateral axillary lymph node metastases are staged as N3. B. N2 disease precludes surgical resection. C. Fluorodeoxyglucose–positron emission tomography (FDG-PET) is the standard for lymph node staging. D. N2 disease results in a minimal stage of IIIB. 4. Which one of the following ndings is a contraindication for resection of non–small cell lung carcinoma? A. Chest wall invasion B. Tumor metastasis to the same lobe C. Ipsilateral pleural metastasis D. Post obstructive pneumonia involving the entire lung
Fig. 84.2
Fig. 84.1
Fig. 84.3
85
Case 85 History: Swelling of the head and neck. 1. What is the most likely cause of the ndings on the computed tomography (CT) scan (Fig.85.1)? A. Superior vena cava syndrome B. Arteriovenous malformation C. Contrast extravasation D. None of the above 2. What is the most common cause of superior vena cava (SVC) syndrome? A. Long-term intravenous device B. Fibrosing mediastinitis C. Lung cancer D. Radiation therapy
Fig. 85.1
86
3. Which one of the following is not a common clinical manifestation of SVC syndrome? A. Head and face edema B. Extremity edema C. Visual disturbances D. Dural venous sinus thrombosis 4. What is the best initial treatment for patients with non–small cell lung carcinoma who present with acute SVC syndrome? A. Chemotherapy B. Surgical resection C. SVC stenting D. Radiation therapy
Case 86 History: Acute onset dyspnea and hypoxia. 1. Which of the following should be included in the differential diagnosis for this patient (Figs. 86.1 and 86.2)? (Choose all that apply.) A. Pulmonary hemorrhage B. Langerhans cell histiocytosis C. Diffuse pneumonia D. Silicosis 2. What is the most common cause of diffuse alveolar hemorrhage? A. Hemophilia B. Drug toxicity C. Vasculitis D. Infection
3. Which of the following has the greatest association with diffuse alveolar hemorrhage? A. Granulomatosis with polyangiitis B. Microscopic polyangiitis C. Eosinophilic granulomatosis with polyangiitis D. Takayasu arteritis 4. What procedure best conrms the diagnosis of diffuse alveolar hemorrhage? A. Bronchoalveolar lavage B. Transbronchial biopsy C. Surgical biopsy D. Sputum analysis
Fig. 86.2
Fig. 86.1
87
Case 87 History: 72-year-old woman with progressive dyspnea on exertion. 1. Which of the following should be included in the differential diagnosis for this patient (Figs.87.1, 87.2 and 87.3)? (Choose all that apply.) A. Desquamative interstitial pneumonia (DIP) B. Usual interstitial pneumonia (UIP) C. Nonspecic interstitial pneumonia (NSIP) D. Lymphoid interstitial pneumonia (LIP) 2. What is the next best approach to tissue diagnosis? A. Surgical lung biopsy B. Transbronchial biopsy C. Computed tomography (CT) guided needle biopsy D. No tissue sampling needed
3. Which of the following collagen vascular diseases has the highest association with usual interstitial pneumonia? A. Rheumatoid arthritis B. Progressive systemic sclerosis C. Polymyositis D. Sjögren syndrome 4. Which of the following occupational exposures has a strong association with usual interstitial pneumonia? A. Coal B. Silica C. Beryllium D. Asbestos
Fig. 87.1 Fig. 87.3
Fig. 87.2
88
Case 88 History: Chest pain. History of stroke. 1. Which of the following should be included in the differential diagnosis for this patient (Figs. 88.1 and 88.2)? (Choose all that apply.) A. Chest wall metastasis B. Abscess C. Hematoma D. Elastobroma 2. The high-attenuation component of the mass on this computed tomography (CT) scan of a patient on warfarin most likely represents which one of the following? A. Tumor B. Blood C. Calcium D. Fat
Fig. 88.1
3. Which one of the following is not a potential complication of intramuscular hemorrhage? A. Malignant degeneration B. Ischemic myopathy C. Neuropathy D. Pressure necrosis of adjacent bone 4. Which of the following is a common cause of spontaneous intramuscular hematoma? A. Aspirin therapy B. Intravenous (IV) contrast administration C. Vitamin K supplements D. Hemophilia
Fig. 88.2
89
Case 89 History: Occupational screening. Patient is asymptomatic. 1. Which of the following should be included in the differential diagnosis for this patient (Figs. 89.1 and 89.2)? (Choose all that apply.) A. Scimitar syndrome B. Congenital pulmonary airway malformation C. Pulmonary arteriovenous malformation D. Bronchial atresia 2. What type of shunt is most likely present? A. Left-to-left B. Left-to-right C. Right-to-right D. Right-to-left
3. What other ndings are associated with this diagnosis? A. Hypoplastic lung with abnormal airway branching B. Lung cancer C. Kidney agenesis D. Muscular dystrophy 4. Which of the following is the best management for this patient? A. Right lower lobectomy B. Endovascular coiling C. Bronchoscopy D. Observation
Fig. 89.2 Fig. 89.1
90
Case 90 History: Chest pain and pericardial rub on auscultation. 1. Which of the following should be included in the differential diagnosis of pericardial effusion (Figs.90.1, 90.2 and 90.3)? (Choose all that apply.) A. Myocardial infarction B. Systemic lupus erythematosus C. Relapsing polychondritis D. Acute pulmonary thromboembolism 2. The lateral view shows (Figs.90.2) which of the following? A. “Oreo cookie sign” B. Myocardial fat stripe C. Epicardial fat stripe D. A and C
Fig. 90.1
3. Which of the following is most sensitive for detecting a pericardial effusion? A. Chest radiograph B. Transthoracic echocardiogram C. Cardiac magnetic resonance imaging (MRI) D. Transesophageal echocardiogram 4. Which of the following denes Dressler syndrome? A. Rheumatoid arthritis-associated pleural and pericardial effusions B. Viral myocarditis-related pleural and pericardial effusions C. Radiation-induced pleural and pericardial effusions D. Myocardial infarction-related pleural and pericardial effusions
Fig. 90.3
Fig. 90.2
91
Case 91 History: Chronic cough and recurrent pneumonia. 1. Which of the following should be included in the differential diagnosis for this patient (Figs.91.1 and 91.2)? (Choose all that apply.) A. Ciliary dyskinesia B. Allergic bronchopulmonary aspergillosis C. Idiopathic pulmonary brosis D. Lymphangioleiomyomatosis 2. What is the most common cause of this abnormality? A. Congenital condition B. Interstitial pneumonia C. Cigarette smoking D. Infection
3. Which of the following is not a specic sign of bronchiectasis on high-resolution computed tomography (HRCT)? A. Bronchial diameter greater than that of the adjacent artery B. Lack of normal bronchial tapering C. Bronchial wall thickening D. Bronchus visible in peripheral 1 cm of lung 4. Which of the following is not a complication of bronchiectasis? A. Hemoptysis B. Atelectasis C. Pneumonia D. Pleural effusion
Fig. 91.1
Fig. 91.2
92
Case 92 History: Dysphagia. 1. Which of the following should be included in the differential diagnosis for this patient (Fig.92.1)? (Choose all that apply.) A. Esophageal duplication cyst B. Achalasia C. Esophageal diverticulum D. Esophageal carcinoma 2. Which one of the following structures is most likely responsible for displacement of the azygoesophageal contour in this patient? A. Esophagus B. Azygos vein C. Right atrium D. Left atrium
3. Which one of the following imaging studies would be most helpful for further evaluation of this nding? A. Computed tomography (CT) scan B. Magnetic resonance imaging (MRI) C. Esophagram D. Lateral radiograph 4. Which one of the following does not result in a chronically constricted lower esophageal sphincter? A. Progressive systemic sclerosis (scleroderma) B. Achalasia C. Chagas disease D. Pseudoachalasia
Fig. 92.1
93
Case 93 History: Cough. 1. Which of the following should be included in the differential diagnosis for this patient (Figs. 93.1 and 93.2)? (Choose all that apply.) A. Granulomatosis with polyangiitis B. Sarcoidosis C. Coccidioidomycosis D. Septic infarct 2. In which geographic location is Coccidioides endemic? A. New England B. Mississippi River valley C. Pacic Northwest D. American Southwest
3. Which one of the following regarding coccidioidomycosis is true? A. Most affected patients are symptomatic. B. Cavities are typical of chronic infection. C. Disseminated disease is common. D. The pneumonic form requires antifungal therapy. 4. What pattern is most associated with disseminated coccidioidomycosis? A. Multifocal lung consolidation B. Numerous small nodules C. Diffuse ground-glass opacity D. Septic pulmonary infarcts
Fig. 93.2
Fig. 93.1
94
Case 94 History: Dyspnea. 1. Which of the following should be included in the differential diagnosis for this patient (Fig.94.1)? (Choose all that apply.) A. Hydrostatic pulmonary edema B. Usual interstitial pneumonia C. Lymphangitic carcinomatosis D. Sarcoidosis 2. Which one of the following chest radiographic nding correlates with the computed tomography (CT) nding of thickened interlobular septa? A. Kerley B lines B. Thickened ssures C. Bronchovascular bundle thickening D. Lung consolidation
3. Which one of the following are required components of high-resolution computed tomography (HRCT) imaging? A. Thin collimation (≤1.5 mm) B. Prone imaging C. Multiplanar reformations D. Smoothing reconstruction kernel 4. Which of the following is the best next step in management of this patient in whom congestive heart failure is not suspected? A. Fluorodeoxyglucose–positron emission tomography (FDG-PET) B. Repeat chest imaging after diuresis C. Transbronchial biopsy D. Surgical lung biopsy
Fig. 94.1
95
Case 95 History: Chest pain (Figs.95.1 and 95.2). 1. Based on the lateral chest radiograph (Fig. 95.1), which of the following should be included in the differential diagnosis for this patient? (Choose all that apply.) A. Aortic coarctation B. Lymphoma C. Breast carcinoma D. Tuberculosis 2. Which one of the following is the most common site of lymph node metastases in patients with breast carcinoma? A. Internal mammary B. Axillary C. Hilar D. Subcarinal
3. True or false? A normal chest radiograph excludes internal mammary lymphadenopathy. A. True B. False 4. Which type of lymphoma is most commonly associated with neoplastic tissue conned to the anterior mediastinum? A. Classical Hodgkin lymphoma B. Follicular lymphoma C. Diffuse large B cell lymphoma D. Small lymphocytic lymphoma
Fig. 95.2
Fig. 95.1
96
Case 96 History: Lung nodule (Fig.96.1). 1. Which of the following is the greatest risk factor for developing pneumothorax as a result of needle biopsy? A. Older age B. Emphysema C. Lower lobe location D. Subpleural location 2. What is the sensitivity of transthoracic needle biopsy (TTNB) for malignant nodules? A. 50% B. 70% C. 80% D. >90%
3. Which one of the following is the most common complication of TTNB? A. Hemoptysis B. Pneumothorax C. Air embolism D. Seeding of biopsy track 4. Which one of the following improves the diagnostic yield of TTNB for benign lesions? A. Use two needles to increase volume of aspirate. B. Have a cytopathologist on-site. C. Use a cutting needle to obtain core needle biopsy specimens. D. Use CT uoroscopy.
Fig. 96.1
97
Case 97 History: Abnormal chest radiograph. 1. Which of the following should be included in the differential diagnosis for this patient (Figs.97.1 and 97.2)? (Choose all that apply.) A. Carcinoid tumor B. Constrictive bronchiolitis C. Bronchial atresia D. Lung abscess 2. What allows aeration of lung distal to a bronchocele? A. Bronchopleural stula B. Incomplete obstruction C. Anomalous bronchus D. Collateral air ow through adjacent lung
3. What is the most common site of bronchial atresia? A. Left upper lobe B. Left lower lobe C. Right upper lobe D. Right lower lobe 4. Which of the following is the best treatment for bronchial atresia? A. Surgical resection B. Endobronchial stenting C. Observation D. Endobronchial laser ablation
Fig. 97.1
Fig. 97.2
98
Case 98 History: Dyspnea on exertion and intermittent fever. 1. Which of the following should be included in the differential diagnosis for this patient (Figs. 98.1 and 98.2)? (Choose all that apply.) A. Sarcoidosis B. Silicosis C. Hypersensitivity pneumonitis D. Respiratory bronchiolitis 2. What anatomic structures are located in the central core of the pulmonary lobule? A. Pulmonary artery and vein B. Pulmonary artery and bronchiole C. Pulmonary vein and bronchiole D. Pulmonary vein and lymphatics
3. Which of the following is not associated with cigarette smoking? A. Respiratory bronchiolitis B. Pulmonary Langerhans cell histiocytosis C. Desquamative interstitial pneumonia D. Chronic eosinophilic pneumonia 4. Which type of immunologic reaction is most strongly associated with hypersensitivity pneumonitis? A. Type 1 (IgE mediated) B. Type 2 (cytotoxic: IgG or IgM mediated) C. Type 3 (immune complex) D. Type 4 (delayed-type hypersensitivity)
Fig. 98.1 Fig. 98.2
99
Case 99 History: Baseline lung cancer screening. 1. Which of the following should be included in the differential diagnosis for this patient (Fig.99.1)? (Choose all that apply.) A. Lung carcinoma B. Metastasis C. Granuloma D. Streptococcal pneumonia 2. What is the appropriate Lung-RADS category for this patient? A. Lung-RADS 1 B. Lung-RADS 2 C. Lung-RADS 3 D. Lung-RADS 4a
Fig. 99.1
100
3. What is the next best management? A. Computed tomography (CT) follow-up in 6 months B. Fluorodeoxyglucose–positron emission tomography (FDG-PET)/CT C. Transthoracic needle biopsy D. Resection 4. What is the probability that this nodule is malignant? A. 20%
Case 100 History: Stridor and cough. 1. Which of the following should be included in the differential diagnosis for this patient (Figs.100.1 and 100.2)? (Choose all that apply.) A. Double aortic arch B. Previous intubation C. Granulomatosis with polyangiitis D. Mounier-Kuhn syndrome 2. Which of the following is not a risk factor for tracheal stenosis? A. Prolonged intubation B. Tracheostomy C. Balloon cuff overdistention D. Tracheal bronchus
3. Which of the following causes of tracheal stenosis characteristically spares the posterior tracheal wall? A. Relapsing polychondritis B. Inammatory bowel disease C. Granulomatosis with polyangiitis D. Amyloidosis 4. Which of the following is the least common sign or symptom of central airway stenosis? A. Cough B. Stridor C. Wheeze D. Hemoptysis
Fig. 100.1
Fig. 100.2
101
Case 101 History: Fatigue. 1. Based on the computed tomography (CT) image (Fig. 101.1), which of the following should be included in the differential diagnosis for this patient? (Choose all that apply.) A. Thymoma B. Thymolipoma C. Thymic hyperplasia D. Thymic cyst 2. Which of the following CT attenuation values is not typical of normal thymus? A. Soft tissue B. Fat and soft tissue C. Fatty replacement D. Calcication
Fig. 101.1
102
3. Pure red cell aplasia is most commonly associated with which of the following thymic lesions? A. Thymic carcinoma B. Thymoma C. Thymic carcinoid D. Thymolipoma 4. Which imaging examination is most useful for distinguishing thymic hyperplasia from thymic neoplasia? A. Fluorodeoxyglucose–positron emission tomography (FDG-PET)/CT B. Magnetic resonance imaging C. Contrast-enhanced CT D. Gallium scan
Case 102 History: Cough. 1. Which of the following should be included in the differential diagnosis for this patient (Fig.102.1)? (Choose all that apply.) A. Respiratory bronchiolitis B. Focal brosis C. Lung carcinoma D. Histoplasmoma 2. For nodules smaller than 10 mm, which of the following is most likely to be malignant? A. Solid nodule B. Ground-glass nodule C. Solid and ground-glass nodule D. Calcied nodule
3. What type of lung cancer most commonly presents as a ground-glass attenuation or part solid nodule? A. Adenocarcinoma B. Squamous cell carcinoma C. Small cell carcinoma D. Large cell carcinoma 4. Which of the following is the next best step in the follow-up of this patient? A. Fluorodeoxyglucose–positron emission tomography (FDG-PET)/CT B. Follow-up computed tomography (CT) C. Left upper lobectomy D. Bronchoscopy
Fig. 102.1
103
SECTION III
Challenge
Case 103 History: Patient with chronic cough and an abnormality on chest radiography. 1. What should be included in the differential diagnosis of the imaging abnormalities (Figs. 103.1 and 103.2)? (Choose all that apply.) A. Pulmonary edema B. Pneumocystic jirovecii pneumonia C. Diffuse alveolar hemorrhage D. Diffuse alveolar damage E. Pulmonary alveolar proteinosis 2. What is the most likely diagnosis in this case? A. Pulmonary edema B. Pneumocystic jirovecii pneumonia C. Diffuse alveolar hemorrhage
Fig. 103.1
104
D. Diffuse alveolar damage E. Pulmonary alveolar proteinosis 3. What computed tomography (CT) sign is demonstrated? A. Signet ring B. Crazy-paving C. Comet tail D. CT halo E. Reverse CT halo 4. What clinical information would support the diagnosis? A. Female sex B. Intravenous (IV) drug abuse C. Chronicity D. Immunosuppression
Fig. 103.2
Case 104 History: Patient with asthma and an abnormality on chest radiography (Fig.104.1). 1. What imaging sign is demonstrated (Figs.104.1 and 104.2)? A. Signet ring B. Crazy-paving C. Finger-in-glove D. Computed tomography (CT) halo E. Reverse CT halo 2. Which of the following organisms is often implicated in this condition? A. Human papillomavirus type 6 B. Mycobacterium tuberculosis
Fig. 104.1
C. Staphylococcus aureus D. Aspergillus fumigatus 3. Patients with which of the following diseases are predisposed to develop this condition? A. Tracheobronchopathia osteochondroplastica B. Primary lung cancer C. Cystic brosis D. Carcinoid tumor 4. What clinical information would support the diagnosis? A. Recurrent aspiration B. Asthma C. Smoking D. Immunosuppression
Fig. 104.2
105
Case 105 History: 60-year-old woman with cough and dyspnea. 1. What is the salient radiographic abnormality (Fig.105.1)? A. Right-sided pneumothorax B. Left lower lobe pneumonia C. Left upper lobe atelectasis D. Left pneumonectomy 2. What is the most likely cause of the branching low-attenuation opacities within the collapsed left upper lobe (Fig.105.2)? A. Obstruction by tumor B. Bronchial atresia C. Allergic bronchopulmonary aspergillosis D. Cystic brosis
Fig. 105.1
106
3. Which of the following would favor a benign cause for the computed tomography (CT) ndings? A. Smoking history B. Hemoptysis C. Enhancing mass D. Mediastinal inltration E. Macroscopic fat and calcium
Fig. 105.2
Case 106 History: 22-year-old female with abnormality on chest radiograph. 1. In which anatomic compartment is the lesion arising (Figs.106.1 and 106.2)? A. Parenchyma B. Pleura C. Chest wall D. Lymphatics 2. Which of the following imaging features may help determine lesion location? A. Ipsilateral pleural effusion B. Obtuse angles with parenchyma C. Chest wall invasion D. Rib erosion 3. Which of the following extrathoracic manifestations might occur in conjunction with this lesion? A. Digital clubbing B. Hypertrophic osteoarthropathy
Fig. 106.1
C. Episodic hypoglycemia D. Galactorrhea E. All of the above 4. Which of the following reects typical magnetic resonance imaging (MRI) features of small, uncomplicated lesions with this histology? A. Hyperintense signal on T1- and T2-weighted images, no enhancement B. Heterogeneous signal on T1- and T2-weighted images, no enhancement C. Hypointense signal on T1- and T2-weighted images, no enhancement D. Hypointense signal on T1- and T2-weighted images, progressive enhancement
Fig. 106.2
107
Case 107 History: 85-year-old man with weight loss undergoes outpatient evaluation. 1. What is the most likely cause of this radiographic abnormality (Fig.107.1)? A. Right-sided pneumothorax B. Left pleural effusion C. Left lung collapse D. Left pneumonectomy 2. Which of the following would be the most likely cause for this pattern in an intubated patient in the intensive care unit? A. Left-sided infusothorax B. Left lung re-expansion edema C. Left lung hemorrhage D. Left main bronchus mucus plug 3. What term is applied to the preservation of enhancing vessels within the collapsed lung (Figs.107.2 and 107.3)? A. Computed tomography (CT) halo sign B. Air bronchogram sign C. CT angiogram sign D. Juxtaphrenic peak sign 4. Which of the following would raise suspicion for a malignant etiology of the abnormality? (Choose all that apply.) A. Smoking history B. Weight loss C. Age of 85 D. Outpatient setting E. Lymphadenopathy
Fig. 107.1
108
Fig. 107.2
Fig. 107.3
Case 108 History: 79-year-old female never smoker with chronic cough. 1. In which anatomic compartment is the computed tomography (CT) abnormality predominantly located (Fig.108.1)? A. Trachea B. Bronchi C. Parenchyma D. Lymphatics 2. Which organism is most likely responsible for the clinical and imaging abnormalities? A. Inuenza-A B. Mucormycosis
C. Aspergillus fumigatus D. Mycobacterium avium complex (MAC) 3. Which form of this disease is most likely in this case? A. Cavitary (classic) B. Bronchiectatic (non-classic) C. Immunocompromised host D. Nodular E. Hypersensitivity
Fig. 108.1
109
Case 109 History: 47-year-old asymptomatic man. 1. Which of the following best describes the pattern of nodules shown (Figs.109.1 and 109.2)? A. Random, diffuse B. Centrilobular, branching C. Centrilobular, non-branching D. Perilymphatic 2. Which of the following should be included in the differential diagnosis for this pattern? A. Sarcoidosis B. Lymphangitic carcinomatosis C. Silicosis D. Lymphoma E. Miliary tuberculosis
3. Which term is used to describe computed tomography (CT) nding of conuent perilymphatic sarcoid granulomas organized into a focal nodule with surrounding smaller nodules and reticulation? A. Comet tail sign B. Mosaic attenuation C. Reverse CT halo sign D. Galaxy sign 4. Which is the most common clinical presentation of patients with this disease? A. Asymptomatic B. Bird fancier with chronic cough C. Profound weight loss D. History of malignancy
Fig. 109.2
Fig. 109.1
110
Case 110 History: 22-year-old male presents with cough and fever. 1. Which computed tomography (CT) pattern is shown (Fig.110.1)? A. Interstitial B. Centrilobular, nonbranching C. Centrilobular, branching D. Perilymphatic 2. Which anatomic structure(s) may be abnormal? A. Alveolar septae B. Interlobular septae C. Centrilobular arteriole D. Centrilobular bronchiole
3. What is the most common cause of this imaging pattern? A. Infectious bronchiolitis B. Aspiration bronchiolitis C. Diffuse panbronchiolitis D. Excipient lung disease E. Cellulose granulomatosis
Fig. 110.1
111
Case 111 History: 42-year-old male with chronic cough. 1. Which computed tomography (CT) pattern is shown (Fig. 111.1)? (Choose all that apply.) A. Crazy paving B. Mosaic attenuation C. Air trapping D. Tree-in-bud 2. Which anatomic structure may be abnormal? (Choose all that apply.) A. Alveolar septae B. Interlobular septae C. Centrilobular arteriole D. Centrilobular bronchiole 3. What is the most common cause of this imaging pattern? A. Infectious bronchiolitis B. Aspiration bronchiolitis C. Diffuse panbronchiolitis D. Excipient lung disease E. Chronic thromboembolism
Fig. 111.1
112
Case 112 History: 90-year-old male former lightbulb assembly worker with dyspnea. 1. Which indirect sign(s) of volume loss are demonstrated on the radiographs (Figs. 112.1 and 112.2)? (Choose all that apply.) A. Superior hilar displacement B. Increased retrosternal clear space C. Juxtaphrenic peak D. Reticulonodular opacities 2. What is the primary distribution of abnormality? A. Upper lung zone B. Lower lung zone C. Diffuse D. Subpleural 3. Which of the following should be included in the differential diagnosis? (Choose all that apply.) A. Tuberculosis B. Chronic (cluster 2) hypersensitivity pneumonitis C. Sarcoidosis (stage IV) D. Coal worker’s pneumoconiosis E. Silicosis 4. Which descriptor is given for the lymph node calcications (Fig. 112.3)? A. Popcorn B. Toothpaste C. Stippled D. Onion skin E. Egg shell
Fig. 112.1
Fig. 112.2
Fig. 112.3
113
Case 113 History: 56-year-old woman asthmatic patient with fevers and cough (Figs.113.1, 113.2 and 113.3). 1. Which entities may manifest with a peripheral consolidation pattern? (Choose all that apply.) A. Cryptogenic organizing pneumonia B. Chronic eosinophilic pneumonia C. Granulomatosis with polyangiitis D. Pulmonary infarcts E. Simple pulmonary eosinophilia 2. Which cell type is implicated in this disorder? A. Neutrophil B. Erythrocyte C. Langerhans cell D. Eosinophil 3. More than 50% of patients with this disorder will have what associated condition? A. Rheumatoid arthritis B. Takayasu arteritis C. Asthma D. Sarcoidosis 4. What is the most appropriate treatment for the airspace opacities in this condition? A. Doxycycline B. Oseltamivir C. Corticosteroids D. Pembrolizumab E. Montelukast
Fig. 113.1
114
Fig. 113.2
Fig. 113.3
Case 114 History: 77-year-old man with chronic obstructive pulmonary disease (COPD) and incidental nding on computed tomography (CT). 1. What is the salient CT abnormality (Fig. 114.1)? A. Tracheal diverticulum B. Apical paraseptal emphysema C. Tracheal rupture D. Pneumomediastinum 2. What is the most common location of this nding? A. Left paratracheal, T1-T2 B. Left paratracheal, T10-T11 C. Right paratracheal, T1-T2 D. Right paratracheal, T10-T11 3. With which clinical scenario is this nding most commonly associated? A. Recurrent aspiration B. Reduced forced expiratory volume/forced vital capacity (FEV1/FVC) ratio C. Smoking D. Histoplasmosis infection
114.1
115
Case 115 History: 66-year-old man undergoing standard postoperative evaluation after coronary artery bypass. 1. What is the salient radiographic abnormality (Figs. 115.1 and 115.2)? A. Sternal wire fracture B. Sternal wire unraveling C. Sternal wire displacement D. Mediastinal hematoma 2. The detection of “wandering wires” implies what underlying sternotomy complication? A. Sternal hematoma B. Mediastinitis C. Sternal dehiscence D. Sternal non-union
Fig. 115.1
116
3. During which postoperative time period may this complication be detected? A. First week B. First month C. 6 months to 1 year D. Greater than 1 year E. Any 4. Which of the following may increase the risk for this complication? A. Reoperation B. Internal mammary bypass graft C. Diabetes mellitus D. Chronic obstructive pulmonary disease E. All of the above
Fig. 115.2
Case 116 History: 61-year-old male with prior tonsillar abscess and worsening fevers. 1. What is the concerning abnormality depicted on computed tomography (CT) (Figs. 116.1 and 116.2)? A. Malpositioned endotracheal tube B. Malpositioned enteric tube C. Mediastinal fat inammation D. Aortic dissection 2. Which of the following are potential causes for the imaging ndings? (Choose all that apply.) A. Sternal osteomyelitis B. Mediastinal hemorrhage
Fig. 116.1
C. Mediastinal radiation D. Esophageal perforation E. Retropharyngeal infection 3. What is the most likely diagnosis? A. Post-traumatic mediastinal hemorrhage B. Diffuse lymphangiomatosis C. Acute descending mediastinitis, retropharyngeal extension D. Acute mediastinitis after median sternotomy E. Acute mediastinitis after esophageal trauma
Fig. 116.2
117
Case 117 History: 56-year-old otherwise healthy, afebrile woman with cough. 1. What is the distribution of the parenchymal abnormalities in the rst image (Fig. 117.1)? A. Peribronchovascular, focal B. Peripheral, focal C. Peribronchovascular, diffuse D. Peripheral, diffuse 2. What imaging sign is demonstrated in the second image after the patient was treated with corticosteroids (Fig. 117.2)? A. Computed tomography (CT) halo sign B. Reverse CT halo sign C. Air bronchogram sign D. Random
3. What is the most likely diagnosis in this case? A. Bacterial pneumonia B. Aspiration pneumonia C. Pulmonary infarcts D. Cryptogenic organizing pneumonia E. Multifocal mucinous adenocarcinoma 4. What clinical entities may be associated with this imaging pattern? (Choose all that apply.) A. Drug reaction B. Collage vascular disorders C. Vasculitis D. Aspiration pneumonia E. Eosinophilic pneumonia
Fig.117.2
Fig. 117.1
118
Case 118 History: 53-year-old male with history of asthma, restrictive cardiomyopathy, and incidental nding on chest computed tomography (CT) (Fig. 118.1). 1. What of the following entities affect only the cartilaginous portions of the tracheal wall? A. Relapsing polychondritis B. Tracheobronchopathia osteochondroplastica C. Granulomatosis with polyangiitis D. Chondrosarcoma E. Amyloidosis 2. What is a possible complication of this condition? A. Hemoptysis B. Reduced diffusing capacity for carbon monoxide (DLCO) C. Swelling of ears D. Protein-losing enteropathy 3. What is the histopathologic abnormality? A. Submucosal deposition of abnormal protein B. Cartilage inammation and destruction C. Necrotizing granulomatosis vasculitis D. Non-caseating granulomas in airway wall 4. What is the most likely diagnosis in this case? A. Tracheobronchial amyloidosis B. Relapsing polychondritis C. Granulomatosis with polyangiitis D. Adenoid cystic carcinoma E. Endobronchial metastases
Fig. 118.1
119
Case 119 History: 43-year-old otherwise healthy man with recurrent episodes of dyspnea (Figs. 119.1 and 119.2). 1. What should be included in the differential diagnosis for a unilateral hyperlucent hemithorax? (Choose all that apply.) A. Pneumothorax B. Mastectomy C. Endobronchial obstruction D. Bronchial atresia E. Fibrosing mediastinitis 2. What is the most likely underlying pathophysiology of the imaging ndings in this case? A. Autosomal recessive mutation in cystic brosis transmembrane conductance regulator (CFTR) gene B. X-linked agammaglobulinemia C. Constrictive bronchiolitis, post-infectious D. Constrictive bronchiolitis, transplant-related 3. Which feature of clinical history is typically associated with these ndings? A. Acute chest pain B. Childhood viral bronchiolitis C. Chest wall radiation D. Inhabitation of Ohio River Valley 4. By which eponym is this condition known? A. Poland syndrome B. Swyer-James-MacLeod syndrome C. Westermark D. Williams-Campbell
Fig. 119.2
120
Fig. 119.1
Case 120 History: 53-year-old woman with remote history of hysterectomy. 1. Based on the chest radiographic ndings (Fig. 120.1), what is the distribution pattern of the pulmonary nodules? A. Peripheral predominant B. Upper lung zone predominant C. Lower lung zone predominant D. Central predominant 2. What is the anatomic distribution of nodules on the corresponding computed tomography (CT) (Fig. 120.2)? A. Centrilobular, branching B. Centrilobular, non-branching
Fig. 120.1
C. Perilymphatic D. Random 3. What is the most common cause of this imaging pattern? A. Hematogenous metastases B. Lymphatic metastases C. Aspiration bronchiolitis D. Respiratory bronchiolitis E. Excipient lung disease
Fig. 120.2
121
Case 121 History: 68-year-old man with crackles on auscultation (Figs. 121.1 and 121.2). 1. What is the best diagnosis? A. Idiopathic pulmonary brosis B. Asbestosis C. Hypersensitivity pneumonitis (HP) D. Sarcoidosis 2. What proportion of usual interstitial pneumonia (UIP) cases can be condently diagnosed solely on chest computed tomography (CT)? A. 20% B. 30% C. 50% D. 70%
Fig. 121.1
122
3. What is the most helpful ndings in differentiating brotic HP from UIP and nonspecic interstitial pneumonitis (NSIP)? A. Subpleural sparing B. Basilar sparing C. Reticulation D. Traction bronchiectasis 4. Which of the following features occurs with equal frequency in chronic beryllium disease and sarcoidosis? A. Isolated lymphadenopathy B. Pulmonary nodularity C. Pulmonary brosis D. Airway involvement
Fig. 121.2
Case 122 History: 45-year-old woman with chronic cough and sputum production. 1. What is the best diagnosis (Figs.122.1 and 122.2)? A. Mounier-Kuhn syndrome B. Tracheomalacia C. Tracheobronchopathia osteochondroplastica D. Relapsing polychondritis 2. Which of the following is not a common nding in MounierKuhn syndrome? A. Bronchiectasis B. Tracheal diverticula C. Hyperination D. Nodules 3. What is the treatment for tracheobronchopathia osteochondroplastica? A. Surgical resection B. Laser ablation C. Immunotherapy D. No treatment 4. What portion of the tracheal wall is free of cartilaginous support? A. Anterior wall B. Lateral walls C. Posterior wall D. Trachea has circumferential cartilaginous rings
Fig. 122.2
Fig. 122.1
123
Case 123 History: 59-year-old woman with chronic dyspnea and history of “asthma.” 1. What is the best diagnosis (Figs.123.1 and 123.2)? A. Saber sheath trachea B. Tracheomalacia C. Relapsing polychondritis D. Tracheal rupture 2. During which phase of the respiratory cycle is tracheomalacia most evident? A. Dynamic inspiration B. Dynamic expiration C. End-inspiration D. End-expiration
3. If the tracheal wall is thickened circumferentially but the posterior wall is spared what other condition (other than relapsing polychondritis) should be considered in the differential diagnosis? A. Amyloidosis B. Tuberculosis C. Granulomatosis with polyangiitis D. Tracheobronchopathia osteochondroplastica 4. What is the name ascribed to a trachea in which the transverse diameter is wider than the anterior-posterior diameter? A. Bowed trachea B. Parabolic trachea C. Lunate trachea D. Quadrilateral trachea
Fig. 123.2 Fig. 123.1
124
Case 124 History: Middle-aged man with shortness of breath and wheezing. 1. Which of the following diagnoses should be included on the differential diagnosis (Figs.124.1 and 124.2)? (Choose all that apply.) A. Metastasis B. Squamous cell carcinoma C. Adenoid cystic carcinoma D. Leiomyoma 2. What degree of tracheal narrowing is typically present before airway symptoms develop in primary tracheal malignancy? A. 25% B. 50% C. 75% D. 90%
3. Which of the listed primary malignancies is most likely to spread hematogenously to the trachea? A. Melanoma B. Testicular cancer C. Thyroid cancer D. Pancreatic cancer 4. In addition to lipomas, what benign airway tumor commonly contains fat? A. Leiomyoma B. Hamartoma C. Adenoma D. Papilloma
Fig. 124.2
Fig. 124.1
125
Case 125 History: 53-year-old man with history of heart transplant with new fever (Fig.125.1). 1. Which of the following diagnoses infectious etiologies are most likely in this setting? A. Histoplasmosis B. Nocardiosis C. Tuberculosis D. Aspergillosis 2. What is the most common cause of death in the rst 30 days after cardiac transplant? A. Graft failure B. Infection C. Acute allograft rejection D. Cardiac allograft vasculopathy 3. Which of the following entities is least likely to manifest with a tree-in-bud pattern on computed tomography (CT)? A. Acute tuberculosis B. Invasive aspergillosis C. Chronic aspiration D. Cystic brosis
Fig. 125.1
126
Case 126 History: 69-year-old man with chronic shortness of breath (Figs.126.1 and 126.2). 1. What is the most likely diagnosis? A. Proximal interruption of the pulmonary artery B. Pulmonary artery sarcoma C. Primary pulmonary hypertension D. Chronic thromboembolic disease 2. Which of the following is the likely cause of mosaic attenuation? A. Air-trapping B. Left-sided heart failure C. Vasculitis D. Infection
3. Which of the following computed tomography (CT) ndings is specic for chronic as opposed to acute thromboembolic disease? A. Mosaic attenuation B. Ground-glass opacity C. Central arterial lling defect D. Decreased arterial caliber 4. What pulmonary nding is invariably present in long-standing proximal interruption of pulmonary artery? A. Mosaic attenuation B. Consolidation C. Bronchiectasis D. Fibrosis/scarring
Fig. 126.1
Fig. 126.2
127
Case 127 History: Middle-aged man with shortness of breath (Figs.127.1 and 127.2). 1. What is the most likely diagnosis? A. Sarcoidosis B. Silicosis C. Langerhans cell histiocytosis D. Metastatic disease 2. Which of the following disease processes most commonly present with a random pattern of diffuse nodular lung disease on computed tomography (CT)? A. Disseminated infection B. Silicosis C. Hypersensitivity pneumonitis D. Chronic beryllium disease
Fig. 127.1
128
3. What is the typical zonal distribution for silicosis? A. Upper B. Mid C. Lower D. Diffuse 4. A woman with an occupational history of nuclear reactor manufacturing presents with chronic shortness of breath. What is the next step in management? A. Puried protein derivative (PPD) skin test B. Chest CT C. Beryllium lymphocyte proliferation test (BeLPT) D. Ventilation/perfusion scan
Fig. 127.2
Case 128 History: 35-year-old woman with an incidental computed tomography (CT) nding (Fig.128.1). 1. What should be included in the differential diagnosis? A. Sarcoidosis B. Tuberculosis C. Castleman’s disease D. Hypervascular metastasis 2. Lymph node metastases from which of the following primary malignancies are typically not hypervascular? A. Melanoma B. Renal cell carcinoma C. Colon cancer D. Thyroid cancer 3. Which of the following is not common in the plasma cell variant of Castleman’s disease? A. Avid nodal enhancement B. Systemic symptoms C. Multicentric involvement D. Progression to lymphoma 4. Which chronic infection predisposes to the development of human herpesvirus 8 (HHV-8) associated Castleman’s disease? A. Tuberculosis B. Human immunodeciency virus (HIV) C. Hydatid disease D. Nocardiosis Fig. 128.1
129
Case 129 History: 37-year-old man with chest tightness and dyspnea. 1. What is the diagnosis (Figs.129.1 and 129.2)? A. Penetrating aortic ulceration B. Intramural hematoma C. Aortic dissection D. Motion artifact 2. What is the next step in management? A. Surgery B. Watchful waiting C. Blood pressure control D. B and C 3. Which portion of the aorta is most commonly affected by penetrating aortic ulcerations? A. Sinus of Valsalva B. Ascending thoracic aorta
Fig. 129.1
130
C. Aortic arch D. Descending thoracic aorta 4. In the setting of ascending aortic intramural hematoma, into which other vessel wall can hematoma extend? A. Right pulmonary artery B. Superior vena cava C. Left pulmonary vein D. Azygous vein
Fig. 129.2
Case 130 History: 53-year-old woman with chronic dyspnea. 1. What is the best diagnosis (Fig.130.1)? A. Granulomatosis with polyangiitis B. Relapsing polychondritis C. Sarcoidosis D. Amyloidosis 2. What portion of the central airways is usually involved in the setting of granulomatosis with polyangiitis? A. Subglottic B. Carina C. Mid-trachea D. Proximal bronchi 3. Which of the following can present with nodular hyperdense foci within the tracheal walls? A. Sarcoidosis B. Relapsing polychondritis C. Amyloidosis D. Granulomatosis with polyangiitis 4. Which of the following conditions can lead to diffuse tracheal dilation? A. Emphysema B. Asthma C. Pulmonary brosis D. Pleural thickening
Fig. 130.1
131
Case 131 History: 23-year-old man with shortness of breath and chronic cough. History of severe childhood pulmonary infection. 1. What of the following should be included in the differential diagnosis (Figs.131.1 and 132.2)? A. Congenital lobar emphysema B. Swyer-James-Macleod syndrome C. Bronchial atresia D. Obliterative bronchiolitis 2. Hyperdense bronchial mucus plugging is typical of which condition? A. Bronchial atresia B. Obliterative bronchiolitis C. Williams-Campbell syndrome D. Allergic bronchopulmonary aspergillosis 3. Computed tomography (CT) performed in expiration is characterized by inward bowing of which? A. Posterior wall of the trachea B. Anterior wall of the trachea
C. Posterior wall of the bronchus intermedius D. Anterior wall of the bronchus intermedius 4. In an older female patient with long history of asthma, a chest CT showed severe air-trapping and multiple, well-dened pulmonary nodules. Biopsy of the dominant nodule was diagnostic of a well-differentiated carcinoid tumor. What is the best diagnosis? A. Granulomatous lymphocytic interstitial lung disease B. Carney complex C. Erdheim-Chester disease D. Diffuse idiopathic pulmonary neuroendocrine cell hyperplasia
Fig. 131.2
Fig. 131.1
132
Case 132 History: Middle-aged man with chronic shortness of breath. 1. What is the correct diagnosis (Figs.132.1 and 132.2)? A. Tracheal bronchus B. Bridging bronchus C. Bronchial atresia D. Cardiac bronchus 2. Which condition is associated with a bridging bronchus? A. Right pulmonary artery atresia B. Interruption of the right pulmonary artery C. Pulmonary artery sling D. Patent ductus arteriosus
3. Which lobe is most commonly affected by bronchial atresia? A. Left upper lobe B. Right upper lobe C. Left lower lobe D. Right lower lobe 4. In the setting of a tracheal bronchus (arising from the distal trachea), an endotracheal tube position close to the carina would most likely lead to atelectasis of which pulmonary lobe? A. Left upper lobe B. Right upper lobe C. Left lower lobe D. Right lower lobe
Fig. 132.2
Fig. 132.1
133
Case 133 History: Young adult with recurrent pneumonia. 1. What is the most likely diagnosis (Figs.133.1 and 133.2)? A. Pulmonary sequestration B. Congenital pulmonary airway malformation C. Bronchogenic cyst D. Pneumatocele 2. Patients with extralobar sequestrations typically present at which age? A. Infancy B. Adolescence C. 20 to 30 years of age D. 30 to 40 years of age
3. What is the typical treatment for congenital pulmonary airway malformations that have come to clinical attention? A. Long-term antibiotics B. Surgical resection C. Annual imaging surveillance D. No treatment 4. Which pulmonary lobe is most commonly affected by bronchogenic cysts? A. Upper lobes B. Right middle lobe/lingula C. Lower lobes D. All lobes are equally affected
Fig. 133.2 Fig. 133.1
134
Case 134 History: Dyspnea. 1. Which of the following should be included in the differential diagnosis for this patient (Figs.134.1 and 134.2)? (Choose all that apply.) A. Acute pulmonary embolism B. Flow artifact C. Pulmonary artery sarcoma D. Pulmonary artery pseudoaneurysm 2. Which other important nding is present? A. Pulmonary infarct B. Pulmonary hemorrhage C. Right heart strain D. Aortic dissection
Fig. 134.1
3. What is the radiographic sign used to describe oligemia distal to an obstructing embolus? A. S sign of Golden B. Hampton hump C. Fleischner sign D. Westermark sign 4. What nding on magnetic resonance imaging (MRI) best distinguishes pulmonary thromboembolism from pulmonary artery sarcoma? A. High T2 signal intensity B. Enhancement with gadolinium C. Shape of lling defect D. High T1 signal intensity
Fig. 134.2
135
Case 135 History: Dyspnea. 1. Which of the following should be included in the differential diagnosis for this patient (Figs. 135.1, 135.2 and 135.3)? (Choose all that apply.) A. Coal-worker’s pneumoconiosis B. Alpha-1 Antitrypsin deciency C. Intravenous drug abuse D. Nontuberculous mycobacterial infection 2. What type of emphysema predominates in this patient? A. Centrilobular B. Paraseptal C. Panlobular D. Paracicatricial 3. What is the pattern of inheritance of alpha-1 antitrypsin deciency? A. Autosomal dominant B. Autosomal recessive C. X-linked dominant D. X-linked recessive 4. Which of the following is another established complication of alpha-1 antitrypsin deciency? A. Liver cirrhosis B. Pancreatitis C. Renal cell carcinoma D. Aortic aneurysm
Fig. 135.2
Fig. 135.1
Fig. 135.3
136
Case 136 History: Abnormal chest radiograph. 1. Based on the computed tomography (CT) ndings (Fig.136.1), which of the following should be included in the differential diagnosis for this patient. (Choose all that apply.) A. Thymoma B. Lymphoma C. Thymic cyst D. Normal thymus 2. Which of the following is not a common cause of mixed solid and cystic anterior mediastinal mass? A. Thymoma B. Germ cell neoplasm C. Hodgkin lymphoma D. Pericardial cyst
3. Which of the following is the most common cause of a thymic mass? A. Thymic cyst B. Thymolipoma C. Thymic carcinoid D. Thymoma 4. If a thymic cyst has been complicated by hemorrhage, how would it most likely appear on T1-weighted magnetic resonance imaging (MRI)? A. Low T1 signal intensity B. High T1 signal intensity C. No T1 signal intensity D. Intermediate T1 signal intensity
Fig. 136.1
137
Case 137 History: Lung nodule. 1. What congenital abnormality is present (Figs.137.1 and 137.2)? A. Pulmonary sling B. Tracheal bronchus C. Cardiac bronchus D. Complete tracheal rings 2. What portion of the lung does a tracheal bronchus usually supply? A. Right upper lobe B. Right middle lobe C. Left upper lobe D. Left lower lobe
Fig. 137.1
138
3. What is the most common clinical presentation of a tracheal bronchus? A. Recurrent infection B. Hemoptysis C. Stridor D. Incidental nding 4. What term describes a tracheal bronchus supplying the entire right upper lobe? A. Cow bronchus B. Pig bronchus C. Sheep bronchus D. Horse bronchus
Fig. 137.2
Case 138 History: Dyspnea, heart disease. 1. Which of the following should be included in the differential diagnosis for this patient (Figs. 138.1, 138.2 and 138.3)? (Choose all that apply.) A. Amiodarone toxicity B. Renal cell carcinoma C. Talcosis D. Bleomycin toxicity 2. Considering the high-attenuation liver parenchyma, what is the most likely cause of these nodules? A. Amiodarone toxicity B. Renal cell carcinoma C. Talcosis D. Bleomycin toxicity
Fig. 138.1
3. What disorder is amiodarone used to treat? A. Acute respiratory distress syndrome (ARDS) B. Cardiac dysrhythmia C. Myocarditis D. Congestive heart failure 4. Approximately what percentage of patients treated with amiodarone develop pulmonary toxicity? A. 75%
Fig. 138.3
Fig. 138.2
139
Case 139 History: Chest pain. 1. Which of the following should be included in the differential diagnosis for this patient (Fig.139.1)? (Choose all that apply.) A. Lymphoma B. Aneurysm C. Pseudoaneurysm D. Thymoma 2. What is the most severe complication of this abnormality? A. Rupture B. Myocardial ischemia C. Fistula formation D. Compression of the superior vena cava (SVC)
Fig. 139.1
140
3. Approximately what percent of patients with venous graft aneurysms contain thrombus? A. 5% B. 15% C. 25% D. 50% 4. What is the best next step in the management of this patient? A. Follow-up radiograph in 3 months B. Cardiac magnetic resonance imaging (MRI) C. Surgical repair D. Coil embolization
Case 140 History: Cough and fever. 1. Which of the following should be included in the differential diagnosis for this patient (Figs.140.1 and 140.2)? (Choose all that apply.) A. Mycoplasma infection B. Staphylococcus aureus infection C. Respiratory syncytial virus (RSV) infection D. Streptococcus pneumoniae infection 2. Which of the following terms describes the nodular and linear branching centrilobular opacities due to small airways disease? A. Acinar nodule B. Tree-in-bud opacity C. Ring shadow D. Mosaic attenuation
3. Which of the following regarding bronchioles is true? A. Bronchioles contain cartilage. B. Normal bronchioles are visible on computed tomography (CT). C. Respiratory bronchioles communicate directly with alveoli. D. Terminal bronchioles participate in gas exchange. 4. Where in the pulmonary lobule are bronchioles located? A. Center B. Interlobular septa C. Subpleural interstitium D. Adjacent to the pulmonary vein
Fig. 140.1
Fig. 140.2
141
Case 141 History: Lung transplantation 2 months ago. Low-grade fever and nonproductive cough. 1. Which of the following should be included in the differential diagnosis for this patient (Figs.141.1 and 141.2)? (Choose all that apply.) A. Tuberculosis B. Drug reaction C. Viral pneumonia. D. Post-transplant lymphoproliferative disorder (PTLD) 2. True or false? Pneumocystis jiroveci pneumonia a common opportunistic infection in transplant recipients. A. True B. False 3. Which of the following is the most common viral pneumonia in solid organ transplant recipients? A. Epstein-Barr virus (EBV) B. Cytomegalovirus (CMV) pneumonia
Fig. 141.1
142
C. Respiratory syncytial virus (RSV) pneumonia D. Adenovirus pneumonia 4. Which of the following regarding renal transplantation is not true? A. Opportunistic infections are uncommon during the rst month after transplant. B. Streptococcus pneumoniae is a common cause of pneumonia after the sixth month following transplant. C. T-cell mediated immunity is most severely depressed during the second through sixth months after transplant. D. Fungal pneumonia is uncommon after the sixth month following transplant.
Fig. 141.2
Case 142 History: Severe pulmonary hypertension. 1. Which of the following should be included in the differential diagnosis for this patient (Figs. 142.1, 142.2 and 142.3)? (Choose all that apply.) A. Diffuse alveolar hemorrhage B. Usual interstitial pneumonia C. Pulmonary veno-occlusive disease (PVOD) D. Noncardiogenic pulmonary edema 2. What is the most likely diagnosis based on the ndings of severe pulmonary hypertension and pulmonary edema? A. PVOD B. Chronic pulmonary thromboembolic disease C. Chronic obstructive pulmonary disease (COPD) D. Primary pulmonary hypertension 3. Which one of the following is true regarding PVOD? A. Prognosis is excellent. B. Most patients are asymptomatic at diagnosis.
C. Venous and venule obstruction results from intimal brosis. D. Central pulmonary veins are dilated on computed tomography (CT). 4. Which of the following is the best next step in the management of this patient with suspected PVOD? A. Cardiac magnetic resonance imaging (MRI) B. Surgical biopsy C. Bronchoalveolar lavage (BAL) D. Fluorodeoxyglucose–positron emission tomography (FDG-PET)/CT
Fig. 142.2 Fig. 142.1
Fig. 142.3
143
Case 143 History: Cough. 1. Which of the following should be included in the differential diagnosis for this patient (Figs.143.1 and 143.2)? (Choose all that apply.) A. Tracheobronchial papillomatosis B. Relapsing polychondritis C. Bacterial tracheitis D. Amyloidosis 2. How can magnetic resonance imaging (MRI) help distinguish amyloid from other causes of tracheal masses? A. Amyloid has high signal intensity on both T1- and T2-weighted images. B. Amyloid has low signal intensity on both T1- and T2-weighted images. C. Amyloid has low signal intensity on T1- and high signal intensity on T2-weighted images. D. Amyloid has high signal intensity on T1- and low signal intensity on T2-weighted images.
Fig. 143.1
144
3. Which virus is most associated with this condition? A. Human immunodeciency virus B. Human papilloma virus (HPV) C. Epstein-Barr virus (EBV) D. Cytomegalovirus (CMV) 4. Which of the following is not a complication of respiratory papillomatosis? A. Life-threatening airway obstruction B. Hemoptysis C. Adenocarcinoma D. Recurrent infection
Fig. 143.2
Case 144 History: Abnormal chest radiograph. Cough. 1. What infection is associated with a mass within a cyst, as shown in the gure (Figs.144.1 and 144.2)? A. Aspergillosis B. Nocardiosis C. Echinococcosis D. Mucormycosis 2. Which term is used to describe air collection within the cyst in the rst gure? A. Crescent sign B. Water lily sign C. Tip-of-the-iceberg sign D. Collar sign
3. What is the signicance of the crescent sign in the rst gure? A. Secondary infection B. Impending rupture C. Healing D. Hemorrhage 4. Which of the following is the most common site of hydatid cysts in humans? A. Lung B. Brain C. Liver D. Spleen
Fig. 144.1 Fig. 144.2
145
Case 145 History: Cough. 1. Which of the following should be included in the differential diagnosis for this patient (Figs.145.1 and 145.2)? (Choose all that apply.) A. Lung abscess B. Lymphangioleiomyomatosis C. Congenital pulmonary airway malformation D. Bronchogenic cyst 2. Which of the following congenital lung lesions is characterized by systemic arterial supply? A. Bronchogenic cyst B. Congenital pulmonary airway malformation C. Pulmonary sequestration D. Congenital lobar overination
Fig. 145.1
146
3. What is the most common location of a bronchogenic cyst? A. Right lower lobe B. Left lower lobe C. Anterior mediastinum D. Middle mediastinum 4. Patients with congenital pulmonary airway malformations are at increased risk for developing which of the following tumors? A. Lymphoma B. Pleuropulmonary blastoma C. Adenocarcinoma D. Small cell lung carcinoma
Fig. 145.2
Case 146 History: Xerostomia and facial swelling. 1. Which of the following should be included in the differential diagnosis for this patient (Figs.146.1 and 146.2)? (Choose all that apply.) A. Usual interstitial pneumonia (UIP) B. Lymphangioleiomyomatosis C. Pulmonary Langerhans cell histiocytosis D. Lymphocytic interstitial pneumonia 2. Lymphocytic interstitial pneumonia is most commonly associated with which of the following? A. Systemic lupus erythematosus B. Sjögren syndrome C. Idiopathic pulmonary brosis D. Acute interstitial pneumonia
Fig. 146.1
3. What is the predominant computed tomography (CT) nding of Pneumocystis jirovecii pneumonia? A. Ground-glass opacity B. Consolidation C. Lung cysts D. Lymphadenopathy 4. Which of the following is not associated with lung cysts? A. Lymphangioleiomyomatosis B. Pulmonary Langerhans cell histiocytosis C. Respiratory bronchiolitis–associated interstitial lung disease D. Birt-Hogg-Dubé syndrome
Fig. 146.2
147
SECTION IV
Answers
Opening Round CASE 1
Imaging Findings
Squamous Cell Carcinoma of the Lung
PA and lateral hest radigraphs (Fig.1.1, Fig.1.2) shw a right lwer lbe mass. The mass and right hilar lymphadenpathy were nrmed n CT (Fig. 1.3, Fig. 1.4). Primary lung arinma mst mmnly presents as a slitary lung ndule r mass. Assiated hilar and mediastinal lymphadenpathy may be apparent n hest radigraphs, althugh CT is superir t radigraphy fr deteting thrai lymphadenpathy. Other assiated ndings an inlude bne destrutin (hest wall invasin r metastasis), pleural effusin, lung metastases, and septal thikening frm lymphangiti arinmatsis. Patients with radigraphs suggestive f lung arinma shuld underg hest CT fr further evaluatin. Flurdexygluse–psitrn emissin tmgraphy/CT (FDG-PET/CT) is used t stage patients with lung aner and is partiularly valuable in deteting ult metastases. Suh upstaging f lung aner urs frequently after PET imaging and an ften alter therapy. Beause f the intrinsially high level f FDG uptake in the brain, magneti resnane imaging (MRI) is used t evaluate fr brain metastasis in patients with lung aner.
1. A and B. The mst mmn auses f a slitary lung ndule r mass are neplasms and infetin. The mst likely neplasti ause is primary lung arinma. Infetins that present as large ndules r masses inlude lung absesses, nardisis, atinmysis, and fungal infetin suh as blastmysis r invasive aspergillsis. Oasinally, mmunity-aquired baterial infetins an present as a rund mass during the early stages f infetin (rund pneumnia). Althugh pulmnary sequestratins an present as medially lated lwer lbe masses n hest radigraphy, lymphadenpathy is nt a typial nding unless the sequestratin is infeted. 2. A. Adenarinma is the mst mmn histpathlgi type f lung aner. Squamus ell arinma and small ell arinma are smewhat less mmn. Carinid aunts fr apprximately nly 1% f all lung malignanies. 3. C. Cigarette smking is the signal largest risk fatr fr develping lung arinma. Other risk fatrs inlude upatinal and envirnmental expsures suh as asbests and silia. Radn may be a fatr ntributing t lung aner in sme patients. Obesity has nt been shwn t be an independent risk fatr fr lung arinma. 4. D. Stage at diagnsis is the best preditr f 5-year survival with lung arinma. The urrent staging system fllws the T(umr), N(de), M(etastasis) mdel and desribes lal, reginal, and systemi extent f malignany.
Comment Differential Diagnosis Primary lung arinma is the leading diagnsti nsideratin. Other entities t nsider inlude infetins partiularly fungal r baterial nditins suh as atinmysis r nardisis.
Lung Cancer Lung aner remains the leading ause f aner-related death in bth males and females. Adenarinma is the mst mmn histpathlgi type f lung aner, fllwed by squamus ell and small ell arinma. Other histpathlgi types are muh less mmn. Cigarette smking remains the leading ause f lung aner, as 85% f lung aners develp in urrent r frmer smkers. Mst lung aners are deteted at later stages when patients beme symptmati. Chest radigraphy has nt been shwn t redue mrtality when used as a sreening test in patients at risk f lung aner. Hwever, the Natinal Lung Sreening Trial shwed a 20% redutin in all-ause mrtality with lw-dse mputed tmgraphy (CT) sreening in patients deemed at highest risk fr develping lung aner. Earlier stage lung aner is assiated with higher survival rates.
148
REFERENCES
Maldnad F, Jett JR. Invasive and nninvasive advanes in the staging f lung aner. Semin Oncol. 2014 Feb;41(1):17–27. Thrai Radilgy: The Requisites, 3rd ed, 405–427.
CASE 2 Spontaneous Pneumothorax 1. C. Playing basketball des nt invlve sufient trauma t prdue a traumati pneumatele. There is n radigraphi evidene f pneummediastinum. Spntaneus pneumthrax typially urs in healthy yung individuals, ften during athleti ativities. Swyer-James-MLed syndrme is haraterized n radigraphy by a hyperluent lung with slightly dereased lung vlume and a small ipsilateral hilum. 2. B. A traumati pneumatele results frm mpressin-dempressin trauma f the hest during partial airway lsure, ausing rupture f small airways. 3. B. Swyer-James-MLed syndrme results frm pstinfetius nstritive brnhilitis, mst mmnly a result f hildhd infetin that is usually viral. There is redued bld w t the affeted lung and air trapping n expiratry imaging, whih manifests as unilateral hyperlueny f the affeted hemithrax. 4. D. Tensin pneumthrax manifests with harateristi displaements that result frm inreased intrathrai pressure in the affeted hemithrax, shifting the mediastinum tward the ppsite side, displaing the ipsilateral hemidiaphragm inferirly, and widening the ipsilateral interstal spaes.
Opening Round
Comment Differential Diagnosis Unilateral hyperluent lung (hemithrax) may result frm pneumthrax, Swyer-James-MLed syndrme, endbrnhial tumr r freign bdy, mastetmy, r Pland syndrme.
Discussion PA hest radigraph (Fig. 2.1 and 2.2) shws abnrmal lueny in the left hemithrax, with ateletasis f the left lung, shift f mediastinal strutures t the right, dwnward displaement f the left hemidiaphragm, and widening f left interstal spaes. Pneumthrax is dened as the presene f air r gas in the pleural spae. Althugh there is a wide variety f auses, spntaneus pneumthrax is the mst mmn etilgy. Affeted patients are typially in their third r furth deade f life. Spntaneus pneumthraes are almst always sendary t rupture f an apial bleb, whih represents an air pket within the elasti bers f the viseral pleura. Suh blebs have been reprted t be detetable n hest radigraphs in apprximately 15% f ases f spntaneus pneumthrax. Hwever, blebs are rarely evident n hest radigraphs fllwing reslutin f the pneumthrax. Cmputed tmgraphy (CT) is muh mre sensitive than radigraphy fr deteting blebs and has been shwn t detet blebs in apprximately 80% f patients fllwing reslutin f spntaneus pneumthraes. The size and number f apial blebs deteted n CT have been shwn t rrelate with the risk fr reurrent pneumthraes and the need fr surgial interventin. The ipsilateral and ntralateral rates f reurrene f spntaneus pneumthrax is apprximately 30% and 10%, respetively. Tensin pneumthrax is a life-threatening nditin. Affeted patients present with linial signs f tahypnea, tahyardia, yansis, sweating, and hyptensin. Radigraphi ndings may inlude ntralateral mediastinal shift, dwnward displaement f the ipsilateral hemidiaphragm, widening f interstal spaes. In severe ases, there may be attening f the nturs f the right heart brder and/r vena avae. REFERENCES
Sahn SA, Heffner JE. Spntaneus pneumthrax. N Engl J Med. 2000;342:868–874. Thrai Radilgy: The Requisites, 3rd ed, 159–192.
CASE 3 Pleural Plaques 1. C. Based n the harateristi radigraphi and mputed tmgraphy (CT) ndings, the mst likely diagnsis is C, asbests-related pleural plaques. Fibrthrax is typially a unilateral press and the pleural thikening and aliatin are harateristially ntinuus rather than disntinuus, and ften invlve and bsure the ardiphreni sulus. Tal pleurdesis may be perfrmed t treat intratable pleural effusin r hrni pneumthrax and ften results in fal r multifal areas f smth r ndular pleural thikening that ften exhibit high attenuatin r aliatin. The ndings typially invlve the psterir and basilar aspets f the treated hemithrax. Calied pleural metastases are unmmn but d ur in patients with stesarmatus malignanies and may mimi pleural plaques. 2. C. Apprximately 25% f patients with the primary pleural malignant mesthelima will als exhibit pleural plaques, an imaging marker f their previus asbests expsure.
149
3. D. The harateristi lateny perid is 30 t 40 years between the time f a persn’s upatinal expsure t asbests and the develpment f mesthelima. 4. E. All of the above. Asbests was ne an ingredient in many different nstrutin materials. In the mid-1960s, sientists nrmed that breathing airbrne asbests bers an ause a variety f lung diseases, inluding mesthelima, lung aner and asbestsis. Many buildings built befre 1980 were nstruted with asbests-ntaining materials inluding rfing materials, ement, r and eiling tiles, pipe and dut insulatin. Asbests was ne nsidered a ritial element in the shipbuilding industry, espeially in the military, beause it uld resist heat and prevent res that wuld be disastrus fr a vessel at sea. It was als regnized as a great insulatr and was resistant t rrsin. Many f the gds that were prdued and pressed in textile mills frm the 1940s t the 1970s were made with asbests bers, whih put many textile wrkers at risk f asbests expsure.
Comment Differential Diagnosis Pleural thikening resulting frm infetin, primary r sendary neplasia, r pleurdesis. Pleural plaques are the mst mmn manifestatin f asbests expsure and typially ur after a lateny perid f apprximately 15 t 20 years. They d nt ause symptms and are usually disvered inidentally, as in this ase. Pathlgially, pleural plaques are mpsed f dense bands f avasular llagen. Pleural plaques may be deteted in patients whse upatin r ther ativities invlved expsure t asbests bers (e.g., nstrutin, bat building). They are typially fund in hest radigraphi and CT examinatins perfrmed 15 t 20 years after suh expsure and have harateristi imaging features. Pleural plaques are nt premalignant lesins, but affeted individuals are at inreased risk fr lung aner, mesthelima, and asbestsis (pulmnary brsis) due t their asbests expsure. Plaques serve as a signpst f asbests expsure. On hest imaging, detetin f bilateral alied pleural plaques alng the diaphragmati pleura (S3.1 and S3.2) is nsidered diagnsti f asbests-related pleural disease. They manifest as bilateral multifal areas f disntinuus pleural thikening, with r withut aliatin, and typially ur adjaent t the 6th t 9th ribs, alng the diaphragmati pleura, and alng the right lwer paravertebral pleura (Fig. 3.1). When mre extensive, the larger plaques may have a distintive en face radigraphi appearane resembling the fringes f a hlly leaf. Pleural plaques may als exhibit inmplete brders n hest radigraphy, an imaging feature that ften urs in pleural and hest wall lesins. REFERENCES
Jamrzik E, etal. Clinial Review: Asbests-related Disease. Intern Med J. 2011;Feb 10 https://di.rg/10.1111/j.1445-5994. 2011. Thrai Radilgy: The Requisites, 3rd ed, 159–192.
CASE 4 Emphysema 1. A, C, and D. Fibrti (hrni) hypersensitivity pneumnitis is haraterized by imaging signs f brsis, namely vlume lss that ften invlves the mid and upper lung znes. The hest radigraph may be nrmal in patients with mild
150
SECTION IV
Answers
entrilbular emphysema and in early ases f lymphangileimymatsis (LAM), but in mderate t advaned ases the lungs may appear hyperinated. Mst patients with asthma have nrmal hest radigraphs, but hyperinatin f the lungs des ur in patients with severe disease and may be transient r xed. 2. B. Swyer-James-MLed syndrme is assiated with pstinfetius brnhilitis urring in hildhd, usually at 2.5 m) n lateral hest radigraphy. Hyperlueny f the upper lung znes urs as result f mderate t severe lung destrutin with dereased perfusin. Rapid tapering and attenuatin f pulmnary vessels is a harateristi radigraphi nding in the upper lung znes f affeted patients. Flattened hemidiaphragms ur beause f hyperinatin and air trapping and are usually mst apparent n the lateral view
Comment PA and lateral views f the hest shw hyperinatin, with attened hemidiaphragms and widening f the retrsternal spae (Fig. 4.1). On the PA view, the upper lung znes appear hyperluent, with a pauity f pulmnary vessels that demnstrate rapid tapering and attenuatin as they emanate frm the hila. The entral pulmnary arterial strutures appear dilated, nsistent with assiated pulmnary hypertensin. Emphysema is dened as permanent, abnrmal enlargement f airspaes distal t the terminal brnhile, ampanied by destrutin f their walls. Radigraphi abnrmalities in patients with emphysema are related t verinatin f the lungs and lung destrutin. The latter is haraterized by redued vasularity r the presene f bullae. Overinatin f the lungs may be haraterized by several ndings, mst ntably attening f the hemidiaphragms best appreiated n the lateral view and an inrease in the retrsternal air spae diameter. Chest radigraphi abnrmalities are usually evident in mderate t severe ases f emphysema, but radigraphs are frequently nrmal in patients with early emphysema. Thin-setin mputed tmgraphy (CT) is superir t hest radigraphs in deteting and haraterizing emphysema and has a high sensitivity and speiity fr establishing the diagnsis. Other nditins that may result in verinatin f the lungs are asthma and ysti brsis.
REFERENCES
Gietema HA, et al. Quantifying the extent f emphysema: fatrs assiated with radilgists’ estimatins and quantitative indies f emphysema severity using the ECLIPSE hrt. Acad Radiol. 2011;18(6):661–671. Raghu G, Remy-Jardin M, Ryersn CJ, et al. Diagnsis f Hypersensitivity Pneumnitis in Adults. An Ofial ATS/JRS/ALAT Clinial Pratie Guideline. Am J Respir Crit Care Med. 2020 Aug 1;202(3):e36–e69. Thrai Radilgy: The Requisites, 3rd ed, 391–404.
CASE 5 Sarcoidosis (Stage I) 1. C. Mediastinal and hilar lymphadenpathy may ur in saridsis, lymphma, metastati disease, infetin, and pneumnisis. In a yung patient with n systemi symptms, as in this ase, the presene f symmetri bilateral hilar and mediastinal lymphadenpathy is mst harateristi f saridsis. 2. E. Head and nek malignanies, melanma, breast aner, and geniturinary malignanies are all assiated with metastases t mediastinal and hilar lymph ndes. Other auses f bilateral symmetri hilar lymphadenpathy inlude lymphma, ther metastases (e.g., renal ell arinma, testiular aner) and pneumnisis (i.e., silisis) in upatinally expsed individuals. 3. C. Patients infeted by histplasmsis may be asymptmati r suffer frm prgressive pulmnary disease and pssible disseminatin; and mediastinal lymphadenpathy is mmn in bth aute and subaute ases. Blastmysis may manifest as aute r hrni pulmnary disease mimiking mmunity-aquired pneumnia and malignany, and may ultimately prgress t disseminated disease with utaneus, geniturinary, and bny lesins. Primary idiidmysis manifests with pulmnary invlvement and affeted patients are ften asymptmati. Symptmati patients usually present with fever, ugh, and hest pain, features that mimi mmunity-aquired pneumnia. Chest radigraphs may shw nslidatin, ndules, r peripheral, slitary thinwalled avities and parapneumni effusin. 4. D. Bilateral asymmetri hilar lymphadenpathy may ur in lymphma, saridsis, and metastati disease (e.g., renal ell arinma, testiular aner, breast aner, lung aner).
Comment Differential Diagnosis Saridsis, lymphma, metastati disease
Discussion Cmpsite image f PA hest radigraph and axial ntrast-enhaned mputed tmgraphy (CT) (Fig. 5.1). The hest radigraph shws right paratraheal and bilateral hilar lymphadenpathy, frming the radigraphi “1,2,3 sign” that is harateristi f saridsis (Stage I). CT images nrm thse areas f lymphadenpathy and reveals additinal invlvement f prevasular and subarinal lymph nde statins. Saridsis is a multisystem hrni inammatry disease f unknwn etilgy, haraterized by widespread nnaseating granulmas. Beause this pathlgi nding may als be seen in a variety f ther nditins, a diagnsis f saridsis requires nrdant radilgi, linial, labratry, pathlgi ndings, and exlusin f ther entities (espeially granulmatus infetins).
Opening Round
151
The hest radigraph is abnrmal in apprximately 90% f patients with saridsis. Bilateral, symmetri hilar lymph nde enlargement is the mst mmn radigraphi abnrmality and is frequently ampanied by bilateral mediastinal lymph nde enlargement. Lung parenhymal disease is usually ndular r retiulndular in appearane, with a prediletin fr the upper and mid-lung znes. On thin-setin CT, sarid granulmas typially manifest as small (2–3 mm) ndules, with a harateristi perilymphati distributin that inludes the peribrnhvasular lymphatis, the interlbular septa, and subpleural lymphatis (peripherally and alng the ssures). Apprximately 20% f patients with saridsis may prgress t have pulmnary brsis with upper lung arhitetural distrtin, vlume lss, and ysti hanges.
mediastinal strutures, vertebral bdy invasin, diret extensin f tumr t the ntralateral pleura, and the presene f distant metastases. The presene f any ne r mre f these ndings preludes surgial resetin. Patients with limited disease may be nsidered andidates fr attempted surgial ure by extrapleural pneumnetmy. Regardless f therapy, hwever, malignant mesthelima is almst always fatal.
REFERENCES
CASE 7
Little BP. Saridsis: verview f pulmnary manifestatins and imaging. Semin Roentgenol. 2015 Jan;50(1):52–64. Thrai Radilgy: The Requisites, 3rd ed, pp 355–376.
CASE 6 Mesothelioma 1. A, B, C, D. The mputed tmgraphy (CT) imaging features f pleural thikening that suggest the presene f malignany inlude the fllwing: >1 m in thikness, ndularity, irumferential grwth pattern within the invlved hemithrax, and invlvement f the mediastinal pleural. 2. C. Pleural metastasis is the mst mmn frm f pleural malignany, whereas mesthelima and pleural invlvement by lymphma are muh less mmn. 3. B. Althugh pleural plaques may be present in the setting f mesthelima, their presene is nt required fr a diagnsis f mesthelima. The earliest manifestatin f mesthelima is unilateral diffuse pleural thikening and a pleural effusin. 4. A. Epitheliid is the mst mmn histlgi subtype f mesthelima. Althugh all are assiated with a pr prgnsis, the sarmatid subtype is typially mst aggressive albeit least mmn.
Comment Differential Diagnosis Pleural metastases, mesthelima, lymphma, invasive thymma.
Discussion Althugh relatively rare, malignant mesthelima is the mst mmn primary neplasm f the pleura. Mst affeted individuals (80% f ases) have a histry f asbests expsure 30 t 40 years befre the diagnsis, typially thrugh their upatin. Males are affeted mre mmnly than females, likely reeting a gender disparity in the upatins assiated with asbests expsure (e.g., nstrutin and shipyard wrkers, re ghters). Affeted patients typially present with mplaints f hest pain and dyspnea. The mst mmn radigraphi nding is the presene f diffuse pleural thikening that is typially ndular and irregular in nguratin and ften irumferential within the invlved hemithrax (Figs. 6.1A and B). In sme ases, diffuse pleural thikening may be ampanied by a redutin in the size f the affeted hemithrax, with assiated ipsilateral shift f the mediastinum. Pleural effusin is ften present. CT and magneti resnane imaging (MRI) are superir t radigraphy in assessing the extent f disease. In many ases, the tw mdalities play mplementary rles in the evaluatin f resetability and may be useful in assessing fr transdiaphragmati extensin, diffuse hest wall invasin, invasin f vital
REFERENCES
Leung AN, Müller NL, Miller RR. CT in differential diagnsis f diffuse pleural disease. AJR Am J Roentgenol. 1990 Mar;154(3):487–492. Miller BH, Rsad-de-Christensn ML, Masn AC, etal. Malignant pleural mesthelima: radilgi pathlgi rrelatin. Radiographics. 1996;16:613–644. Thrai Radilgy: The Requisites, 3rd ed, 159–192
Junction Lines 1. D. The strutures indiated by the arrw and arrwhead are the anterir and psterir juntin lines, respetively. 2. A, B, C, and D. Pneummediastinum may develp in individuals wh engage in inhalatinal substane abuse and is als knwn t ur in the setting f a sustained Valsalva maneuver (e.g., vmiting, straining, weightlifting). It is als assiated with bstrutive lung disease and urs in up t 15% f patients with pulmnary brsis. 3. B. The nept f radilgi mediastinal mpartments was intrdued by Dr. Benjamin Felsn in the 1970s using the lateral hest radigraph t evaluate a deteted mediastinal mass and help frmulate a differential diagnsis. In his methd, the mediastinum was divided int anterir, middle, and psterir mpartments using anatmi landmarks visible n the lateral hest radigraph. Felsn’s anterir mpartment extends frm the sternum t a line drawn alng the anterir aspet f the trahea and alng the psterir aspet f the heart, and the middle mpartment extended frm that same line t a psterir line drawn alng the anterir third f eah thrai vertebral bdy. The psterir mpartment extends psterirly frm that line and inludes the paravertebral regin. This designatin differs frm the denitin f mediastinal mpartments used by anatmists and pathlgists in whih the heart upies the middle mediastinum. 4. B. Ttal ateletasis f either lung is assiated with inreased mpensatry mehanisms as mpared t lbar ateletasis. The anterir juntin line is typially displaed ipsilaterally tward the ateletati lung, and the ntralateral lung hyperinates in a mpensatry fashin and ften rsses the midline tward the ateletati lung. The ipsilateral hemidiaphragm may be elevated, partiularly in ttal ateletasis f the left lung, and there is ften marked ipsilateral shift f the mediastinum, partiularly f the anterir mediastinal mpnents sine the psterir mediastinal strutures are relatively tethered t paraspinal strutures.
Comment Differential Diagnosis Nne.
Discussion The ned-dwn frntal radigraph demnstrates the nrmal appearane f the anterir (arrw) and psterir (arrwhead) juntin lines (Fig. 7.1), frmed by the lse appsitin f viseral and parietal layers f pleura f bth lungs as they apprximate in
152
SECTION IV
Answers
the anterir and psterir aspets f the mediastinum, respetively. The anterir and psterir juntin lines may be seen n 24.5% t 57% and 32% f hest radigraphs, respetively. The anterir prtin f the thrax begins at the thrai inlet and the superir aspet f the anterir juntin line begins at the undersurfae f the laviles and typially urses bliquely frm right t left but may appear mre vertial in alignment as demnstrated in this ase. The psterir prtin f the thrax extends mre superirly than the anterir prtin and thus the psterir juntin line may be seen t extend abve the level f the laviles, as in this illustratin. It typially appears as a straight vertial line, ften visible thrugh the traheal air lumn. Obliteratin r abnrmal nvexity f the anterir juntin line suggests underlying anterir mediastinal disease (e.g., thyrid r thymi mass, lymphadenpathy, lipmatsis) whereas similar ndings related t the psterir juntin line suggest a psterir mediastinal abnrmality (e.g., esphageal mass, lymphadenpathy, arti disease, r neurgeni tumr). Either juntin line may als be displaed by vlume lss r hyperinatin f the surrunding lung A line typially measures 3 m being malignant is 5.23. A lung ndule dubling time between 30 and 400 days has a likelihd rati f 3.40 f being malignant. Ndules that duble in vlume in less than 30 days are mst likely benign, typially infetius. Lung aners ur mre mmnly in the upper lbes than the lwer lbes, with slight right upper lbe predminane. The likelihd rati f an upper lbe ndule being malignant is 1.22.
Comment Differential Diagnosis The hest radigraph demnstrates a densely alied ndule in the left lwer lung, regnized benign pattern. These alied ndules are typially the result f remte granulmatus infetin suh as tuberulsis r histplasmsis.
161
Solitary Pulmonary Nodule There are tw aepted radigraphi riteria fr a benign slitary pulmnary ndule: lak f grwth ver at least 2 years and identiatin f a benign aliatin pattern within a smthly marginated pulmnary ndule. Rughly half f all reseted slitary pulmnary ndules prve t be benign. Clinial indiatrs that suggest a benign diagnsis inlude age yunger than 35 years and histry f expsure t tuberulsis r residene in areas with endemi tuberulsis r fungal infetins. Suh indiatrs are, unfrtunately, insufiently spei t be helpful in mst individual ases. Fr patients with ndules that d nt meet the aepted radigraphi riteria fr benignany, nnntrast CT with thin-setin imaging is usually the preferred methd fr further evaluatin. CT is mre sensitive than nventinal radigraphs fr deteting alium and fat within a ndule. In ertain ases, CT imaging allws a ndent diagnsis f a spei benign entity suh as granulma, hamartma, arterivenus malfrmatin, pulmnary infartin, muid impatin, and pulmnary sequestratin. When CT is nndiagnsti, the methd f further evaluatin depends n patient harateristis and ndule mrphlgy. Nninvasive imaging mdalities inlude ntrast-enhaned CT t assess fr abnrmal ndule enhanement and urdexygluse(FDG)-psitrn emissin tmgraphy (PET)/CT imaging, whih relies n abnrmal gluse analgue (FDG) uptake t distinguish benign frm malignant ndules. Hwever, false psitive and false negative examinatins an ur with FDG-PET/ CT beause f inreased metabli ativity in inammatry ndules and lw metabli ativity in lw-grade malignanies. REFERENCES
Erasmus JJ, Cnnly JE, MAdams HP, Rggli VL. Slitary pulmnary ndules: part I. Mrphlgi evaluatin fr differentiatin f benign and malignant lesins. Radiographics. 2000;20:43–58. Erasmus JJ, MAdams HP, Cnnly JE. Slitary pulmnary ndules: part II. Evaluatin f the indeterminate ndule. Radiographics. 2000;20:59–66. Thrai Radilgy: The Requisites, 3rd ed, 428–440.
CASE 25 Radiation Pneumonitis 1. C and D. The sharp and relatively straight lateral and superir margins, nnanatmi distributin n the lateral view, and distrtin and vlume lss are highly suggestive f radiatin indued injury beause the inammatin urs nly within the radiatin prt. Infetin an ause perihilar dense nslidatin as well, althugh margins are typially less well dened exept when nslidatin abuts a pulmnary ssure. Saridsis an ause perihilar and entral paities, whih asinally appear nslidative, but it is nearly always bilateral and typially symmetri. Idipathi pulmnary brsis is haraterized by peripheral and basal predminant interstitial brsis. In this patient, there is nslidatin, whih is limited t the entral prtin f the right lung. 2. C. Radiatin pneumnitis is usually apparent n the hest radigraph by 6 t 8 weeks fllwing mpletin f therapy. On asin, radiatin pneumnitis an have a rapid r delayed nset. Hwever, this is nt the usual time urse. Numerus fatrs suh as radiatin dse, nmitant hemtherapy, and hst fatrs ntribute t radiatin indued lung injury. Cmputed tmgraphy (CT) an shw shw abnrmalities befre they beme apparent n hest radigraphy. 3. A. Althugh different tumr types an alter the radiatin treatment plan fr a patient, the tumr histlgy itself des nt diretly ntribute t the patient’s risk fr develping
162
SECTION IV
Answers
radiatin-indued lung injury. Sme hemtherapeuti agents suh as gemitabine an predispse a patient t develping radiatin-indued lung disease. Higher radiatin dses inrease the risk f lung injury. Larger tumrs require targeting therapy t a larger vlume f tissue, thus inreasing a patient’s risk f develping radiatin-indued lung disease. 4. D. Radiatin brsis usually develps 6 t 12 mnths after the mpletin f therapy. Radiatin pneumnitis is nt a prerequisite fr develping radiatin brsis, and patients with radiatin pneumnitis might nt develp brsis. Patients wh have radiatin pneumnitis and are symptmati an have a nrmal radigraph, while sme patients with abnrmal radigraphs are asymptmati. Radiatin brsis usually stabilizes 12 t 24 mnths after mpletin f therapy, depending n the tehnique(s) used. A prgressing abnrmality n the hest radigraph after this time shuld raise suspiin fr reurrent neplasm r anther press.
Comment Differential Diagnosis Psteranterir (PA) and lateral radigraphs shw dense perihilar nslidatin in the right lung with assiated vlume lss. Nte the sharp lateral and superir margins n the PA radigraph and the nnanatmi distributin n the lateral view where nslidatin rsses the pulmnary ssures. This nnanatmi distributin f dense nslidatin with vlume lss is typial f radiatin indued injury. Infetin and aspiratin uld have a similar appearane, althugh the margins typially are less well dened. Adenarinma uld be nsidered with hrni nslidatin.
Discussion The hest radigraph (Figure 1) shws dense right lung nslidatin with sharp lateral and superir margins with a nnanatmi distributin, spanning the right lung in the sagittal plane. The gegraphi margins and gemetri shape rrespnd t the eld f irradiatin. Radiatin pneumnitis and brsis an ur in patients wh have reeived denitive radiatin therapy fr lung aner. Radiatin pneumnitis is generally bserved n hest radigraphs within 6 t 8 weeks fllwing mpletin f treatment, but CT might detet subtle abnrmalities earlier than radigraphy (within a few weeks after mpletin f treatment). Suh paities are harateristially sharply demarated and are nt limited by anatmi bundaries suh as ssures. Fibrsis usually develps within 6 t 12 mnths fllwing radiatin therapy. With time, radiatin brsis an ensue. The parenhymal paities generally beme mre linear in nguratin and are usually ampanied by vlume lss and tratin brnhietasis. Several methds have been develped t deliver an adequate dse f radiatin t tumrs while limiting the amunt f expsure t nrmal lung parenhyma. These tehniques inlude limited radiatin prtals, tangential beams, nfrmed therapy, intensity-mdulated radiatin therapy. These methds an result in variable patterns f radiatin-indued lung injury and steretati bdy radiatin therapy (SBRT). Knwledge f the tempral relatinship and type f therapy an help t distinguish radiatin hanges frm infetin and malignany. Flurdexygluse psitrn emissin tmgraphy (FDG PET)/CT an be helpful fr distinguishing radiatin-indued lung disease frm malignany when perfrmed at least 6 t 12 mnths after mpletin f radiatin therapy. Radiatin pneumnitis an be treated expetantly, r patients may be given rtisterids. Severe pneumnitis an require supplementary xygen r mehanial ventilatin.
REFERENCES
Chi YW, Munden RF, Erasmus JJ, etal. Effets f radiatin therapy n the lung: radilgi appearanes and differential diagnsis. Radiographics. 2004;24:985–997. Thrai Radilgy: The Requisites, 3rd ed, 405–427.
CASE 26 Acute Respiratory Distress Syndrome With Barotrauma 1. A, B, and D. ARDS is a frm f aute hypxi respiratry failure with myriad auses inluding infetin, trauma, inhalatinal injury, drug txiity, aspiratin, and llagenvasular disease. The hest radigraph is always abnrmal and usually shws extensive lung paiatin with air brnhgrams. Saridsis mst mmnly presents as symmetri mediastinal and hilar lymphadenpathy with r withut lung invlvement, whih inludes bilateral perilymphati ndules and less mmnly large paities. 2. A and D. The hest radigraph (Fig. 26.1) shws a luent gas lletin in the right hemithrax separating the viseral and parietal pleura laterally. There is als a pneummediastinum. There is air streaking int the next superimpsed n the spinus presses f the upper vertebral bdies. The mputed tmgraphy (CT) (Supplemental Fig. 26.1) shws air in the anterir mediastinum. Pneummediastinum typially nsists f streaks f air in the mediastinum n the frntal radigraph, with gas smetimes utlining strutures suh as the arta. Subutaneus gas is ften a sendary sign f pneummediastinum. Pneumperiardium is muh less mmn than pneummediastinum exept fr patients wh had reent heart surgery. Pneumperiardium usually surrunds ne r bth ventriles as a thin band f gas. N etpi subdiaphragmati gas is evident n this hest radigraph. 3. B. With evlutin f ARDS, rganizatin and healing, ften with brsis, ensue. The lungs beme stiff, and bartrauma an result frm higher pressure ventilatin, whih is required t maintain xygenatin. Pneumthrax r pneummediastinum an develp. Central venus atheter plaement an als ause pneumthrax, althugh inreased used f ultrasund guidane has redued this mpliatin. Pulmnary laeratin an ause pneumthrax, but there are nt ther signs f traumati injury t the hest in this patient. 4. D. Pleural effusin is nt a requirement fr the diagnsis f ARDS. By denitin, Pa2/Fi2 ≤300 is required fr ARDS, and the severity is assessed by the degree f impaired xygenatin. Bilateral lung paity n hest radigraphy is a riterin fr the diagnsis f ARDS. Hypxemi respiratry failure must develp within ne week f an insult.
Comment Differential Diagnosis The AP prtable radigraph shws supprt tubes in expeted latins, diffuse lung paity, and small right pneumthrax as well as pneummediastinum. The differential diagnsis f diffuse lung paity in an autely hypxemi patient is usually limited t ardigeni r nn-ardigeni edema, diffuse alvelar hemrrhage, r infetin; and distintin amng these auses is ften difult n imaging alne.
Discussion ARDS is a severe frm f aute lung injury that is thught t enmpass a variety f distint disrders that share mmn
Opening Round
pathphysilgi and linial features. The denitin f ARDS was updated in 2012 (the Berlin Denitin) t better reet understanding f ARDS and pitfalls in diagnsis. A variety f pulmnary and extrapulmnary nditins, an preipitate ARDS inluding infetin, trauma, inhalatinal injury, drug txiity, aspiratin, and llagen-vasular disease. The linial denitin f ARDS inludes Pao2/Fio2 ≤300, bilateral lung paity n the hest radigraph, aute nset, and exlusin f hydrstati lung edema. Beause f dereased lung mpliane and the need fr prlnged mehanial ventilatin, patients with ARDS an develp bartrauma, inluding subutaneus emphysema, pneumthrax, pneummediastinum, and pulmnary interstitial emphysema. Treatment f ARDS inludes ventilatry supprt and treatment f the underlying ause. The mrtality rate is high, apprximately 50%. Survivrs an have residual pulmnary deits. REFERENCES
ARDS Denitin Task Fre. Aute respiratry distress syndrme: the Berlin Denitin. JAMA. 2012 Jun 20;307(23):2526–2533. Thrai Radilgy: The Requisites, 3rd ed, 226–237.
CASE 27 Cavity From Postprimary Tuberculosis 1. A, B, and D. Lung aner an avitate, usually as the result f nersis. Granulmatsis with plyangiitis (frmerly Wegener’s granulmatsis) an present as a slitary avitary mass lesin with an irregular, thikened wall, althugh multiple lesins are mre mmn. Pstprimary tuberulsis (reativatin r reinfetin) ften manifests as avitary disease. Hwever, the status f the patient’s immune system determines the presentatin f tuberulsis, whether it is primary infetin, reativatin, r reinfetin. Cavities an frm in saridsis but are typially the sequelae f brsis and brnhietasis. Unilateral upper lbe disease in saridsis wuld be highly unusual. 2. C. Squamus ell arinmas have the highest assiatin with avitatin, espeially when they are large. Adenarinmas, small ell lung arinmas, and large ell arinmas an avitate, albeit less frequently than with squamus ell arinmas. 3. A. Wall thikness is nt spei fr the nature f a avitary mass lesin, but wall thikness 5 m (>50% nrmal diameter).Measurements may be made n axial mputed
Fig. 51.1C
Fair Game
are f vasular etilgy. When nrmal artinturs are indistinguishable frm a mediastinal mass, the silhuette sign may assist in lalizing the lesin t the arta. Other radigraphi lues might inlude nvexity f the right superir mediastinum, the hilum verlay sign, traheal deviatin, and lateralizatin f the left paraarti interfae. Peripheral aliatin may be a lue t the presene f an atherslerti aneurysm. Cntrast-enhaned CT and magneti resnane imaging (MRI) play imprtant rles in the diagnsis and surveillane farti aneurysms (Fig. 51.1C). REFERENCES
Cmputed tmgraphy pulmnary angigraphy (CTPA) has beme the mdality f hie fr diagnsis f PE with sensitivity and speiity reahing 96% and 100%, respetively. Cnversely, in patients with lw pre-test prbability fr PE, CTPA may prdue false-psitive rates as high as 42%. Findings f PE inlude mplete lusin r partial lling defets within pulmnary arteries. Oasinally, PE an be deteted as a hyperattenuating lling defet n nn-ntrast mputed tmgraphy (CT). Dual-energy CT maps idine perfusin within nrmal lung tissue, analgus t sintigraphy, and may allw visualizatin f perfusin defets distal t small embli.
Meyershn NM, etal. Rle f mputed CT in the assessment f the thrai arta. Curr Treat Options Cardio Med. 2015;17:35. Thrai Radilgy: The Requisites, 3rd ed, 97–136.
CASE 52 Pulmonary Infarct 1. E 2. B 3. C
Comment Differential Diagnosis Tumr thrmbus, primary pulmnary artery sarma, vasulitis, artifat (respiratry mtin, w, streak), mimis (pulmnary vein, muid impatin).
Discussion Pulmnaryemblus (PE) is a mmn linial prblem requiring prmpt diagnsis and management, as untreated PE is assiated with up t 30% mrtality. Symptms f aute PE are nn-spei and inlude dyspnea and pleuriti hest pain, as are the linial signs f tahypnea, tahyardia, and hemptysis. Clinial deisin supprt and validated linial sring systems (Wells, simplied Wells, and mdied Geneva sres) an assist the liniian in direting apprpriate wrk-up and management. If PE is suspeted in a patient with lw pretest prbability and a negative D-dimer test, an alternate diagnsis ther than PE shuld be nsidered. If the D-dimer test is psitive, then imaging is the next step in diagnsti wrkup. Chest radigraphs are ften the rst imaging perfrmed but may shw n abnrmality. Signs f PE n radigraphy inlude ateletasis, Westermark sign (parenhymal lueny beynd the luded vessel), Fleishner sign (an enlarged entral pulmnary artery), and “knukle sign” (tapered distal vessels due t entral thrmbus.) Pulmnary infartin may manifest as peripheral wedge-shaped nslidatin in the lwer lateral aspet f the invlved hemithrax that abuts the pleura (Hamptn hump) (Fig 52.1), r as ateletasis, an elevated hemidiaphragm, r pleural effusin. Chest radigraphy is primarily used t exlude ther diagnses that might mimi a PE, suh as pneumthrax, pneumnia, r rib frature. A reent hest radigraph is required t interpret any subsequent ventilatin-perfusin sintigraphy san. Ventilatin sintigraphy utilizes inhaled raditraers xenn 133 r tehnetium-99m (99mT) diethylenetriamine-pentaaeti aid whih are distributed thrugh the lungs. 99mT maraggregated human albumin is injeted intravenusly t map the lung perfusin. Diagnsis f PE is made when tw r mre segmental r larger wedge-shaped perfusin defets are fund in areas f nrmal ventilatin (V/Q mismath).
179
Fig. 52.2
Fig. 52.3
180
SECTION IV
Answers
On CT, pulmnary infarts typially manifest as peripheral, ften wedge-shaped nslidatin that extends t the pleural surfae (Figs. 52.23 and 52.3). Grund-glass paity and bubble-like luenies may be seen—representing hemrrhage and infarted tissue—and beme mre ndular as they reslve. CTPA may identify patients with hemdynami mprmise wh are at inreased risk fr fatal r nnfatal adverse events. Imaging signs f right heart strain (right ventrile–left ventrile rati f >1.0), straightening r leftward bwing f the interventriular septum, ntrast reux int the hepati veins, and dilated mainpulmnaryartery. CTPA an als detet alternative diagnses suh as pneumnia, pneumthrax, periarditis, and arti dissetin. Pitfalls in CTPA interpretatin inlude respiratry mtin, image nise, w-related artifat, and streak artifat frm nentrated ntrast material within the superir vena ava. Mimis f PE inlude slw w in vessels (espeially within areas f nslidatin r ateletasis) and muus-plugged brnhi. Treatment inludes antiagulatin, inferir vena ava lter, systemi thrmblysis, atheter–direted mehanial r pharmalgi thrmblysis, and surgial embletmy. Mstpulmnaryembli reslve withut sequelae. Others may result in inmplete reslutin and hrni thrmbembli disease. REFERENCES
Kirsh J, etal. Amerian Cllege f Radilgy Apprpriateness Criteria ® Aute Chest Pain—Suspeted Pulmnary Emblism. J Am Coll Radiol. 2017;14(5):S2–S12. Kligerman SJ, etal. Radilgist perfrmane in the detetin f pulmnary emblism: features that favr rret interpretatin and risk fatrs fr errrs. J Thorac Imaging. 2018;33(6):350–357. Thrai Radilgy: The Requisites, 3rd ed, 238–258.
CASE 53 Lymphoma 1. E 2. D 3. C
plane as the hila. There may be traheal deviatin r narrwing frm mass effet. CT features typially inlude a midline mass f hmgeneus sft tissue attenuatin. Masses tend t be large and are dened as “bulky” if ver 10 m r if greater than ne-third f the intrathrai diameter is upied, prtending pr prgnsis. If ysti hange and nersis are present, the mass may appear mre hetergeneus. Lymphma tends t exert signiant mass effet n and displae adjaent mediastinal strutures. Invasin f adjaent hest wall r lung parenhyma may ur. Midline latin f the mass favrs lymphma, while an ff-midline latin might suggest an alternate diagnsis suh as thymma. Treated lymphma will derease in size and attenuatin with asinal aliatin. Caliatins are rare in untreated lymphma. Treatment respnse n MRI may manifest as inreased T2 signal in areas f tumr nersis and eventually, lw T2 signal indiating brsis. 18FDG-PET-CT plays an imprtant rle in staging and restaging f lymphma, with mplete respnse nsidered with reslutin f FDG ativity whether there is derease in lesin size. Whle-bdy and PET-MRI may be emplyed fr serial imaging in surveillane fr lymphma with added benet f n inizing radiatin. REFERENCES
Barringtn SF, etal. Rle f imaging in the staging and respnse assessment f lymphma: nsensus f the Internatinal Cnferene n Malignant Lymphmas Imaging Wrking Grup. J Clin Oncol. 2014;32(27):3048–3058. Shah KJ, et al. Diffuse large B-ell lymphma in the era f preisin nlgy: hw imaging is helpful. Korean J Radiol. 2017;18(1):54–70. Thrai Radilgy: The Requisites, 3rd ed., p. 113.
CASE 54 Traumatic Aortic Injury (Transection) 1. A, B, and E 2. D 3. E
Comment Differential Diagnosis
Comment
Mediastinal Hdgkin lymphma, thymi epithelial neplasm, mediastinal germ ell neplasm, mediastinal giter, lymph nde metastasis.
Differential Diagnosis
Discussion Primarymediastinal lymphma (PML) arises frm the thymus r lymph ndes, typially manifesting as a prevasular r (less mmnly) viseral mediastinal mass. Classiatin f PML is divided int Hdgkin (HL) r nn-Hdgkin (NHL). HL is dened by the presene f Reed-Sternberg ells and inludes lassial HL and ndular lymphyte predminant subtypes. Subtypes f primary mediastinal NHL inlude diffuse large B-ell lymphma, lymphblasti lymphma, and primary thymi large B-ell lymphma. Clinial symptms are nn-spei inluding ugh r tahypnea. Oasinally, mass effet might ause dysphagia r superir vena ava (SVC) syndrme. Fever and weight lss an als be present. Radigraphi features f lymphma mst mmnly inlude mediastinal wideningn the frntal prjetin and retrsternal lear spae lling n the lateral prjetin. A hilum overlay sign may be seen as preservatin f hilar vasulature interfaes, indiating the mass is nt in the same anterpsterir
Dutus bump, arti spindle, brnhial artery infundibulum, arti pseudaneurysm, penetrating atherslerti uler, pseudartatin.
Discussion Prmpt exlusin f traumati arti injury (TAI) after blunt hest trauma is ritial as suh injuries have a greater than 70% in-eld mrtality and up t 50% 24-hur mrtality if left untreated. TAI urs mst mmnly at sites where the arta is relatively xed, namely the rt, isthmus, and desending arta at the level f the diaphragmati hiatus. Patients wh survive lng enugh t reeive hspital are typially have injuries at the arti isthmus, just distal t the left sublavian artery rigin, near the ligamentum arterisum. Prpsed injury mehanisms inlude (1) rapid differential deeleratin f the xed prtins f the arta relative t ther mediastinal strutures, (2) shear stress n the wall, (3) sseus pinh sendary t psterir translatin f the sternum, and (4) hydrstati r water-hammer phenmenn aused by sudden inreases in intrathrai pressure resulting in tears. Rt injuries are assiated with ther serius ardivasular
Fair Game
181
esphagus, r nasgastri tube; and a left apial ap, indiating hemrrhage extending int the apial extrapleural spae. Cmputed tmgraphy angigraphy (CTA) is the mst apprpriate study fr evaluatin f TAI, yielding rapid, high-reslutin depitin f the injury and assiated ndings with a sensitivity and speiity f mre than 90% fr detetin f TAI. Diret CT signs f TAI inlude arti ntur abnrmality, intimal ap, intraluminal thrmbus, pseudaneurysm, and frank ntrast extravasatin. Preise CT haraterizatin f the lesin is imprtant fr treatment planning (stent versus surgery) and shuld inlude infrmatin related t the length f defet, arti diameter abve and belw, and distane frm the great vessel rigins. Detetin f a periarti hematma is an indiret sign f TAI, demnstrating bld in ntat with the arta (Fig.54.2). Mediastinal bld with a preserved fat plane arund the arta suggests a different sure f bleeding – frm mediastinal veins r interstal r internal mammary arteries. Fratures f the rst r send ribs shuld prmpt lse examinatin f the arta and arh vessels t exlude traumati vasular injury. Treatment f TAI typially inludes surgial r endvasular graft repair (Fig. 54.3). REFERENCES
Fig. 54.2
Steenburg SD, etal. Aute traumati arti injury: imaging evaluatin and management. Radiology. 2008;248:748–762. Cullen EL, et al. Traumati arti injury: CT ndings, mimis, and therapeuti ptins. Cardiovasc Diagn Ther. 2014;4(3):238–244. Thrai Radilgy: The Requisites, 3rd ed, 279–288.
CASE 55 Lipoid Pneumonia 1. A, B, C, D, E 2. C 3. D
Comment Differential Diagnosis Lung adenarinma (muinus), lymphma, alvelar prteinsis, alvelar sarid, hrni esinphili pneumnia, rganizing pneumnia.
Discussion
Fig. 54.3
mpliatins, inluding rnary arterial injury, hemperiardium, and tampnade. Supine hest radigraphy is the rst imaging mdality used in the trauma setting, mainly t exlude aute life-threatening nditins like tensin pneumthrax (Fig. 54.1). Several lues may assist in the diagnsis f TAI n hest radigraphy. These inlude widening f the vasular pedile (>8 m r 25% f the width f the thrax); lss f nrmal arti arh and desending arti nturs; rightward displaement f the trahea,
Lipid pneumnia (LP) is an unmmn ause f hrni lung paiatin n hest radigraphy and mputed tmgraphy (CT). The etilgies inlude exgenus and endgenus auses f lipid aumulatin in alveli. Exgenus sures inlude aspiratin f animal fats, mineral il, r vegetable ils. Endgenus LP results frm phagytsis f lipids by intraalvelar marphages that ur sendary t seretins that may aumulate beynd a pint f endbrnhial bstrutin, fat strage diseases, r alvelar prteinsis. Exgenus LP may have an aute nset if a large quantity f petrleum-based prdut is intrdued t the lung, resulting in an aute pneumnitis. Chrni LP typially fllws frm repeated, lng-standing aspiratin f animal fats r ther ils and manifests similarly t aute LP. Predispsing nditins t hrni LP inlude lder age, intelletual develpmental disrders, rpharyngeal dysphagia, gastresphageal reux, r ther fatrs whih may ause aspiratin (e.g., Zenker divertiulum). Chrni use f mineral il laxative is als lassially assiated. Radigraphi manifestatins vary arding t the ause, with aute exgenus lipid pneumnia prduing grund-glass and nslidative paities ften in a lbar and basilar distributin. Chrni LP may manifest as a nn-reslving nslidatin
182
SECTION IV
Answers
n hest radigraphy. On CT, LP is haraterized by the demnstratin f lw attenuatin within fal r multifal areas f parenhymal nslidatin equal t that f marspi fat (-30 HU) (Figs. 55.1 and 55.2). The areas f nslidatin are ften irregular r spiulated, frming ndular r mass-like paities that may mimi lung aner. CT detetin f fat attenuatin within a ndule r mass must be differentiated frm ther fat-ntaining lung lesins (e.g., hamartma, fat-ntaining metastases [sarmas].) Chrni LP may be assiated with arhitetural distrtin r ther evidene f brsis due t hrni inammatin. Pneumateles may develp in areas f previus pneumnitis and beme mpliated by pneummediastinum r pneumthrax if they rupture. A “razy-paving” pattern may als be seen n CT with nn-reslving grund-glass paity with superimpsed interlbular septal thikening. Lipid pneumnia may be avid n urdexygluse (FDG)-psitrn emissin tmgraphy (FDG PET) if there is assiated inammatin. Chrni airspae paities must be differentiated frm ther auses suh as pulmnary infetin, alvelar prteinsis, lung aner, and lymphma.
and ateletasis. Fifty perent f patients with PCD will have situs abnrmalities (hetertaxy r situs inversus) (Fig. 56.3). REFERENCES
Knwles MR, et al. Primary iliary dyskinesia. Clin Chest Med. 2016;37(3):449–461. Thrai Radilgy: The Requisites, 3rd ed, 137–158, 103–209.
CASE 57 Invasive Aspergillus 1. 2. 3. 4.
A, B, and C C C D
Comment Differential Diagnosis
REFERENCES
Betanurt SL, etal. Lipid pneumnia: spetrum f linial and radilgi manifestatins. AJR Am J Roentgenol. 2010;194:103–109. Marhiri E, etal. Exgenus lipid pneumnia. Clinial and radilgial manifestatins. Respiratory Medicine. 2011;105:659–666. Thrai Radilgy: The Requisites, 3rd ed, 355–376, 405–427.
CASE 56 Kartagener Syndrome (Primary Ciliary Dyskinesia) 1. B 2. A, C, D 3. B
Comment Differential Diagnosis Cysti brsis, allergi brnhpulmnary aspergillsis, pst-infetius brnhietasis, mmn variable immune deieny, human immundeieny virus (HIV).
Discussion Kartagener syndrme is a subset f the primary iliary dyskinesia (PCD) spetrum f ilipathies haraterized by a lassi triad f brnhietasis, situs inversus, and sinusitis. Patients with PCD inherit a geneti defet that leads t impaired iliary struture and funtin in an autsmal reessive pattern. Early symptms f brnhitis, sinusitis, titis media, and rhinitis ften lead t early hildhd presentatin, whih may be mpliated by reurrent pneumnia. Patients may present later in life with redued fertility beause f impaired spermatzn iliary funtin. Brnhietasis assiated with this disrder is typially less severe than that f ysti brsis. Imaging features inlude situs inversus and brnhietasis, with r withut superimpsed infetin (Fig. 56.1). Brnhietasis in this nditin typially affets the right middle and lwer lbes. Centrilbular ndules and tree-in-bud paities indiate the small airways affeted by muus and infetin. Msai attenuatin and air-trapping may be seen. Maxillfaial mputed tmgraphy (CT) will shw sequelae f aute and hrni sinusitis inluding musal thikening and muperisteal reatin, with r withut superimpsed air-uid levels (Fig. 56.2). Other assiated ndings inlude brnhial wall thikening, lung hyperinatin,
Other pneumnia (baterial, mybaterial, r fungal), granulmatsis with plyangiitis, pulmnary infartin, lung aner.
Discussion Invasive pulmnary aspergillsis is the mst mmn fungal infetin t affet immunmprmised patients, usually thse with severe neutrpenia. Ppulatins at risk fr this ptentially fatal infetin inlude transplant reipients, hematlgi malignanies, r thse reeiving hrni highdse sterids. Radigraphi manifestatins f invasive aspergillus inlude slitary r multiple ill-dened pulmnary ndules r masses, and nuent nslidatin (Fig. 57.1). When Aspergillus fumigatus rganisms invade pulmnary bld vessels (angiinvasive frm), lalized small t medium sized pulmnary arteries beme luded leading t peripheral infartin. CT ndings f pulmnary infartin inlude peripheral wedge-shaped subpleural nslidatin with surrunding grund-glass paity (CT halo sign) in whih the hal represents hemrrhage (Fig. 57.2). This sign is nt spei fr Aspergillus but is highly suggestive in the apprpriate linial setting. Other invasive fungal rganisms suh as Mucor and Candida speies may als manifest with hals n CT. Early regnitin f the hal sign may prmpt the additin f anti-fungal treatment, thereby reduing mrbidity and mrtality. After nset f therapy, areas f nslidatin may avitate and frm the air crescent sign, indiating revery f neutrphil funtin and respnse t treatment. REFERENCES
Walsh S, et al. Imprtane f the reversed hal sign fr the diagnsis f angiinvasive pulmnary aspergillsis. Respir Med. 2014;108(8):1240. Prasad A, etal. Pulmnary aspergillsis: what CT an ffer befre it is t late! J Clin Diagn Res. 2016;10(4):TE01–TE05. Thrai Radilgy: The Requisites, 3rd ed, 155.
CASE 58 Granulomatosis With Polyangiitis 1. 2. 3. 4.
A, B, C, D, and E E C D
Fair Game
Comment Differential Diagnosis Septi embli, lung absesses, tuberulsis, trahebrnhial papillmatsis, pulmnary metastases.
Discussion Antineutrphil ytplasm antibdy (ANCA)-assiated vasulitis is the mst mmn f the primary small vessel vasulitides that inlude granulmatsis with plyangiitis (GPA) (frmerly Wegener granulmatsis), esinphili granulmatsis with plyangiitis (E-GPA), and mirspi plyangiitis. GPA represents a multisystem nertizing vasulitis resulting in the lassi linial triad f upper airway disease with lung and renal invlvement (glmerulnephritis). The upper respiratry trat is invlved in almst all patients and may inlude symptms and signs f titis, sinusitis, rhinrrhea, epistaxis, r airway stensis. Lungs and kidneys are invlved in 90% and 80% f patients, respetively. The presene f ytplasmi ANCA (ANCA) suggests the diagnsis. Treatment typially inludes ylphsphamide and sterids. Chest radigraphi and CT ndings assiated with GPA inlude multiple pulmnary ndules and masses, seen in up t 90% f patients with pulmnary disease (Fig. 58.1). The distributin f ndules and masses is usually bilateral, lwer lung zne, and subpleural. Ndules may alese int larger masses >10 m in diameter, with predispsitin t avitatin when greater than 2 m, frming thik-walled irregular avities and dereasing in size with treatment (Fig. 58.2). On ntrastenhaned CT, mst ndules r masses demnstrate entral lw attenuatin, indiating nersis. Less mmnly grund-glass paity may surrund the ndule (CT halo sign). Other CT ndings inlude reverse CT halo and feeding vessel signs, the latter representing an artery, leading int a ndule r mass. Cnslidatin and grund-glass paities are the send mst mmn manifestatin after ndules and masses, reeting pneumnitis r alvelar hemrrhage related t vasulitis. Airway abnrmalities inlude traheal and brnhial wall thikening. Cnentri inammatry wall thikening an result in airway stensis, with subgltti traheal invlvement being mst typial. The differential diagnsis fr subpleural ndules and masses f GPA inludes septi embli and absess, neplasms (inluding hematgenus metastases and lymphma), rganizing pneumnia, and Kapsi sarma. REFERENCES
Feragalli B, etal. The lung in systemi vasulitis: radilgial patterns and differential diagnsis. Br J Radiol. 2016;89(1061):20150992. Thrai Radilgy: The Requisites, 3rd ed, 238–258.
CASE 59 Lymphangioleiomyomatosis 1. A-C. The CT images shw diffuse ysti lung disease thrughut the lungs. Of the listed hies, the nly hie whih is nt a knwn ause f diffuse ysti lung disease is Hermansky-Pudlak syndrme, whih is an inherited ause f pulmnary brsis and ulutaneus albinism, mre mmn amng peple frm Puert Ri. 2. B. The ysts in Langerhans ell histiytsis (LCH) are usually upper lung prepnderant with sparing f the stphreni angles. Cysts an have thiker walls than in ther ysti lung diseases and may alese t frm bizarre shapes. The ysts ften arise frm entrilbular ndules whih
183
prdue small airway bstrutin. Cmbined with a histry f smking, these ndings are essentially diagnsti f LCH. 3. A. Lymphangileimymatsis (LAM) may ause pleural effusins whih are hylus (hylthrax) in up t 1/3 f patients. The mst mmn auses f hylthrax are trauma t the thrai dut and lymphma. It wuld be unusual fr the ther listed nditins t lead t hylthrax. 4. C. Hepatellular arinma is nt assiated with tuberus slersis mplex (TSC). Cmmn manifestatins f TSC inlude a LAM-like ysti pattern f lung disease, rhabdmymas in the heart, renal angimylipmas, a myriad f skin lesins, brain lesins, slerti bne lesins and multindular multifal pneumyte hyperplasia. The latter entity prdues multiple small pulmnary ndules.
Comment LAM is haraterized by abnrmal prliferatin f immature smth musle ells. Thin-walled lung ysts in LAM may rupture, leading t spntaneus pneumthraes. Lymphati bstrutin may result in hylus pleural effusins. Vasular endthelial grwth fatr-D (VEGF-D) levels are ften elevated in LAM but nt in ther ysti lung diseases and an be used t supprt the diagnsis and t mnitr prgressin f disease. Reently, sirlimus has been shwn t stabilize lung funtin in LAM. Reent evidene als suggests that sirlimus may be beneial in the treatment f ther manifestatins f TSC. The differential diagnsis fr diffuse ysti lung disease is nt as brad as in ther diffuse lung diseases. Mst ases are either due t TSC, LAM, r LCH. In mst ases, the CT imaging manifestatins f LAM and TSC in the lungs are indistinguishable, althugh multindular multifal pneumyte hyperplasia (MMPH) urs almst exlusively in TSC with r withut manifestatins f LAM. The features f MMPH n CT nsist f multiple small ndular lesins usually 5 mm r less in diameter whih an be slid r subslid in attenuatin. Obviusly, if ther nn-pulmnary manifestatins f TSC are evident, the diagnsis an be made readily. LAM and LCH are usually quite different in their imaging appearane. Cysts in LAM are unifrmly thin-walled and evenly spaed thrughut the lungs. In LCH, the ysts may be asymmetri r bizarrely shaped beause ysts may alese ver time. Small ndules may be present in LCH whih wuld be very unusual in LAM. In LCH, the diffuse lung disease lassially spares the stphreni angles. Regarding demgraphis, all LAM patients (and vast majrity with TSC) are wmen, usually f hild-bearing age. LCH als tends t affet yung adults wh are smkers but is nt limited t wmen. Less mmn auses f diffuse ysti lung disease inlude lymphyti interstitial pneumnitis (LIP), Birt-Hgg-Dube syndrme (BHD), and amylidsis. Helpful diagnsti pearls fr these nditins: LIP: Almst always sendary rather than idipathi. In adults, mst ften assiated with Sjgren syndrme. May als ur in immundeieny syndrmes. BHD: Assiated with renal tumrs (mst ften hrmphbi renal ell arinma) and skin lesins (brflliulmas mst mmn). Autsmal dminant inheritane. Amylidsis: May ur with LIP and Sjgren syndrme. Prdues ndular amylid fi whih may be partially alied/ ssied. REFERENCES
Seaman DM, Meyer CA, Gilman MD, MCrmak FX. Diffuse ysti lung disease at high-reslutin CT. AJR Am J Roentgenol. 2011;Jun;196(6):1305–1311. Thrai Radilgy: The Requisites, 3rd ed, 355–376.
184
SECTION IV
Answers
CASE 60
CASE 61
Tuberculosis
Pneumocystis jirovecii Pneumonia
1. A-C. The mputed tmgraphy (CT) images shw large nerti lymph ndes in the mediastinum.), Tuberulus lymphadenitis, lymphma, and metastati disease (usually frm a squamus ell arinma primary) are the mst mmn auses fr nerti lymphadenpathy. Saridsis is a very mmn ause f large bulky lymph ndes in the mediastinum and hilar regins; hwever, nersis within lymph ndes wuld be unusual. 2. D. Cnslidatin is a mmn nding f tuberulsis in hildren. Fibravitary disease is seen in ative tuberulsis and brnhietasis may ur in ative disease r as late nsequene f remte tuberulsis. Bth are mre frequently identied in adults. Grund-glass paity is nt a majr nding in mst ases f tuberulsis. 3. A. Lymphadenpathy in tuberulsis (TB) is mmn in hildren and thse with human immundeieny virus– aquired immundeieny syndrme (HIV-AIDS) with CD4 unts belw 200/mm^3. 4. A. TB pleural effusins are very unmmn in infants and unmmn in hildren. Up t 15% f adults may have pleural effusins as a manifestatin f TB; this may be the sle and initial manifestatin f TB in sme ases. Therefre, a high level f linial suspiin is neessary. A pleural bipsy is required fr diagnsis.
1. A, B, C, and D. The CT images shws diffuse grundgrund glass paity with superimpsed retiulatin septal thikening and intralbular lines nsistent with a razy paving pattern. All the answer hies an manifest this pattern n hest CT. Crazy paving n hest CT is lassially desribed in pulmnary alvelar prteinsis; hwever, given the relative rarity f this nditin, ther mre mmn disease presses are usually respnsible fr the pattern in mst linial praties. Anther rare ause f the razy paving pattern is hrni lipid pneumnia, whih urs in thse wh are at risk fr aspiratin r inhalatin f lipid-laden substanes (mineral ils, il-based nse drps, lip glss, et). 2. B. Up t 1/3 f HIV patients with Pneumocystis jirovecii pneumnia (PJP) will have assiated air ysts; very ften there is an upper lung prepnderane. The prevalene f ysts in PJP is lwer in nn-HIV afited patients. Interestingly, ysts may reslve with treatment. As in ther ysti lung diseases, these patients are at risk fr spntaneus pneumthraes. Signiant pleural effusins r lymphadenpathy is unusual in PJP. In the setting f PJP, tree-in-bud ndules n hest CT suggest superimpsed infetin in the small airways frm anther mirbial agent r frm aspiratin. 3. A. Lymphangileimymatsis (LAM) may ause pleural effusins whih are hylus (hylthrax) in up t 1/3 f patients. The mst mmn auses f hylthrax are trauma t the thrai dut (ften pst-surgial) and lymphma. It wuld be unusual fr the ther listed nditins t lead t hylthrax. 4. A. Trimethprim-sulfamethxazle (Batrim) is rst line therapy and prphylaxis fr PJP.
Comment The terms primary and pst-primary TB ntinue t be used in the medial literature. It was previusly believed that primary and pst-primary TB uld be differentiated n linial and imaging grunds. Hwever, DNA ngerprinting evidene has shwn that primary and pst-primary TB annt be differentiated reliably n imaging (i.e., time frm initial infetin t the ineptin f ative disease annt predit the imaging manifestatins f TB). Hwever, the imaging manifestatins f TB in adults, hildren, and HIV-psitive patients differ. Lymph nde enlargement is a harateristi feature f TB in hildren but is muh less mmn in adults. Lymph nde enlargement may ur alne r in assiatin with parenhymal nslidatin, thugh islated nslidatin in hildren is unusual. On ntrast-enhaned CT sans f patients with mediastinal and hilar tuberulus lymphadenitis, enlarged ndes ften demnstrate a lw-attenuatin enter and peripheral rim enhanement. Histlgially, suh ndes have been shwn t demnstrate entral nersis and a highly vasular, inammatry apsular reatin. Althugh lw-attenuatin ndes are harateristi f TB, they are nt spei fr this entity. Suh ndes may als be enuntered in atypial mybaterial and fungal infetins. Neplasti lymph ndes (e.g., metastati squamus ell arinma and lymphma) may als demnstrate this appearane. Regarding TB in HIV-psitive patients, the radigraphi appearane varies depending n the patient’s CD4 unt. In patients with CD4 unts abve 200/mm3, a pst-primary pattern is typially seen. In patients with CD4 unts belw 200/ mm3, yu will usually bserve a primary pattern, inluding lw-attenuatin lymph ndes and nslidatin. REFERENCES
Jeng YJ, Lee KS. Pulmnary tuberulsis: up-t-date imaging and management. AJR Am J Roentgenol. 2008 Sep;191(3):834–844. Thrai Radilgy: The Requisites, 3rd ed, 323–334
Comment The inidene f PJP has dereased as highly ative antiretrviral therapy (HAARTT) and hemprphylaxis has beme standard treatment in HIV-psitive patients. Hwever, PJP ntinues t be the mst mmn pprtunisti infetin in peple with AIDS. In additin t HIV-psitive patients, PJP may als affet ther immunmprmised individuals (in the setting f stem ell r slid transplant, hemtherapy fr malignany, hematlgial malignany, r hrni rtisterid therapy). The lassi hest radigraphi presentatin f PJP nsists f bilateral perihilar r diffuse symmetri paities, whih may be nely granular, retiular, r grund glass in appearane. Imprtantly, the hest radigraph may be nrmal at the time f presentatin in a signiant minrity f ases f PJP. CT, partiularly HRCT, is mre sensitive than hest radigraphs fr deteting PJP and thus may be helpful in evaluating symptmati patients with nrmal r equival radigraphi ndings. The lassi CT nding in PJP is extensive grund-glass attenuatin, whih rrespnds t the presene f intraalvelar exudate, nsisting f uid, rganisms, and debris. It is ften distributed in a pathy r gegraphi fashin, with a prediletin fr the entral, perihilar regins f the lungs. Grund-glass attenuatin is asinally ampanied by thikened septal lines, and fi f nslidatin may als be evident in severe ases. In up t a third f ases f PJP, grund-glass paities are ampanied by ysti lung disease. Suh ysts have an upper lbe predminane and demnstrate varying sizes and wall thiknesses. REFERENCES
Kanne JP, Yandw DR, Meyer CA. Pneumystis jirvei pneumnia: high-reslutin CT ndings in patients with and withut HIV infetin. AJR Am J Roentgenol. 2012 Jun;198(6):W555–W561. Thrai Radilgy: The Requisites, 3rd ed, 310–322.
Fair Game
CASE 62 Metastatic Osteosarcoma 1. A, B, C, and D. There are multiple bilateral alied mediastinal and hilar lymph ndes. All the listed nditins an ause alied mediastinal and hilar lymph ndes and wuld be viable diagnsti nsideratins althugh silisis wuld be unusual in suh a yung patient. 2. D. Lymphma frequently alies after hemtherapy and radiatin therapy. Caliatin within lymph ndes affeted by lymphma befre therapy is highly unusual. 3. D. Of the listed nditins, histplasmsis is the least likely t ause eggshell aliatin. 4. A. Garland’s triad (als knwn as the 1-2-3 sign and desribed n standard radigraphy) is dened as lymphadenpathy lalized t the right paratraheal and bilateral hilar regins. This symmetri pattern f lymphadenpathy is mst ften assiated with saridsis.
Comment Axial maximum intensity prjetin image frm hest mputed tmgraphy (CT) shws multiple hyperdense mediastinal and hilar lymph ndes as well as sattered hyperdense pulmnary ndules within the lungs nsistent with ssied metastati disease in the setting f stesarma. Calied lymph ndes are usually benign, and they are ften related t granulmatus presses, suh as tuberulsis (TB), histplasmsis, r saridsis. Neplasti auses f alied lymph ndes are less mmn. They inlude metastases frm muinus adenarinmas and lymphma. Regarding lymphma, aliatin is frequently seen fllwing radiatin therapy, but it is rarely enuntered in untreated ases. Ossied lymph ndes are a rare manifestatin f metastati stesarma. Suh ndes appear like alied lymph ndes. In patients with stesarma, the presene f lymph nde metastases indiates a pr prgnsis. Lymphati invlvement is usually ampanied by metastases within the lung whih is the mst mmn site f metastati disease. Lung metastases frequently demnstrate ssiatin. REFERENCES
Se JB, Im JG, G JM, Chung MJ, Kim MY. Atypial pulmnary metastases: spetrum f radilgi ndings. Radiographics. 2001 Mar–Apr;21(2):403–417. Thrai Radilgy: The Requisites, 3rd ed, 428–440.
CASE 63 Septic Infarcts 1. A, B, and C. The mputed tmgraphy (CT) images shw bilateral avitary ndules with peripheral prepnderane and pleural effusins. The differential diagnsis fr avitary ndules inludes metastati disease, septi infarts, multifal nerti pneumnia, and small vessel vasulitis. Cavitary ndules are nt a feature f adult respiratry distress syndrme; mrever, pleural effusins are als unmmn in aute respiratry distress syndrme (ARDS) unless there is superimpsed heart failure. 2. B. In a patient with hrni avitary ndules, subutaneus ndular lesins, and jint pain, rheumatid arthritis is the mst likely etilgy. Althugh there are many radilgi manifestatins f tuberulsis in the thrax, this nstellatin f ndings is nt suggestive f that diagnsis. Granulmatsis with plyangiitis (Wegner granulmatsis) is nt
185
typially assiated with jint pain r subutaneus ndules. Invasive aspergillsis is an aute nditin in neutrpeni patients. 3. C. Indwelling entral venus atheters, prstheti devies, triuspid endarditis (ften frm intravenus drug use), skin infetin, and peridntal disease are all mmn auses f septi infarts/embli. 4. A. Diffuse grund-glass paity with mild superimpsed ndularity are typial ndings in fat emblism. Risk fatrs fr fat emblism inlude pelvi and lng bne fratures, hemglbinpathies, burns, and panreatitis. Usually, there is a 1- t 2-day delay in the appearane f abnrmalities in the lungs beause mst f the pulmnary inammatry respnse fllws the breakdwn f fat glbules int free fatty aids.
Comment Axial image frm hest CT shws bilateral peripheral predminant avitary ndules with pleural effusins. Septi infarts mst ften riginate frm right-sided triuspid endarditis r frm infeted thrmbi within systemi veins. Other sures inlude skin infetins, peridntal disease, entral venus atheters, and paemaker wires. On hest radigraphs and CT sans f patients with septi infarts, yu may bserve prly dened ndular paities and areas f wedge-shaped parenhymal paiatin. Suh paities are usually peripheral in latin and abut the pleural surfae. They have a prediletin fr the lwer lbes. Cavitatin is frequently bserved, partiularly n CT sans. A harateristi nding n CT is the identiatin f feeding vessels leading t the ndules (arrws in the send gure) and wedge-shaped parenhymal paities. Thus, the CT nding f avitating ndules with feeding vessels is highly suggestive f septi infarts. REFERENCES
Khashper A, Disepla F, Ksiuk J, Qanadli SD, Mesurlle B. Nnthrmbti pulmnary emblism. AJR Am J Roentgenol. 2012 Feb;198(2):W152–W159. Thrai Radilgy: The Requisites, 3rd ed, 289–309.
CASE 64 Bronchopleural Fistula 1. C. The riginal hest radigraph shws near mplete paiatin f the left hemithrax after left pneumnetmy nsistent with uid lling the left pneumnetmy spae. The fllw-up hest radigraph shws new gas/uid levels within the upper left hemithrax, essentially diagnsti f a brnhpleural stula sendary t a brnhial stump dehisene. 2. B. The shrter brnhial stump after right pneumnetmy as well as the mre tenuus bld supply f the right main brnhus relative t the left main brnhus renders brnhpleural stulas fllwing pneumnetmy mre mmn n the right side. 3. A. Right middle lbe trsin after right upper lbetmy is the mst mmn type f lbar trsin. This surgial emergeny may result in hemrrhagi infartin r nersis f the invlved lbe due t entral kinking f vessels and lymphatis. 4. D. The next step in management f a suspeted brnhpleural stula wuld be brnhspy t nrm its presene. In additin, nulear mediine xenn sans uld als be pursued t nrm the diagnsis and pinpint the latin f the stula.
186
SECTION IV
Answers
Comment Brnhpleural stula is a relatively unmmn but serius mpliatin fllwing pneumnetmy, with a prevalene f up t 5% and a mrtality rate f apprximately 20%. Majr predispsing fatrs relate t perative auses f brnhial ishemia, suh as a lng brnhial stump, ligatin f the brnhial arteries t prximally, and disruptin f brnhial bld supply frm extensive lymph nde dissetin. Additinal risk fatrs inlude preperative radiatin therapy, sterid therapy, malnutritin, and resetin thrugh infeted r anerus tissue. Fllwing pneumnetmy, the mediastinum is nrmally shifted tward the side f resetin, and the pneumnetmy spae gradually lls with uid ver time. Brnhpleural stula shuld be nsidered when any f the fllwing are bserved: (1) the pneumnetmy spae fails t ll with uid; (2) there is an abrupt dwnward shift in the air-uid level in the pneumnetmy spae identied n an upright radigraph; (3) there is a new lletin f air in a previusly paied pneumnetmy spae; r (4) there is ntralateral shift f the mediastinum assiated with any f the abve ndings. The diagnsis an be nrmed with a xenn ventilatin study, whih will demnstrate xenn ativity in the pneumnetmy spae r brnhspy. REFERENCES
Chae EJ, Se JB, Kim SY, D KH, He JN, Lee JS, Sng KS, Sng JW, Lim TH. Radigraphi and CT ndings f thrai mpliatins after pneumnetmy. Radiographics. 2006 Sep–Ot;26(5):1449–1468. Thrai Radilgy: The Requisites, 3rd ed, 259–278.
CASE 65 Superior Sulcus Tumor 1. B. Squamus ell arinma is the mst mmn ell type f lung aner t present in the superir sulus (apex f the lung) with invasin f mediastinal and hest wall strutures. 2. C. Stage III B r stage IV lung aner patients are nsidered inperable. In the TNM staging system, hest wall invasin is nsidered T3 disease. T3 disease is nt sufient t establish stage III B status. Therefre N3 disease (ntralateral mediastinal/hilar lymphadenpathy r supralaviular/anterir salene lymphadenpathy) wuld be the nly setting in whih the patient wuld be nnperable. 3. B. Invasin f the mediastinum, heart, great vessels, trahea, reurrent laryngeal nerve, esphagus, vertebral bdy, r arina and separate tumr ndule(s) in anther ipsilateral pulmnary lbe is nsidered T4 disease and nnperable regardless f ther N and M ndings. 4. C. Standard therapy fr superir sulus tumrs inlude hemtherapy and raditherapy (CRT) with subsequent surgery 3 t 5 weeks after mpletin f CRT.
Comment A lung aner arising at the extreme apex f the lung is referred t as a superir sulus tumr r Panast tumr. Affeted patients typially present with symptms f shulder pain in 44% t 96%, Hrner’s syndrme (ptsis, misis, anhidrsis) in 14% t 50%, and weakness and atrphy f intrinsi musles f the hand in 8% t 22%. Chest radigraphs shw asymmetri paity in either apex (Figs. 65.1 and 65.2). Chest mputed tmgraphy (CT) better haraterizes superir sulus tumrs and an delineate inltratin f the adjaent mediastinal fat and ther strutures Fig. 65.3. Magneti resnane imaging (MRI) is mre aurate
than CT fr determining the resetability f a superir sulus tumr wing t its superir ability t evaluate tumr extensin int the vertebral bdy, neural framina, spinal rd, and brahial plexus. Psitrn emissin tmgraphy (PET)-CT allws fr detetin f ndal and distant metastases and is thus helpful fr staging. Like ther nn-small ell lung aners, superir sulus tumrs are staged using the TNM staging system. Operable andidates are usually treated with preperative radiatin and hemtherapy fllwed by surgery. REFERENCES
Bruzzi JF, Kmaki R, Walsh GL, etal. Imaging f nn–small ell lung aner f the superir sulus. Part 1: anatmy, linial manifestatins, and management. Radiographics. 2008;28:551–560. Thrai Radilgy: The Requisites, 3rd ed, 405–427.
CASE 66 Tuberculosis (Post-Primary Type) 1. A, B, and C. The mputed tmgraphy (CT) images shw a large avitary lesin in the right lung. TB (pst-primary pattern), fungal pneumnia, and primary lung aner are all pssible diagnses. Pulmnary infarts are usually smaller, invlve a peripheral prtin f the lung parenhyma abutting the pleural surfae, and seldm avitate unless infeted. 2. D. Squamus ell arinma is the mst frequent lung aner ell type assiated with avitatin. 3. C. Tree-in-bud paities were rst desribed in the setting f TB. They indiate endbrnhial spread f disease int the small airways (brnhiles). Typially, the brnhiles are nt visualized n hest CT unless there is impatin f material (pus, bld, tumr, et.) within them. 4. A. There is a dual arterial bld supply t the lungs mprised f the pulmnary and brnhial arteries. Lalized disruptin f pulmnary arterial w urs with pulmnary emblism. If there is superimpsed ngestive heart failure, the brnhial arterial supply is als affeted. This mbinatin inreases the likelihd f develpment f fal pulmnary infart within the territry f the emblism.
Comment The terms “primary” and “pst primary” tuberulsis are inaurate. DNA ngerprinting has demnstrated that in adults, there is n signiant differene in the imaging features between reently aquired tuberulsis and remtely aquired tuberulsis. Hwever, knwledge f the patterns assiated with the traditinal lassiatin is still imprtant. The primary pattern f tuberulsis manifests as nslidatin, ften in the mid r lwer lung znes, with lymphadenpathy. This pattern is mre mmn in hildren and in human immundeieny virus (HIV) psitive patients with lw CD4 unts. The pst primary pattern f tuberulsis is haraterized by upper lung paities ften with assiated avitary r bravitary disease. Chest mputed tmgraphy (CT) axial image demnstrates a avitary lesin with assiated fal areas f tree-in-bud paities in the right lung. The patient had knwn tuberulsis. Based n imaging ndings, this wuld be mst nsistent with a pst primary pattern f tuberulsis. Tree-in-bud paities suggest endbrnhial spread f disease. Thugh riginally desribed in the setting f tuberulsis, and thught t be pathgnmni fr this nditin, we nw knw that this imaging pattern is quite mmn in ther inammatry nditins invlving the small airways (brnhiles) inluding aspiratin and nn-tuberulus mybaterial pneumnia. Any nditin, whih leads t impatin f the brnhiles an prdue the tree-in-bud pattern. Rarely, the tree-in-bud pattern is due t small metastati fi
Fair Game
within pulmnary arteries, whih mirrr the branhing pattern f the airways with whih they are in prximity. REFERENCE
Jeng YJ, Lee KS. Pulmnary tuberulsis: up-t-date imaging and management. AJR Am J Roentgenol. 2008 Sep;191(3):834–844.
CASE 67
anatmi-metabli rrelatin. Reent data indiates that diffusin weighted images (DWI) n MRI ffer better speiity than PET-CT. REFERENCES
Kligerman S, Abbtt G. A radilgi review f the new TNM lassiatin fr lung aner. AJR Am J Roentgenol. 2010;194:562–573. Thrai Radilgy: The Requisites, 3rd ed, 405–427.
Lymphadenopathy From Lung Cancer
CASE 68
1. A, B, D, and E. The subarti (AP windw), lw paratraheal, para-arti, and hilar lymph ndes are all FDG avid nsistent with ndal metastases in these statins. FDG PET CT image shws that the right upper lbe ndule, the paratraheal, para-arti, and prevasular lymph ndes are all FDG-avid, nsistent with ndal metastases (Figs. 67.1 and 67.2). FDG PET CT image mre inferirly demnstrates a highly FDG-avid right hilar lymph nde (Fig. 67.3). Para-arti and prevasular lymph ndes are nt enlarged. N FDG avid high paratraheal lymph ndes are visualized in the prvided images. The distintin between high paratraheal and lw paratraheal lymph ndes is the tp f the arti arh. 2. C. Cntralateral mediastinal lymphadenpathy (N3 disease) makes this patient’s disease stage III B whih is nsidered unresetable. Islated T3 r N2 disease is ptentially resetable. Obviusly, metastati disease is a ntraindiatin fr surgery (M1) but there is n denite evidene f distant metastases in the prvided images. 3. A. The right upper lbe is mst mmnly affeted by lung aner. Interestingly, in the setting f pulmnary brsis, lung aner is mst ften lated within the lwer lbes. 4. D. Spiulated margins have a high assiatin with lung aner and wuld prdue a true psitive result with PET/CT in mst ases. Lesins smaller than 1 m may be difult t image reliably using PET imaging unless fal metabli ativity is high. Carinid tumrs ften have lw FDG avidity beause f their slw grwth rate. Lung aners with grund glass attenuatin (typially adenarinma) are assiated with lw FDG ativity.
Subsolid Nodule (Adenocarcinoma) 1. A, B, C. and D. The magnied CT image in Fig. 68.1 demnstrates a fal grund-glass ndule within the superir segment f the left lwer lbe. All f the listed nsideratins may manifest in this manner. On sagittal r rnal refrmats, sar may be distinguished as having a at r linear nguratin. 2. B. A mixed grund-glass and slid pulmnary ndule is mst nerning fr primary lung aner (mst ften adenarinma) (Fig. 68.2). Slid ndules with entral avitatin may be malignant but are frequently f infetius rigin (e.g., septi embli, fungal pneumnia) r inammatry (e.g., granulmatsis with plyangiitis, rheumatid ndules). 3. D. Pure grund-glass ndules measuring 6 mm r smaller require n further dediated CT fllw-up per urrent Fleishner Siety guidelines (see Referene). 4. B. Centrilbular grund-glass ndules are mmnly fund in the setting f hypersensitivity pneumnitis, respiratry brnhilitis, and pulmnary hemrrhage. Metastati disease presenting with suh a CT pattern wuld be highly unusual.
Comment On thin-setin CT, a malignant slitary pulmnary ndule (SPN) may be f sft tissue (slid), pure grund-glass attenuatin, r a mbinatin f bth grund-glass and slid mpnents (i.e., part-slid r mixed attenuatin). Of thse patterns, the part-slid ndule is the mst likely t be malignant (Fig.68.2),
Comment Lymph nde enlargement is a mmn ause f a mediastinal r hilar mass and shuld be suspeted whenever a spherial r vid mass r masses are identied within a knwn anatmi lymph nde latin. There are a variety f infetius, inammatry, and neplasti auses f thrai lymph nde enlargement. Neplasti etilgies inlude primary lung aner, metastati disease, and lymphma. Bth CT and magneti resnane imaging (MRI) rely n the anatmi features f lymph ndes, mst ntably lymph nde size (shrt axis larger than 1 m), t distinguish between malignant and benign lymph ndes. This strategy is limited by a lw sensitivity and speiity. Thus, in patients with primary lung aner, enlarged ndes must be bipsied fr staging purpses. FDG-PET imaging, whih relies upn metabli (gluse metablism) rather than anatmi features, is the mst aurate nninvasive imaging test fr assessing mediastinal lymph ndes. False negatives are unmmn and usually related t small lymph nde size (mirspi metastases.) Hwever false psitives are mre frequent beause f the FDG avidity seen in inammatry ndes. The auray f FDG-PET an be further enhaned by using an integrated PET-CT sanner, whih imprves visual quality and quantitative auray f PET images, while ptimizing
187
Fig. 68.2
188
SECTION IV
Answers
and the sft tissue attenuatin ndule is the mst likely t be benign. Arding t urrent guidelines, slitary subslid ndules require n further imaging fllw up if they measure less than 6 mm. A grund-glass ndule larger than 6 mm an be reevaluated with CT at 6 t 12 mnths t nrm persistene. If it persists, annual CT fllw-up every 2 years fr a minimum f 5 years is then suggested. Part-slid ndules greater than r equal t 6 mm in ttal dimensin are fllwed by CT at 3 t 6 mnths t nrm persistene. If unhanged r if the slid mpnent remains retiular abnrmality)—the answer in this ase d. Cnslidatin in brnhpulmnary segment(s)/lbe(s) e. Prfuse mirndules (bilateral, predminantly upper lbes) f. Disrete ysts (multiple, bilateral, away frm areas f hneymbing) g. Diffuse msai attenuatin/air-trapping (bilateral, in three r mre lbes) 3. C. UIP is mst mmnly basilar and peripheral in distributin. It is haraterized by retiular paities whih extend t the pleural surfae. Subpleural sparing wuld be highly unusual in UIP and muh mre supprtive f a NSIP diagnsis. Hwever, a substantial minrity f UIP (up t 25%) may nt be basilar predminant in znal distributin. 4. D. NSIP is ften seen in llagen vasular disease, as a drug reatin r it may be the histlgi pattern identied by the pathlgist in sme patients with hypersensitivity pneumnitis. Idipathi NSIP is muh less mmn.
Comment Nnspei interstitial pneumnitis (NSIP) is lassied as ne f the idipathi interstitial pneumnias. Pathlgially, it is haraterized by temprally and spatially hmgeneus interstitial inammatin with r withut brsis. Traditinally, NSIP has been ategrized as either ellular, brti, r mixed ellular and brti. On CT, NSIP is typially basilar predminant but has a variable pattern in the axial plane (peripheral, diffuse, r entral). Grund-glass paity is almst always present with variable degrees f superimpsed retiulatin and tratin brnhietasis as well as brnhiletasis. Often the degree f tratin brnhietasis appears t be ut f prprtin t the degree f nmitant lung disease. A signiant degree f subpleural hneymbing is unusual. Hwever, a small minrity f ases f NSIP eventually prgress t UIP. Therefre a mbined NSIP and UIP CT pattern may asinally be bserved. In ntrast t UIP, whih is mst ften idipathi, NSIP is almst always sendary t an underlying nditin--mst mmnly llagen vasular disease, mediatin/drugs, r hypersensitivity pneumnitis (HP). The esphagus is dilated in this ase, nsistent with esphageal dysmtility in this patient with underlying histry f llagen vasular disease. The main imaging differential diagnsis fr NSIP inlude UIP and HP. Differentiatin f NSIP frm UIP and HP may be difult if nt impssible in sme ases. The mst helpful differentiating ndings f NSIP frm UIP are substantial grund glass abnrmality and subpleural sparing. A substantial degree f grund-glass paity in UIP withut a superimpsed nditin wuld be unusual. In additin, subpleural sparing in UIP. UIP lassially riginates in the subpleural lung and prgresses mre entrally. In many ases f brti HP, the distributin f lung disease is upper lung predminant, whih wuld be unusual in UIP and rare in NSIP. Als, there is very ften substantial air trapping in HP, whih wuld be unusual in islated UIP r NSIP. In patients with NSIP features f llagen vasular disease may be evident n imaging inluding esphageal dilatin (esphageal dysmtility), pulmnary arterial dilatin (pulmnary hypertensin), and pleural r periardial effusin/ thikening.
Fair Game
REFERENCES
Kelsh TL, Chung JH, Lynh DA. Radilgi evaluatin f idipathi interstitial pneumnias. Clin Chest Med. 2015 Jun;36(2): 269–282. Thrai Radilgy: The Requisites, 3rd ed, 355–376.
CASE 75 Ascending Aortic Aneurysm 1. A, B, C, and D. All f the listed nditins are ptential auses fr the develpment f asending arti aneurysms. 2. D. Typially, thrai arti aneurysms are nsidered fr repair when they are larger than 5 t 6 m r are grwing at a rapid rate. 3. D. Biuspid arti valves are assiated with asending arti aneurysms, intraranial arterial aneurysms, Turner syndrme, and arti artatin. Partial anmalus pulmnary venus return is nt mmnly assiated with biuspid arti valves. 4. B. The desriptin f the imaging ndings is nsistent with annularti etasia, whih is mst ften assiated with Marfan syndrme r Ehlers-Danls syndrme.
Comment An aneurysm is dened as an abnrmal dilatin f a vessel. Regarding the asending arta, there is sme variability in diameter with inreasing age, but a diameter f greater than 4 m is generally nsidered abnrmal. Axial image frm ntrast enhaned hest mputed tmgraphy (CT) shws a very large asending arti aneurysm measuring 5.7 m. A small right pleural effusin is nted. Aneurysms may be lassied based n the integrity f arta wall (true vs. false), latin, and shape. Regarding shape, fusifrm aneurysms are haraterized by ylindrial dilatin f the entire irumferene f the arta, and saular aneurysms are haraterized by a fal utpuhing f the arta. Fusifrm aneurysms are mst assiated with atherslersis, whereas saular aneurysms are mst ften traumati r infetius in etilgy. Thrai arti aneurysms are less mmn than abdminal arti aneurysms. Up t ne furth f thrai arti aneurysms exist with an abdminal arti aneurysm Althugh aneurysmal dilatin f the asending arta is frequently aused by atherslersis, this press usually invlves ther prtins f the arta as well. Annularti etasia refers t the presene f dilated sinuses f Valsalva with effaement f the sintubular juntin, resulting in a pear-shaped asending arta that tapers t a nrmal- aliber arti arh. This disrder may be idipathi r assiated with nnetive tissue disrders suh as Ehlers-Danls and Marfan’s syndrmes. Syphilis, ne a relatively mmn ause f asending arti aneurysms, is nw rare. Asending arti aneurysms may als ur in the setting f biuspid arti valves, irrespetive f any funtinal valvular w abnrmality (stensis r regurgitatin) suggesting an inherent weakness f the arti wall in these patients. The majr mpliatin f aneurysms is rupture. The risk fr rupture is related t the size f the aneurysm. Fr this reasn, eletive surgial repair is generally remmended when aneurysms exeed 5 t 6 m in diameter. REFERENCES
Agarwal PP, Chughtal A, Matzinger F, Karerni EA. Multidetetr CT f thrai arti aneurysms. Radiographics. 2009;29:537–552. Thrai Radilgy: The Requisites, 3rd ed, 97–136.
191
CASE 76 Carcinoid Tumor 1. D. Cmputed tmgraphy (CT) images demnstrate a fal endbrnhial tumr in the prximal right lwer lbe brnhus with a fus f arse peripheral aliatin. In a yung patient, arinid tumr is the mst likely diagnsis. 2. B. The mst mmn benign pulmnary tumr is a hamartma. The ther hies are less mmn benign tumrs f the lung. 3. C. Apprximately 30% f arinid tumrs demnstrate aliatin. 4. D. DIPNECH urs almst exlusively in middle-aged r elderly wmen. On CT, the mbinatin f small, welldened pulmnary ndules (representing arinid tumrs and tumrlets) in the mid and lwer lung znes and substantial air trapping is suggestive f the diagnsis, espeially in a patient with refratry asthma-like symptms.
Comment Brnhial arinid tumrs are unmmn neurendrine neplasms that ur entrally (80%) mre mmnly than peripherally (20%). Affeted patients are usually in the third t seventh deade f life and typially present with ugh, hemptysis, and pstbstrutive pneumnia. On hest radigraphs, arinids typially appear as a entral, hilar, r perihilar mass that may be assiated with pstbstrutive ateletasis, pneumnia, muid impatin, r brnhietasis. On CT, arinids typially demnstrate well-dened margins and slightly lbulated brders. Carinids are usually lated lse t the entral brnhi, usually near airway bifuratins. Caliatin is bserved in apprximately 30% f ases n CT but is nt usually evident n nventinal radigraphs. Mst lesins demnstrate intense ntrast enhanement. A minrity f arinids present as a slitary pulmnary ndule (SPN) in the periphery f the lung. Typial arinid tumrs in the periphery f the lungs usually grw at a slw rate. Atypial arinids, whih aunt fr 10% f all arinids, ur mst ften in the lung periphery. These lesins are usually large at the time f presentatin and grw at a faster rate than typial arinids. Althugh typial arinids rarely metastasize, atypial arinids exhibit metastases in up t half f patients. Therapy f arinid tumrs nsists f surgial resetin, with a mre aggressive surgial apprah fr atypial lesins. Adjuvant hemtherapy has als been emplyed with sme suess in patients with advaned atypial arinid tumrs. Typial arinids have an exellent prgnsis, with a 5-year survival f apprximately 90%. In ntrast, atypial arinids are assiated with a 5-year survival f apprximately 70%. Beause arinid tumrs have a high number f smatstatin reeptrs, sintigraphi imaging with the radilabeled smatstatin analgue tretide may be helpful fr deteting ult tumrs. Cnversely, urdexygluse–psitrn emissin tmgraphy (FDG-PET) imaging is less useful in this setting beause f a high rate f false-negative results fr typial arinid tumrs. REFERENCES
Bensn RE, Rsad-de-Christensn ML, Martínez-Jiménez S, Kunin JR, Pettavel PP. Spetrum f pulmnary neurendrine prliferatins and neplasms. Radiographics. 2013 Ot;33(6):1631–1649. Little BP, Junn JC, Zheng KS, Sanhez FW, Henry TS, VeeraraghavanS, Berkwitz EA. Diffuse idipathi pulmnary neurendrine ell hyperplasia: imaging and linial features f a frequently delayed diagnsis. AJR Am J Roentgenol. 2020 De;215(6):1312–1320. Thrai Radilgy: The Requisites, 3rd ed, 391–404
192
SECTION IV
Answers
CASE 77 Boerhaave Syndrome 1. A, B, and C. The hest radigraph (Fig. 77.1) demnstrates a large amunt f etpi gas entered within the inferir aspet f the mediastinum. Small pleural effusins with bibasilar ateletasis are als present. Axial ntrast enhaned hest mputed tmgraphy (CT) image (Fig. 77.2) demnstrates a large amunt f pneummediastinum arund the distal esphagus and arta as well as small pleural effusins in this patient with Berhaave syndrme. The differential diagnsis fr pneummediastinum inludes freful exhalatin against a lsed glttis, asthma, esphageal r large airway injury, and extensin f retrperitneal r retrpharyngeal gas. Extensin f air frm a pneumthrax int the mediastinum is nt mmn. The mst likely diagnsis given the nentratin f gas arund the dilated esphagus is esphageal rupture in this patient with histry f vmiting, the Berhaave syndrme. 2. B. Pneummediastinum in the setting f asthma is thught t be due t the Maklin effet. In the setting f small airway bstrutin, alvelar rupture allws gas t trak alng the pulmnary interstitium entrally int the hila and eventually the mediastinum. 3. D. Esphageal rupture in the setting f Berhaave syndrme typially urs in the left lwer aspet f the esphagus. 4. C. Mediastinitis is the mst serius mpliatin f esphageal rupture with high mrtality reprted. Treatment ranges frm medial management t endspi therapy and/r surgery depending n the severity f the rupture and whether it is ntained. Hwever, in all ases, nthing shuld be allwed by muth and the patient shuld be plaed n brad-spetrum intravenus antibitis, prtn pump inhibitr, and any lalized uid lletins drained. Empyema partiularly in the left pleural spae is nt unmmn.
Comment Esphageal perfratin is a mmn ause f aute mediastinitis and may ur sendary t a variety f mehanisms. Berhaave syndrme refers t transmural perfratin f the distal esphagus that results frm repeated episdes f vmiting. Rupture typially urs psterirly, near the left diaphragmati rus. Patients with esphageal perfratin typially present with symptms f fever, leukytsis, dysphagia, and retrsternal hest pain, whih ften radiates int the nek. Pneummediastinum is a frequent hest radigraphi nding as in this ase. It an be subtle, and air may be difult t appreiate n the standard hest radigraph. Air r gas in the sft tissues f the nek an be a helpful sign. Additinal hest radigraphi ndings may inlude diffuse mediastinal widening, pneumthrax, pleural effusin, and empyema. When the diagnsis f esphageal perfratin is delayed, additinal mpliatins may inlude mediastinal absess, esphagpleural stula, and esphagbrnhial stula. A diagnsis f suspeted esphageal perfratin an be made fllwing the administratin f water-sluble ntrast medium. Suh a study demnstrates extravasatin f ntrast at the site f perfratin, but false negatives ur in up t 10% f ases. CT may be helpful in ases fr whih urspy is nndiagnsti. It may als be helpful t delineate the latin and extent f uid lletins in ases that have prgressed t mediastinal absess frmatin and empyema. It is imprtant t be aware A delay f greater than 24 hurs in the diagnsis f this mpliatin
is assiated with high mrbidity and mrtality rates. Thus, prmpt diagnsis and treatment are ritial. REFERENCES
Yung CA, Menias CO, Bhalla S, Prasad SR. CT features f esphageal emergenies. Radiographics. 2008 Ot;28(6):1541–1553. Katabathina VS, Restrep CS, Martinez-Jimenez S, Riass RF. Nnvasular, nntraumati mediastinal emergenies in adults: a mprehensive review f imaging ndings. Radiographics. 2011 Jul–Aug;31(4):1141–1160. Nrtn-Gregry AA, Kulkarni NM, O’Cnnr SD, Budve JJ, Zrn AP, Desuhes SL. CT esphaggraphy fr evaluatin f esphageal perfratin. Radiographics. 2021 Mar–Apr;41(2):447–461. Thrai Radilgy: The Requisites, 3rd ed, 97–136, 226–237.
CASE 78 Bronchiectasis 1. A, B, and C. The axial CT image demnstrates ndings nsistent with brnhietasis. The differential diagnsis wuld inlude nntuberulus mybaterial pneumnia, allergi brnhpulmnary aspergillsis, and ysti brsis. There is n evidene f pulmnary brsis t suggest usual interstitial pneumnitis. Tratin brnhietasis is a feature f pulmnary brsis but ther ndings f brsis suh as retiulatin, arhitetural distrtin, and/r hneymbing are absent. 2. D. In the setting f severe brnhietasis, differentiatin f brnhietasis frm ysti lung disease may be difult. A helpful CT pstpressing tehnique is minimum intensity prjetin. With the use f thik minimum intensity prjetin slies, the ntinuity f ysti air spaes with mre nrmal appearing entral brnhi is readily apparent in ases f brnhietasis. 3. A. Cylindrial, variid, and ysti (saular) brnhietasis are all subtypes f brnhietasis Spherial brnhietasis isnt a subtype. 4. B. Up t 5% f adults with ysti brsis will develp hepati irrhsis. With the inrease in life span f patients with ysti brsis, mrbidity and mrtality frm hrni liver disease is likely t inrease.
Comment The term bronchiectasis refers t abnrmal, irreversible dilatin f the brnhi. The denitive pathlgi desriptin f brnhietasis was reprted by Reid and is based n the mrphlgy f the brnhi and the number f brnhial subdivisins that are present. In ylindrial brnhietasis, the brnhi are minimally dilated and have a straight, regular ntur (Fig. 78.2). The average number f brnhial subdivisins frm the hilum t the lung periphery is 16 (17 t 20 is nrmal). In variid brnhietasis, the brnhi demnstrate a beaded appearane with sequential dilatatin and nstritin (Fig. 78.3). The average number f brnhial divisins is 8. In ysti brnhietasis, the brnhi have a ballned appearane (Fig. 78.4). The average number f brnhial divisins is nly 4. Brnhietasis an be distinguished frm ysti lung disease by applying the fllwing imaging riteria: 1. When dilated brnhi urse perpendiular t the sanning plane, a pulmnary artery r arterile will always urse adjaent t it (signet ring sign). In ntrast, true lung ysts, suh as thse assiated with LAM, are lated randmly in the lung parenhyma. 2. Dilated brnhi urse parallel t the sanning plane, yu will bserve that the ysti spaes nnet with ne anther. 3. Cysti brnhietasis is ften assiated with air-uid levels, a nding that is nt generally bserved in ysti lung disease.
Fair Game
193
Fig. 78.2
Fig. 78.4
REFERENCES
Millirn B, Henry TS, Veeraraghavan S, Little BP. Brnhietasis: Mehanisms and imaging lues f assiated mmn and unmmn diseases. Radiographics. 2015 Jul–Aug;35(4):1011–1030. Thrai Radilgy: The Requisites, 3rd ed, 137–158.
CASE 79 Giant Bulla 1. A and B. The imaging ndings are highly suggestive f very large bulla. Althugh, a right-sided pneumthrax uld mimi this appearane. Hwever, the absene f a distint viseral pleural line as well as gradual attenuatin f lung vessels at the juntin f the superir aspet f the right lung and mid aspet f the right lung are muh mre suggestive f a bulla rather than a pneumthrax. A prtin f the inferir wall f the bulla an be identied inferirly and medially. 2. B. A bulla is dened as a gas ntaining spae in the lung ≥ 1m with a thin well-dened wall. Bth bullae and blebs represent subpleural ysti air spaes, whih are usually well-dened. 3. D. Of the listed mdalities, hest CT is the mst sensitive tl t detet early emphysema. 4. D. Althugh basilar predminant panlbular emphysema is almst always assiated with alpha-1 antitrypsin deieny, in a minrity f patients, previus IV Ritalin use is the underlying ause f this pattern.
Comment Fig. 78.3
The hest radigraph in this ase demnstrates a large area f absent pulmnary vessels in the right lung with sparing f the right
194
SECTION IV
Answers
base It displaes the remainder f the lung inferirly and appears t have a well-dened wall inferirly and medially. A large pneumthrax is a nsideratin. Hwever, the absene f a viseral pleural line and gradual transitin frm the nrmal lung parenhyma t an area f absent pulmnary vessels demarated by a wall indiates the presene f a large bulla rather than a pneumthrax. Bullae may develp in assiatin with any type f emphysema, but they are mst mmnly assiated with paraseptal and entrilbular emphysema. Hwever, they are nt always assiated with diffuse emphysema. Bullae usually enlarge ver mnths t years, but the grwth rate is quite variable. Oasinally, bullae an beme quite large and may be fal in distributin. Large bullae may mprmise respiratry funtin. The resulting syndrme has been referred t by varius terms, inluding bullus emphysema, vanishing lung syndrme, and primary bullus disease f the lung. This entity urs mst ften in yung men and is haraterized by large, prgressive upper lbe bullus disease. Althugh it may ur in nnsmkers, mst affeted patients are smkers. Cmputed tmgraphy (CT) is the preferred mdality fr the assessment f patients with suspeted bullus emphysema. CT is helpful fr delineating the number, size, and latin f bullae. It an als assess the degree f mpressin f underlying nrmal lung and determine the presene and severity f emphysema in the remaining prtin f the lung parenhyma. In symptmati patients, surgial resetin f bullae an result in marked imprvement in pulmnary funtin. The greatest benet frm surgery is bserved in patients with a large bulla (upying 50% r mre f a hemithrax) and a mderate redutin in fred expiratry vlume in 1 send (FEV1). In ntrast, patients with severe generalized emphysema tend t d prly and are thus nt ideal andidates fr bulletmy. REFERENCES
Lynh DA, Austin JH, Hgg JC, Grenier PA, Kauzr HU, Bankier AA, Barr RG, Clby TV, Galvin JR, Gevenis PA, Cxsn HO, Hffman EA, Newell Jr JD, Pistlesi M, Silverman EK, Crap JD. CT-denable subtypes f hrni bstrutive pulmnary disease: a statement f the Fleishner Siety. Radiology. 2015 Ot;277(1):192–205. Thrai Radilgy: The Requisites, 3rd ed, 19–60, 391–404.
CASE 80 RML and RLL atelectasis 1. D. The psteranterir (PA) radigraph demnstrates a fal paity in the entral and inferir aspet f the right lung with bsuratin f the right hemidiaphragm and right heart brder. The upper brder is sharply dened, and that brder extends medial t the interlbar artery in the right hilum. The upper brder is prdued by the depressed minr ssure laterally and the majr ssure medially (arrw) (the minr ssure an never extend medial t the lateral brder f the interlbar artery s the lwer lbe must be llapsed.) There is mild right lung vlume lss suggested by rightward shift in the mediastinum and trahea. These ndings are typial f mbined right middle and right lwer lbe llapse. 2. C. Obstrutin f the brnhus intermedius wuld ause mbined right middle and right lwer lbe ateletasis. The brnhus intermedius arises frm the right mainstem brnhus distal t the takeff f the right upper lbe brnhus. 3. D. Althugh all listed answer hies uld ause mplete right lung ateletasis, in a reently intubated patient, muus plugging is mst likely. If the endtraheal tube is advaned t far, it mst ften extends int the right mainstem brnhus rather than the left mainstem brnhus due t the right mainstem brnhus’s mre btuse angulatin
relative t the trahea; this wuld result in left lung ateletasis rather than right lung ateletasis. In the utpatient setting, mplete left lung ateletasis is very sendary t a entral lung arinma. 4. D. Hepatellular arinma des nt mmnly metastasize t the entral airways. Cmmn distant primary malignanies whih metastasize t the trahea and entral brnhi inlude melanma, renal ell arinma, breast aner, and ln aner.
Comment The radigraphi appearane is typial f mbined right middle and lwer lbe llapse (Fig. 80.1). There is a well-dened paity in the entral aspet f the right lung with assiated silhuetting f the right hemidiaphragm and right heart brder with right lung vlume lss. The sharp upper brder is prdued by the minr and majr ssures whih are bth depressed. Cmbined right middle and lwer lbe llapse an ur when a tumr bstruts the brnhus intermedius. This mbinatin urs mre frequently than mbined right upper and right middle lbe llapse beause the brnhi t these lbes are remte frm ne anther. When the latter mbinatin urs, the appearane is idential t left upper lbe llapse. In this patient, the mbined lbar llapse urred sendary t endbrnhial metastati disease. Als nte the presene f pulmnary metastases, best visualized in the left lung. Endbrnhial metastases are unmmn and are fund in less than 5% f patients at autpsy. Presenting symptms may inlude ugh, wheeze, and hemptysis. Cughing may infrequently result in expetratin f tumr fragments; rarely, this is the rst indiatin f metastati disease. Radigraphi ndings in the setting f partial airway bstrutin inlude ligemia and air trapping. In the setting f mplete brnhial bstrutin, ndings inlude lbar, segmental, r subsegmental ateletasis and pstbstrutive pneumnitis. Ahilar r entral mass may als be evident. REFERENCES
Se JB, Im JG, G JM, etal. Atypial pulmnary metastases: spetrum f radilgi ndings. Radiographics. 2001;21:403–417. Thrai Radilgy: The Requisites, 3rd ed, 19–60.
CASE 81 Lipoma 1. A and C. Axial image frm a nn-ntrast hest CT demnstrates a large fatty mass in the left axilla. Crnal nnntrast T1-weighted MRI demnstrates diffuse hyperintense signal thrughut the mass whih extends superirly int the nek harateristi f a lipma. The imaging ndings are essentially diagnsti f a lipma. There is a small amunt f wispy sft tissue within this lesin; therefre, lipsarma uld be nsidered in the differential diagnsis but is muh less likely. 2. D. On T1 weighted MRI, nn-prteinaeus r hemrrhagi uid is usually T1 hypintense. The ther listed materials (fat, melanin, methemglbin) are all T1 hyperintense. 3. C. The greater degree f sft tissue within a fatty mass, the greater the likelihd f lipsarma. 4. C. Lipmatus hypertrphy f the interatrial septum represents a benign fatty inltratin f the atrial septum. This lesin is typially dumbbell shaped that lassially spares the fssa valis. Thugh this lesin is usually asymptmati, it may be a ause f symptmati arrhythmias and vasular bstrutin. In 80% f patients, there is inreased FDG uptake n PET-CT, likely beause f metablially ative brwn fat.
Fair Game
Comment The CT and MRI images demnstrates a fatty mass in the left axilla that ntains a minimal amunt f sft tissue attenuatin material. T1 hyperintensity is typial f fatty lesins. The mass extends int the lwer aspet f the left nek. Lipmas may ur in a variety f latins in the thrax, inluding the mediastinum, hest wall, extrapleural spae, esphagus, heart, airway, and rarely, the lung parenhyma. Althugh lipmas typially appear as well-marginated lesins haraterized by hmgeneus fat attenuatin, sft tissue elements may be bserved. In suh ases, it may nt be pssible t distinguish lipma frm thymlipma r lw-grade lipsarma. The pliability and lak f invasiveness f lipmas may aid in their differentiatin frm lipsarmas; fr example, lipmas typially drape arund adjaent vessels, ribs, and mediastinal strutures withut invading them. Lipsarmas typially ntain a larger sft tissue mpnent, have irregular margins, and frequently invade adjaent mediastinal and hest wall strutures. Thus, the presene f well-dened margins and lak f invasiveness favr a diagnsis f lipma ver lipsarma. REFERENCES
Gaerte SC, Meyer CA, Winer-Muram HT, etal. Fat-ntaining lesins f the hest. Radiographics. 2002;22:S61–S78. Thrai Radilgy: The Requisites, 3rd ed, 159–192.
CASE 82 Apical Cap Secondary to Extrapleural Hematoma 1. A and D. Bld frm interstal vessel injury, rib frature, r disseting mediastinal hematma (ruptured arta) an aumulate between the parietal pleura and ribs, frming an apial ap. Mst aute hemthraes aumulate dependently. Hwever, pleural bld uld ause an apial ap if the patient had pre-existing pleural adhesins. Pulmnary ntusin manifests as lung nslidatin r grund-glass paity in a nnanatmi distributin. Asbests-related pleural thikening usually extends t the stphreni angle. The apial pleura is typially spared. The ap in this ase is lnger in dimensin than a lassi pleural plaque. 2. C. Bne destrutin adjaent t a pleural ap is mst suggestive f a primary lung arinma (superir sulus tumr). Hilar lymphadenpathy and pleural effusin an be aused by extrathrai tumr metastases r inammatin. Mediastinal widening may be prdued by infetin (mediastinitis) r hematma bth f whih an trak int the apial extrapleural spae. 3. C. Many patients develp apial brsis and pleural sarring with age. Mst f these patients are asymptmati. The apial aps usually have irregular margins, are less than 5 mm thik, and are typially symmetri. The prevalene inreases with age. Apial aps frm extensin f mediastinal hematma are typially unilateral and mre mmnly ur n the left. These may be the result f a traumati arti injury. Radiatin fr breast arinma typially is limited t the anterir hest wall. Radiatin t these areas an auses brsis in the adjaent ipsilateral lung apex and result in a unilateral apial ap. Head and nek arinmas ften metastasize t ervial and supralaviular lymph ndes and an prdue an apial ap when the lymph nde metastases are large. These metastases are mre likely t be unilateral than bilateral. Als, radiatin t the ipsilateral nek in suh ases may als prdue an apial ap.
195
4. A. CT f the hest an best haraterize the abnrmality and delineate its full extent. Althugh MRI an prvide a lt f infrmatin, a CT san is a better rst-line test t further haraterize the abnrmality beause f its superir spatial reslutin and ability t rapidly image the entirety f the hest and nek. Ultrasund may be useful t assess pleural and extrapleural abnrmalities, but it is limited in its ability t fully delineate the extent f disease. Furthermre, deep tissues may nt be visible. FDG-PET des nt have a rle in evaluating the autely ill patient.
Comment Differential Diagnosis AP radigraph (Fig. 82.1) shws a left apial ap. There is a subtle displaed frature f the left rst rib, indiating that extrapleural hematma r lulated hemthrax is the mst likely ause. Crnal refrmatted ntrast-enhaned CT image (Fig. 82.2) nrms the presene f bld in the upper left hemithrax. The bld abuts the inner margin f the adjaent ribs, indiating an extrapleural latin. The relatively smth margins and the presene f an aute rib frature argue against neplasm.
Discussion The term apical cap has been used t desribe the presene f paity lated in r adjaent t the extreme apex f the lung n hest radigraphs (see Fig. 82.1). On hest radigraphs f nrmal, asymptmati patients, biapial smth r irregular paities lated at the apies f the lungs, usually measuring less than 5 mm in diameter, are ften present. The lwer margin is usually sharply delineated but ften has an undulating brder. Apial aps are thught t represent the result f nnspei subpleural sarring and apial pleural thikening, and they are usually f n linial signiane. The prevalene f apial aps inreases with age. This nding when islated des nt represent ld healed granulmatus disease (i.e., tuberulsis). A variety f entities an result in an enlarged apial ap. The varius auses f a unilateral enlarged ap inlude lymphma r absess extending dwn frm the nek, primary lung arinma (superir sulus r Panast tumr), and mediastinitis r hematma traking int the apial extrapleural spae. Regarding bilaterally enlarged apial aps, they may be assiated with radiatin brsis (e.g., fr Hdgkin lymphma), mediastinal lipmatsis, and vasular abnrmalities suh as artatin f the arta with enlarged llateral vessels. In this ase, the presene f a smthly marginated enlarged left apial ap is the result f an extrapleural hematma. The apial ap is smthly marginated, reeting the extrapleural latin. Extrapleural hematmas typially d nt require treatment. Ative interstal arterial bleeding may be treated with emblizatin. It is imprtant t distinguish hemthrax frm extrapleural hematma, as the frmer usually requires perutaneus drainage t avid the subsequent develpment f a brthrax. REFERENCES
MLud TC, Isler RJ, Nvelline RA, etal. The apial ap. AJR Am J Roentgenol. 1981;137:299–306. Thrai Radilgy: The Requisites, 3rd ed, 159–192.
196
SECTION IV
Answers
CASE 83
CASE 84
Bleomycin Lung Toxicity
Primary Lung Adenocarcinoma With N2 Nodal Disease
1. A and D. Blemyin lung txiity, and fungal infetin an present as pathy nslidatin in a patient n hemtherapy. Cngestive heart failure typially presents with diffuse lung edema and pleural effusins. Cllagen vasular disease is usually nt assiated with peripheral nslidatin. 2. A. Apprximately 4% f patients reeiving blemyin develp lung txiity. 3. A. Althugh blemyin is used t treat sme patients with lymphma, lymphma itself des nt inrease a patient’s risk f blemyin indued lung txiity. Cnurrent raditherapy, renal insufieny, and advaned age all raise a patient’s risk fr develping blemyin indued lung txiity. 4. C. Mst patients imprve within a few weeks, nt a few days, althugh a small number f patients may take muh lnger t rever, inluding up t tw years. Blemyin must be disntinued t prevent wrsening lung injury, and mst patients are treated with rtisterids, whih are slwly tapered as the patient’s linial nditin imprves.
Comment Differential Diagnosis Axial and rnal mputed tmgraphy (CT) images shw pathy peripheral nslidatin in the right lung. Cnslidatin in a patient treated with hemtherapy an result frm infetin r drug reatin.
Discussion Blemyin is an antitumr agent that is used t treat lymphmas, testiular arinmas, and ertain squamus ell arinmas. Lung txiity urs in apprximately 4% f patients and is the prinipal dse-limiting fatr fr this agent. Fibrsis is the mst serius pulmnary mpliatin but rganizing pneumnia reatin and aute hypersensitivity reatin an ur. Affeted patients typially present with an insidius nset f dyspnea, nnprdutive ugh, and asinal fever. Pulmnary funtin tests reveal a dereased DLco, a sensitive measure fr early blemyin lung injury. Chest radigraphs may be nrmal r might shw retiular paities in a basilar and subpleural distributin. Peripheral and peribrnhial nslidatin an als develp. Thin setin CT is mre sensitive than radigraphs fr deteting lung abnrmalities and an shw harateristi ndings even when the hest radigraph is nrmal. Cnslidatin (Figs. 83.1 and Fig. 83.2) an represent drug-indued rganizing pneumnia, whih an als be mass like r ndular in appearane. Ndules an vary in size frm 5 mm t 3 m. Infetin, partiularly fungal infetin is the ther majr diagnsis t nsider. Organizing pneumnia is usually subpleural in a peripheral distributin and/r in a peribrnhial distributin. In ntrast t metastases, air brnhgrams are ften present within the ndules. Early detetin f blemyin-indued lung injury is imprtant beause prmpt disntinuatin f blemyin an result in imprved lung funtin and healing in patients with early stages f disease. In patients with mre-advaned disease, the prgnsis is variable. Althugh sme patients respnd t sterids, thers develp prgressive brsis, whih an lead t respiratry failure and death. REFERENCES
Rssi SE, Erasmus JJ, MAdams HP, et al. Pulmnary drug txiity: radilgi and pathlgi manifestatins. Radiographics. 2000;20:1245–1259. Thrai Radilgy: The Requisites, 3rd ed, 355–376.
1. A, B, and D. The radigraph shws right hilar and right paratraheal lymphadenpathy, whih an result frm primary lung arinma r metastases, inluding renal ell arinma and testiular arinma amng thers. Primary tuberulsis an als present with hilar and mediastinal lymphadenpathy, althugh this appearane is mre mmn in hildren than adults. Silisis an lead t hilar and mediastinal lymphadenpathy. Hwever, n underlying pneumniti ndules are present n the standard radigraph. Als, lymphadenpathy in silisis is usually bilateral in distributin. 2. C. N2 dentes metastasis t ipsilateral mediastinal r subarinal lymph ndes. N0 dentes n lymph nde metastasis, N1 dentes ipsilateral intrapulmnary, brnhial, r hilar lymph nde metastasis, and N3 dentes any supralaviular, salene, r ntralateral mediastinal r hilar lymph nde metastasis. 3. D. Cmbined with T1-3, N1 disease determines the designatin f stage IIIA. The patient is staged as IIIB when N2 disease urs with a T3 r T4 lesin. Any axillary lymph nde metastasis is staged as M1. In the absene f a T3 r T4 lesin and n distant metastases, sme patients with N2 lymph nde metastases may be andidates fr surgial resetin typially after neadjuvant hemtherapy. FDG-PET is sensitive fr lymph nde metastases and is superir t mputed tmgraphy (CT), but the false psitive rate is high. Surgial r brnhspi (EBUS) ndal bipsy is usually required fr adequate ndal staging. 4. C. Ipsilateral pleural metastasis is staged as M1a and is therefre a ntraindiatin fr resetin. Chest wall invasin is staged as T3 and is ptentially resetable. A tumr metastasis t the same lbe is staged as T3 and ptentially resetable. A entral lesin resulting in bstrutive pneumnia f the entire lung is staged as T2 and may be resetable.
Comment Differential Diagnosis The PA hest radigraph (Fig. 84.1) shws right hilar enlargement and a right lwer paratraheal mass, nsistent with lymphadenpathy. Cntrast-enhaned CT image shws right hilar (Fig. 84.2) and right lwer paratraheal (Fig. 84.3) lymphadenpathy. The differential diagnsis is brad but mst mmnly wuld inlude primary lung arinma, metastases (testiular and renal, espeially), lymphma, and granulmatus infetins suh as tuberulsis and histplasmsis.
Discussion In patients with nn–small ell lung aner, the ndal status prvides imprtant infrmatin fr determining prgnsis and planning apprpriate therapy. Lymph ndes are ategrized int seven spei znes: supralaviular, upper, artipulmnary, subarinal, lwer, hilar-interlbar, and peripheral. N hanges were made fr the N designatin in the 8th editin f the TNM lassiatin system. Arding t the TNM lassiatin system, ndal invlvement is graded frm N0 t N3 as fllws: • N0 = n demnstrable metastases t reginal lymph ndes • N1 = metastasis t lymph ndes in the ipsilateral peripheral r hilar-interlbar regins • N2 = metastasis t ipsilateral mediastinal ndes (upper, artipulmnary, lwer r subarinal)
Fair Game
• N3 = metastasis t any supralaviular ndes, r t ntralateral mediastinal (upper, artipulmnary, lwer), hilar-interlbar, r peripheral regins. CT and magneti resnane imaging (MRI) play an imprtant but limited rle in the assessment f the ndal status in patients with lung arinma. These imaging studies rely primarily n anatmi features f lymph ndes, mst ntably lymph nde size. Shrt axis diameter >1 m is generally nsidered abnrmal. This strategy is assiated with sensitivities in the range f 60% t 79% and speiities in the range f 60% t 80%. Thus, fr staging purpses, enlarged ndes must be evaluated by bipsy. The primary rle f these imaging examinatins is t identify the latin f enlarged ndes. This infrmatin allws apprpriate bipsy predures t be planned. In reent years, FDG-PET imaging has been shwn t be superir t CT and MRI in the assessment f mediastinal lymph nde metastases. This tehnique relies n physilgi (gluse metablism) rather than anatmi features t identify abnrmal lymph ndes. Thus, it has the ptential t identify neplasti invlvement within small ndes and t distinguish enlarged, hyperplasti ndes frm neplasti nes. Hwever, the number f false-psitive and false-negative studies (partiularly in small ndes with mirspi metastases) requires that lymph nde sampling still be perfrmed in mst ases. REFERENCES
Sharma A, Fidias P, Hayman LA, etal. Patterns f lymphadenpathy in thrai malignanies. Radiographics. 2004;24:419–434. Carter BW, Lihtenberger 3rd JP, Benveniste MK, de Grt PM, Wu CC, Erasmus JJ, Trung MT. Revisins t the TNM staging f lung aner: ratinale, signiane, and linial appliatin. Radiographics. 2018 Mar–Apr;38(2):374–391. Thrai Radilgy: The Requisites, 3rd ed, 19–60, 450–459.
CASE 85 Superior Vena Cava Syndrome from Lung Cancer 1. A. Lung aner an invade r metastasize t the mediastinum and mpress r lude the superir vena ava. Althugh there are numerus dilated vessels in the right hemithrax, they ntain undiluted ntrast, whih indiates that they are in ntinuity with the vein int whih the ntrast was administered and nt deriving w frm an arterivenus malfrmatin, whih shuld have bld mixed with ntrast. Extravasated ntrast llets in the sft tissues and des nt ll multiple vessels. 2. C. Lung aner is a mmn disease and ften results in diret mediastinal invasin r mediastinal adenpathy bth f whih an ause SVC bstrutin. Lng-term intravenus devies suh as paemaker leads an ause SVC stensis r thrmbsis. Hwever, these devies are nt the mst mmn auses f SVC syndrme. Fibrsing mediastinitis is assiated with SVC syndrme but is a relatively rare disease. Radiatin therapy an ause brsis in the mediastinum and result in SVC stensis, but its urrene is unusual. 3. D. Dural venus sinus thrmbsis is nt a typial manifestatin f SVC syndrme. Impaired venus return frm the head and nek an result in head and fae edema, extremity edema, and visual disturbanes. 4. D. Targeted radiatin therapy t the mediastinum an lead t rapid imprvement f the symptms f SVC syndrme. Chemtherapy may be useful in hemsensitive tumrs; hwever, it is nt the preferred rst-line treatment fr SVC syndrme related t nn–small ell lung arinma. Surgial resetin is generally nt indiated beause patients with
197
SVC syndrme related t nn–small ell lung arinma generally have unresetable disease. Surgial bypass may be used in sme ases fr palliatin where ther therapies have failed. SVC stenting may be useful in sme patients wh fail initial therapy. Hwever, data are smewhat limited.
Comment Differential Diagnosis Cntrast-enhaned CT image (Fig. 85.1) shws a sft tissue mass inltrating the mediastinum with bliteratin f the superir vena ava. Numerus hest wall and mediastinal llateral vessels are present lled with undiluted ntrast. The inltrating sft tissue mass is mst nsistent with lung arinma. Other neplasti auses wuld inlude lymphma and thymi arinma. Fibrsing mediastinitis an als ause SVC syndrme. This mst mmnly presents with a large, partially alied ndal mass in the mediastinum.
Discussion Superir vena ava (SVC) syndrme is aused by bstrutin f the SVC by external mpressin, intraluminal thrmbsis, neplasti inltratin, r a mbinatin f these presses. Mst ases result frm neplasm, mst mmnly lung arinma (espeially small ell arinma). Lymphma, thymi arinma, and metastati arinma are additinal malignant auses. There are a variety f benign auses, inluding lng-term intravenus devies (e.g., Hikman atheters and permanent paemakers) and brsing mediastinitis. Chest radigraphs ften shw a mass in the right paratraheal regin that may be ampanied by distentin f the azygs vein. In the setting f brsing mediastinitis, the right paratraheal mass is ften an enlarged alied nde. In patients wh develp thrmbsis f the SVC beause f an indwelling atheter, lateral displaement f the atheter may be apparent. Cmputed tmgraphy (CT) r magneti resnane imaging (MRI) an nrm the diagnsis f SVC bstrutin. On CT, the diagnsis is based n dereased r absent ntrast paiatin f the SVC in njuntin with paiatin f llateral vessels. Bth ndings are neessary t make a reliable diagnsis. Cntrast-enhaned CT with multiplanar refrmatin and 3D renstrutins is highly aurate at deteting the presene and level f SVC bstrutin. It is als valuable in determining the ause f bstrutin and fr delineating the llateral venus irulatin. The presene f llateral venus vessels shuld prmpt a searh fr a entral venus bstrutin. Treatment fr SVC syndrme related t lung aner is primarily with radiatin therapy. Chemtherapy may be used in sme patients espeially thse with small ell arinma. Surgial bypass is generally reserved fr patients with benign auses f SVC bstrutin r fr palliatin in thse wh fail radiatin and hemtherapy fr malignany. REFERENCES
Eran S, Karaman A, Okur A. The superir vena ava syndrme aused by malignant disease: imaging with multidetetr rw CT. Eur J Radiol. 2006;59:93–103. Carter BW, Lihtenberger 3rd JP, Benveniste MK, de Grt PM, Wu CC, Erasmus JJ, Trung MT. Revisins t the TNM staging f lung aner: ratinale, signiane, and linial appliatin. Radiographics. 2018 Mar–Apr;38(2):374–391. Thrai Radilgy: The Requisites, 3rd ed, 450–459.
198
SECTION IV
Answers
CASE 86 Diffuse Alveolar Hemorrhage From Vasculitis 1. A and C. The differential diagnsis fr widespread nslidatin in the autely ill patient inludes pulmnary hemrrhage, pulmnary edema, diffuse infetius pneumnia, massive aspiratin, and aute lung injury. Langerhans ell histiytsis is a smking related disease haraterized by upper lung predminant ndules and ysts. Silisis is haraterized by well-dened ndules with an upper lbe predminane with r withut prgressive massive brsis. Aute silisis hwever an prdue diffuse lung disease whih is mstly grund glass in appearane. The pathlgy is that f alvelar prteinsis. 2. C. Pulmnary vasulitis is the mst mmn ause f diffuse alvelar hemrrhage. Hemphilia is rare. Drug indued pulmnary hemrrhage is rare and an ur with therapeuti drugs suh as erltinib r illiit drugs suh as rak aine. Infetin rarely auses diffuse alvelar hemrrhage. 3. B. The mst mmn lung manifestatin f mirspi plyangiitis is diffuse alvelar hemrrhage. Diffuse alvelar hemrrhage related t granulmatsis with plyangiitis (Wegener’s granulmatsis) nly urs in 10% t 15% f patients. Diffuse alvelar hemrrhage frm esinphili granulmatsis with plyangiitis (frmerly Churg-Strauss syndrme) is rare. Takayasu arteritis primarily affets the arta and its branhes. Large pulmnary artery invlvement des ur, but diffuse alvelar hemrrhage is nt a feature. 4. A. The diagnsis f diffuse alvelar hemrrhage is nrmed by brnhalvelar lavage, whih shws prgressive inrease in return f bld in lavage uid. Transbrnhial and surgial bipsy an shw hemsiderin-laden marphages r areas f hemrrhage but are nt the best hies fr diagnsis. Sputum analysis has lw yield and may shw red bld ells but des nt nrm diffuse alvelar hemrrhage.
Comment Differential Diagnosis The prtable anterpsterir (AP) radigraph (Fig. 86.1) shws diffuse lung nslidatin. Cmputed tmgraphy (CT) shws dense bilateral nslidatin psterirly and pathier grundglass paity anterirly (Fig. 86.2). The differential diagnsis fr diffuse lung paity in the autely ill patient is brad and inludes diffuse pneumnia frm infetin, ardigeni and nn-ardigeni edema, hemrrhage, massive aspiratin, r aute lung injury. Hemptysis, when present, and dereased hematrit favr diffuse alvelar hemrrhage.
Discussion Diffuse alvelar hemrrhage (DAH) is haraterized by bleeding int the alvelar spaes and results frm disruptin f the alvelar-apillary basement membrane. The linial presentatin is aute, and patients usually present with ugh, fever, dyspnea, and hemptysis. Hwever, apprximately ne-third f patients with DAH d nt have hemptysis. Althugh a wide variety f insults an lead t DAH, the histpathlgi injury an be gruped int apillaritis, bland hemrrhage, and diffuse alvelar damage (DAD). Capillaritis an result frm systemi vasulitis (espeially anti-neutrphil ytplasmi antibdy assiated), antiglmerular basement membrane antibdies, llagen vasular disease, r be idipathi. Causes f DAD are numerus and inlude infetin, systemi sepsis, reatin t drugs, trauma, and
llagen vasular disease. Bland DAH may develp sendary t left heart failure, antiagulatin, and hemrrhagi disrders. Radigraphi ndings f DAH vary by the extent and severity f DAH. Radigraphs f patients with mild DAH may be nrmal r near nrmal, while thse f patients with severe disease may shw extensive nslidatin. CT ndings are similar, ranging frm mild grund-glass paity t extensive multilbar nslidatin. The distributin f paities tends t be mre entral and perihilar, but the ndings are ften indistinguishable frm ther auses f diffuse lung paity suh as edema, infetin, and alvelar damage. Brnhalvelar lavage is the referene standard fr establishing a diagnsis f DAH. Inreased bld prduts in returned lavage uid is diagnsti f DAH. Hemsiderin-laden marphages an als be identied in lavage uid. Treatment inludes management f underlying auses (when knwn) and supprtive therapy as needed. REFERENCES
Krause ML, et al. Update n diffuse alvelar hemrrhage and pulmnary vasulitis. Immunol Allergy Clini North Am. 2012 Nv;32(4):587–600. Thrai Radilgy: The Requisites, 3rd ed, 238–258, 335–354.
CASE 87 Usual Interstitial Pneumonia (UIP) Caused by Idiopathic Pulmonary Fibrosis (IPF) 1. B. Subpleural and basal predminant retiulatin with hneymbing is highly diagnsti f usual interstitial pneumnia. Desquamative interstitial pneumnia is haraterized by extensive grund-glass paity, smetimes with ampanying mild ne retiulatin and asinal small ysts. Nnspei interstitial pneumnia is usually haraterized by grund-glass paity with r withut retiulatin in basal and peribrnhial distributin. Lymphid interstitial pneumnia an be quite variable but is usually haraterized by grund-glass paity, ndules, and ysts. 2. D. Fr patients with a denite UIP pattern f brsis n CT, bipsy is rarely warranted. Surgial bipsy is nt withut risk and rarely will hange the diagnsis with this pattern. Transbrnhial and CT-guided needle bipsy are nt sufient t make a pathlgi diagnsis f UIP, primarily beause f the small sample size and relative peripheral latin f brsis. A multidisiplinary apprah t diagnsis is imprtant with invlvement f the radilgist, pulmnlgist, and pathlgist. 3. A. A usual interstitial pneumnia (UIP) pattern f brsis is mst strngly assiated with rheumatid arthritis. Nnspei interstitial pneumnia (NSIP) is the mst mmn pattern f diffuse lung disease in patients with prgressive systemi slersis. NSIP and rganizing pneumnia are the mst mmn patterns f diffuse lung disease in patients with plymysitis. Lymphid interstitial pneumnia (LIP) is assiated with Sjögren syndrme. 4. D. Asbests an result in pulmnary brsis (asbestsis), typially with a usual interstitial pneumnia (UIP) pattern f brsis. Cal and silia ause pneumnisis haraterized by small perilymphati ndules predminantly in the upper lungs. Ndules an alese t frm large paities, termed prgressive massive brsis. Beryllium auses a granulmatus respnse in the lungs indistinguishable frm saridsis.
Fair Game
Comment Differential Diagnosis Axial (Figs. 87.1 and 87.2) and rnal refrmatted (Fig. 87.3) CT images shw subpleural and basal predminant retiulatin with hneymbing and tratin brnhietasis, nsistent with UIP pattern f pulmnary brsis. This pattern n CT is highly spei fr UIP and mst mmnly represents IPF.
Discussion UIP is a pattern f hrni diffuse lung injury haraterized by restritive physilgy and redued diffusing apaity. Mst patients with UIP have n identiable ause, termed IPF. Other auses f UIP histlgy and CT pattern inlude llagen vasular disease (partiularly rheumatid arthritis), familial brsis, drug txiity, and asbests expsure (asbestsis). Older age and male sex are risk fatrs fr IPF. Patients present with prgressive dyspnea, dry ugh, restritive pulmnary physilgy, and redued diffusin apaity. Digital lubbing may be present, and arse end expiratry rakles are heard n expiratin, partiularly in the lwer lung znes. The diagnsis f UIP is usually made by hest CT when the harateristi pattern f subpleural and basal predminant retiulatin with hneymbing is present. In mst ases, this CT pattern and linial evaluatin alne are enugh t establish a diagnsis f IPF. Oasinally, a surgial lung bipsy may be required either beause f nfunding ndings n hest CT r linial evaluatin. Histpathlgially, UIP is haraterized by spatially and temprally hetergeneus areas f brsis and lbular llapse with harateristi brblasti fi at the leading edge f brsis. Mirspi hneymbing may be present, and inammatin is minimal r absent. UIP, regardless f ause, is assiated with a pr prgnsis. Many patients with IPF die f their disease within 3 t 4 years f diagnsis. Cmpliatins inlude prgressive respiratry deline, aute exaerbatins, and lung aner. Treatment ptins fr IPF are limited t tw nvel drugs, whih have been shwn t imprve prgressin free survival but nt verall survival REFERENCES
Ryu JH, Mua T, Daniels CE, et al. Idipathi pulmnary brsis: evlving nepts. Mayo Clin Proc. 2014 Aug;89(8):1130–1142. Raghu G, Remy-Jardin M, Myers JL, et al. Amerian Thrai Siety, Eurpean Respiratry Siety, Japanese Respiratry Siety, and Latin Amerian Thrai Siety. Diagnsis f idipathi pulmnary brsis: an fial ATS/ERS/JRS/ALAT linial pratie guideline. Am J Respir Crit Care Med. 2018 Sep 1;198(5):e44–e68. Thrai Radilgy: The Requisites, 3rd ed, 355–376.
CASE 88 Intramuscular Hematoma From Supratherapeutic Warfarin Therapy 1. A and C. Fi f high attenuatin in an intramusular mass are typial in aute hematmas. An intramusular metastasis suh as renal ell arinma an spntaneusly hemrrhage, as well. Sft tissue absesses will have a lw attenuatin entral prtin. High attenuatin fi are nt typial. Elastbrmas ur inferir t the sapular tip and usually feature sft tissue and fat attenuatin. 2. B. High attenuatin, nnalied fi within a hetergeneus lletin are typial f bld prduts. Sme tumrs ntain high attenuatin fi, but mst are equal r slightly lwer attenuatin than skeletal musle. The fi f high attenuatin
199
are nly mderately greater than thse f skeletal musle, making alium very unlikely. Fat is f lw attenuatin. 3. A. Intramusular hematma is nt a risk fatr fr malignant degeneratin. Intramusular hematma an impede bld w t the affeted musle and result in ishemi mypathy. Neurpathy an result frm an intramusular hematma hrnially mpressing peripheral nerves. Intramusular hematma an ause pressure nersis f adjaent bne, espeially when the hematma is quite large. 4. D. Spntaneus intramusular hematma an be a mpliatin f hemphilia. Aspirin prlngs bleeding time but des nt predispse t spntaneus intramusular hematma. Extravasatin f IV ntrast medium an ur during pwer injetin but des nt ause spntaneus intramusular hematma. Vitamin K supplementatin wuld interfere with warfarin therapy, reduing its antiagulatin effet.
Comment Differential Diagnosis Anterpsterir (AP) hest radigraph demnstrates hazy paiatin f the left hemithrax with enlargement f the sft tissues in the left lateral hest wall (Fig. 88.1). The CT image shws a large, well-marginated, hetergeneus left psterlateral hest wall mass entered within the musulature (Fig. 88.2). The high-attenuatin fi in this mass n unenhaned CT represent bld prduts and are nsistent with aute intramusular hematma, a mpliatin f supratherapeuti warfarin therapy. One uld nsider a hemrrhagi metastasis, suh as renal ell arinma.
Discussion Crss-setinal imaging studies an be helpful in the identiatin f the site and extent f intramusular hematmas in patients with either traumati injury r spntaneus hemrrhage. Beause f its relative lw st and lak f inizing radiatin, ultrasngraphy is usually the preferred test fr this purpse. In patients with large hematmas, CT and magneti resnane imaging (MRI) are asinally helpful in determining the extent and the age f hemrrhage and identifying the effet f the hemrrhage n adjaent rgans. Therapy is usually supprtive. Hwever, large hematmas may require surgial interventin if ishemi injury r neurpathy is suspeted. Mysitis ssians an ur in up t 10% f patients with intramusular hematma and an ause pain and dereased funtin. REFERENCES
MKenzie G, Raby N, Rithie D. Pitrial review: Nn-neplasti sft tissue masses. Br J Radiol. 2009 Aug;82(981):775–785. Thrai Radilgy: The Requisites, 3rd ed, 159–192.
CASE 89 Scimitar Syndrome 1. A and C. The large urvilinear paity in the right lwer lung is mst likely a vasular struture, likely an anmalus pulmnary vein. A vein draining a pulmnary arterivenus malfrmatin is likely withut identiatin f a similar size artery feeding the malfrmatin. Cngenital pulmnary airway malfrmatin is typially nt assiated with a large vasular anmaly, and the muele f brnhial atresia usually has a smaller diameter. 2. B. Beause the pulmnary vein drains int the right atrium, a left-t-right shunt is mst likely present. A left-t-left shunt
200
SECTION IV
Answers
urs when an anmalus pulmnary vein drains t the left atrium. A right-t-right shunt wuld ur when a pulmnary artery drained int anther artery r the right atrium. A rightt-left shunt urs with a pulmnary arterivenus malfrmatin, where bld bypasses the nrmal pulmnary apillary bed. 3. B. Simitar syndrme is assiated with a hst f ngenital abnrmalities, and hypplasti lung with abnrmal airway branhing is the mst mmn. There is n apparent inreased inidene f lung aner, kidney abnrmalities, r musular dystrphy. 4. D. Simitar syndrme in an asymptomatic adult is generally an inidental nding that requires n management. Resetin, endvasular iling, r brnhspy are nt indiated fr this patient.
Comment Differential Diagnosis Psteranterir (PA) hest radigraph (Fig. 89.1) shws a large, urvilinear struture ursing frm the right infrahilar regin t the medial right lung base, reeting the simitar vein, named beause f its likeness t a Turkish swrd. The right lung is slightly hypplasti, and the right main brnhus is smaller than usual. Crnal maximum intensity prjetin (MIP) frm magneti resnane (MR) angigram f the hest (Fig. 89.2) shws the urvilinear struture draining int the suprahepati inferir vena ava. Differential nsideratins wuld inlude pulmnary vein varix r meandering pulmnary vein. Differential auses fr the slightly hypplasti right hemithrax uld inlude pulmnary underdevelpment, interruptin f the right pulmnary artery, pulmnary vein stensis, and Swyer-JamesMaLed syndrme.
Discussion Simitar syndrme (venlbar syndrme) is a frm f partial anmalus pulmnary venus drainage, where the simitar vein drains all r part f the ipsilateral lung int the systemi venus irulatin, typially the inferir vena ava. In tw-thirds f ases, the simitar vein drains the entire lung and in ne-third f ases, nly the inferir half f the lung drains thrugh the simitar vein. In additin t anmalus venus drainage, ther anmalies are present t varying extents. Hypplasia f the ipsilateral lung urs in nearly all ases with varying degrees f abnrmal lbatin, airway branhing, and dextrpsitin f the heart. The ipsilateral pulmnary artery is usually smaller than nrmal, and systemi arterial supply t a prtin f the affeted lung urs in up t tw-thirds f patients. Other defets inlude seundum type atrial septal defet, diaphragmati hernia, and hrseshe lung. The pathgenesis f simitar syndrme is unlear, and the develpmental abnrmalities that lead t the anmalus anatmy are nt well desribed. Simitar syndrme has tw distint linial manifestatins. The infantile frm has a high assiatin with signiant ngenital ardivasular anmalies inluding ventriular septal defet, arti artatin and hypplasia, tetralgy f Fallt, trunus arterisus, and abnrmal rigin f the left rnary artery. Beause f the assiated malfrmatins, mrtality and mrbidity rates are high. In ntrast t the infantile frm, the adult frm f simitar syndrme is ften asymptmati r presents nly with milder symptms frm shunting r ther anmalies. Asymptmati patients typially d nt require treatment. Cmputed tmgraphy (CT) r MR angigraphy an better delineate the anatmy f simitar syndrme. MR has the
advantage f being able t quantify shunts whereas CT an better delineate lung abnrmalities. REFERENCES
Gudjnssn U, Brwn JW. Simitar syndrme. Semin Thorac Cardiovasc Surg Pediatr Card Surg Annu. 2006:56–62. Thrai Radilgy: The Requisites, 3rd ed, 193–209.
CASE 90 Pericardial Effusion 1. A and B. Myardial infartin with left ventriular failure is the mst mmn ause f periardial effusin. Sersitis related t systemi lupus erythematsus and ther llagen vasular diseases an ause pleural and periardial effusin. Periardial effusin is nt a typial nding f relapsing plyhndritis, whih primarily affets the trahea and brnhi. Aute pulmnary emblism an ause pleural effusin, but periardial effusin des nt ur. 2. D. The lateral view shws a lw-density vertial line behind the sternum (mediastinal fat) and anther lw-density vertial line psterir t it (epiardial fat). These tw lines are separated by uid density whih represents the periardial effusin. The appearane resembles an Ore kie. 3. C. Cardia MRI is extremely sensitive fr deteting periardial effusins. The entire periardium an be imaged in multiple planes. Chest radigraphs have very lw sensitivity fr periardial effusin, espeially with vlumes less than 200 mL. Transthrai and transesphageal ehardigraphy are highly sensitive fr periardial effusin but an miss effusins lulated psterirly beause f limited austi windws. 4. D. Dressler syndrme refers t pleural and periardial effusins, whih develp 2 t 10 weeks after aute myardial infartin. Dressler syndrme is nt related t rheumatid arthritis, viral myarditis, and radiatin indued injury.
Comment Differential Diagnosis The psteranterir (PA) radigraph shws and enlarged ardia silhuette (Fig. 90.1). The lateral radigraph shws a vertially riented sft tissue stripe anterir t the heart brdered by tw stripes f fat attenuatin (Fig. 90.2), representing a periardial effusin. Unenhaned mputed tmgraphy (CT) image (Fig. 90.3) shws the full extent f the periardial effusin and illustrates the anterir mediastinal and epiardial fat, whih enable the effusin t be visible n the lateral radigraph. Pleural effusins are als present. The differential diagnsis fr periardial effusin is brad and inludes infetin, trauma, radiatin therapy, drug txiity, llagen vasular diseases, metabli disrders, and neplasms.
Discussion The lateral radigraph shws a urvilinear retrsternal stripe braketed by fat attenuatin, als referred t as the dublelueny, sandwih r Ore kie sign. This sign refers t widening (>4 mm) f the sft tissue paity f the periardium between the luent stripes that represent fat lated anterir (mediastinal) and psterir (epiardial) t the periardium. This sign has a relatively lw sensitivity but a high speiity fr deteting periardial effusin. Chest radigraphy is assiated with a relatively pr sensitivity fr deteting periardial effusins. It has been estimated
Fair Game
that 200 mL f periardial uid must be present t reliably make the diagnsis radigraphially. In ntrast, CT and ehardigraphy are highly sensitive fr deteting periardial effusin and is the studies f hie fr sreening patients with suspeted periardial effusin. MRI may be helpful fr haraterizing mplex periardial uid lletins. Furthermre, MRI prvides detailed imaging f the entire periardium in ntrast t ehardigraphy, whih an be limited by austi windws. The mst mmn ause f periardial effusin is myardial infartin with left ventriular failure. Dressler syndrme refers t the develpment f periardial and pleural effusins 2 t 10 weeks fllwing aute myardial infartin. Suh effusins an be hemrrhagi, partiularly in patients wh have reeived antiagulatin therapy. Large periardial effusins an require drainage t prevent r relieve ardia tampnade. Sme effusins will reslve withut treatment. Periardial thikening an result in nstritive periarditis. REFERENCES
Kligerman S. Imaging f Periardial Disease. Radiol Clin North Am. 2019 Jan;57(1):179–199. Thrai Radilgy: The Requisites, 3rd ed, 226–237.
CASE 91 Idiopathic Bronchiectasis 1. A and B. Ciliary dyskinesia predispses patients t reurrent sinpulmnary infetins, and brnhietasis an result frm reurrent airway inammatin. Allergi brnhpulmnary aspergillsis urs primarily in patients with asthma and ysti brsis and leads t brnhietasis beause f hrni brnhial inammatin. Tratin brnhietasis and brnhiletasis an develp in the setting f interstitial brsis. Hwever, this is a sendary phenmenn and nt related t an intrinsi brnhial abnrmality. In additin, the dilated peripheral airways will be within areas f brsis. These etati airways may appear varise in mrphlgy. Lymphangileimymatsis is haraterized by the develpment f lung ysts with smth, thin walls. Brnhietasis is nt a mpnent f the disease. Cigarette smking leads t hrni airway inammatin and brnhial wall thikening but generally des nt result in brnhietasis. 2. D. Infetin is the mst mmn ause f brnhietasis, espeially infetins during hildhd. Cngenital brnhietasis is unmmn and is ften the result f deieny in brnhial artilage (e.g., Williams-Campbell syndrme). Interstitial pneumnia is unmmn and an be assiated with tratin brnhietasis in areas f brsis. Lymphangileimymatsis is haraterized by the develpment f lung ysts with smth, thin walls. Brnhietasis is nt a mpnent f the disease. 3. C. Brnhial wall thikening is a nnspei nding n mputed tmgraphy (CT) and an be enuntered in presses suh as brnhitis, lymphangiti arinmatsis, and lung edema. Brnhial diameter greater than that f the adjaent artery, lak f nrmal brnhial tapering, and brnhi visible in peripheral 1 m f lung are nsidered diret signs f brnhietasis. 4. D. Pleural effusin is nt a diret nsequene f brnhietasis. Hwever, a parapneumni effusin an develp frm brnhietasis-related pneumnia. Brnhietasis is ne f the mst mmn auses f hemptysis. The hrnially inamed brnhi are ften friable and prne t hemrrhage. Ateletasis frm brnhietasis an result frm muid impatin, pr learane f seretins, and brnhmalaia. Chrni brnhietasis limited t a lbe may result in
201
mplete ateletasis f that lbe withut brnhial bstrutin. Dilated air brnhgrams are a prminent feature. Pr muiliary learane in dilated and inamed brnhi predispses affeted patients t reurrent pneumnia.
Comment Differential Diagnosis Axial (Fig. 91.1) and rnal (Fig. 91.2) CT images shw mild brnhietasis haraterized by ylindrial dilatin f peripheral airways, brnhial wall thikening, and muus plugging. Als nte the presene f tree-in-bud paities and msai attenuatin, reeting assiated small airways disease. The auses f brnhietasis are myriad and inlude infetin (inluding atypial mybaterial), immune deieny, iliary dyskinesia, ngenital artilage deieny in the airways, hrni aspiratin, allergi brnhpulmnary aspergillsis, ysti brsis, small airways disease, and thers.
Discussion Brnhietasis is dened as abnrmal, irreversible dilatin f the brnhi. Brnhietasis an arise sendary t a wide variety f ngenital and aquired abnrmalities. Cysti brsis is the mst mmn assiated ngenital abnrmality, and prir infetin, espeially hildhd infetins, is the mst mmn aquired abnrmality. Chest radigraphs are ften nrmal in patients with mild degrees f brnhietasis but asinally reveal parallel, thikened brnhial walls, als referred t as a tram-trak appearane. With ysti brnhietasis, radigraphs an reveal lusters f air-lled ysts, ften with uid levels. CT is highly sensitive and spei fr diagnsing brnhietasis. Findings inlude a brnhial wall diameter greater than its adjaent artery (resulting in a signet-ring sign when the dilated brnhus and ampanying artery are viewed in rss setin), identiatin f brnhi within the peripheral entimeter f the lung, lak f nrmal brnhial tapering, brnhial wall thikening, and strings r lusters f ysts. Beause brnhial wall thikening may als be seen in ther frms f airway disease, it shuld nt be used as a sle riterin fr diagnsing brnhietasis. Cmpliatins f brnhietasis inlude reurrent infetins, hemptysis, muus impatin, and ateletasis. Patients are usually treated nservatively with antibitis as needed. Severe hemptysis an require brnhial artery emblizatin r resetin f affeted lung. REFERENCES
Bnavita J, Naidih DP. Imaging f brnhietasis. Clin Chest Med. 2012;Jun;33(2):233–248. Millirn B, Henry TS, Veeraraghavan S, Little BP. Brnhietasis: mehanisms and imaging lues f assiated mmn and unmmn diseases. Radiographics. 2015 Jul–Aug;35(4):1011–1030. Thrai Radilgy: The Requisites, 3rd ed., pp. 137–158.
CASE 92 Achalasia 1. B and D. Ahalasia results in esphageal bstrutin and dysmtility. The esphagus prximal t the hypertni segment an be lled with air, liquid, r fd debris. Esphageal arinma an ause esphageal bstrutin, leading t prximal distensin and retained fd, liquid, and gas. Hwever, the degree f esphageal dilatin illustrated in this ase is rare in esphageal arinma and usually indiates a mre hrni nditin.
202
SECTION IV
Answers
Tumr ells an als invade the nerves f the lwer esphageal sphinter, preventing nrmal relaxatin during swallwing (pseudahalasia). Mst esphageal dupliatin ysts are uid lled and typially are rund, masslike strutures. Resultant high-grade bstrutin wuld be highly unusual. An esphageal divertiulum is a fal utpuhing r rund paraesphageal mass and des nt ause diffuse dilatin f the esphagus. 2. A. The tubular nature f the gas-lled struture is typial f the esphagus. Althugh an enlarged azygs vein displaes the azygesphageal ntur, it des nt ntain gas. The gaslled struture extends abve the expeted latin f the azygs arh is at the level f the trahebrnhial angle. The right atrium is nt enlarged. Its nturs are anterir t the azygesphageal reess. The left atrium is nrmal in this patient. 3. C. An esphagram an prvide infrmatin regarding esphageal mtility, unlike the ther tests whih are listed. A CT an shw esphageal dilatin, thikening, and narrwing, but it annt evaluate esphageal mtility. MRI has a very limited rle in the evaluatin f esphageal disrders. The lateral radigraph an prvide further supprt that the abnrmal struture is the esphagus, but it will nt prvide any funtinal infrmatin. 4. A. Prgressive systemi slersis (slerderma) typially results in dilatin f the esphagus and dysmtility with a patulus lwer esphageal sphinter. Ahalasia results frm failure f the lwer esphageal sphinter t relax. Chagas disease is aused by Trypanosoma cruzi, whih an invade and destry the nerve plexus respnsible fr relaxatin f the lwer esphageal sphinter. It urs primarily in Suth Ameria. Tumr invasin f the nerve plexus respnsible fr relaxatin f the lwer esphageal sphinter (pseudahalasia) an mimi ahalasia.
Differential Diagnosis The psteranterir (PA) hest radigraph (Fig. 92.1) shws a dilated, tubular struture ntaining a uid level ursing thrugh the mediastinum and displaing the nrmal azygesphageal ntur, representing the esphagus. Ahalasia is the mst likely ause f severe esphageal dysmtility and dilatin. Other auses inlude Chagas disease, pseudahalasia (neplasti invasin f the nerves f the lwer esphageal sphinter), and prgressive systemi slersis (slerderma).
Discussion The azygesphageal interfae is prdued by the juxtapsitin f aerated lung in the right lwer lbe and the sft tissue paity f the right lateral margin f the azygs vein, esphagus, r bth. On nrmal hest radigraphs, the azygesphageal interfae begins at the level f the azygs arh and extends inferirly t the level f the diaphragm. It nrmally prdues a nave slpe as it urves slightly tward the left. Abnrmalities f either the azygs vein (e.g., azygs ntinuatin f the inferir vena ava) r esphagus (e.g., ahalasia) an result in rightward displaement f this interfae. Subarinal masses suh as brnhgeni ysts and lymph nde enlargement an als result in fal rightward displaement f the azygesphageal interfae, usually prduing a rightward nvexity in the subarinal regin. The hest radigraph (Fig. 92.1) shws diffuse, marked rightward displaement f the azygesphageal ntur. When an air-lled, distended esphagus displaes the azygesphageal ntur, a stripe is frmed rather than an interfae. This patient has a histry f ahalasia. In mst patients with this disrder, the esphagus ntains a large amunt f retained seretins. Therefre, the radigraph typially shws a displaed azygesphageal interfae rather than a stripe. Retained seretins
and fd an als result in a disrete uid level within the distended esphagus. Treatment fr ahalasia inludes a Heller mytmy t release the lwer esphageal sphinter. Sme patients require balln dilatin. Sendary gastresphageal reux an develp after mytmy. REFERENCES
Whitten CR, Khan S, Munneke GJ, Grubni S. A diagnsti apprah t mediastinal abnrmalities. Radiographics. 2007;27:657–671. Thrai Radilgy: The Requisites, 3rd ed., pp. 19–60, 97–136.
CASE 93 Cavity Caused by Coccidioidomycosis 1. A and C. Granulmatsis with plyangiitis (Wegener’s) an ause lung ndules and avities. Cavities ften have thik irregular walls; hwever, with learing, the walls an beme thin. A slitary avitary lesin is a mmn manifestatin f hrni idiidmysis. Cavities an frm in advaned saridsis but usually d s in the setting f tratin brnhietasis and brsis. There are n ndings n the images t suggest saridsis. Septi infarts are usually multiple, peripheral and predminate in the lwer lbes. They an avitate and usually have thik walls and the brad surfae usually abuts the pleura. Mst are smaller than the avity in this patient. 2. D. Coccidioides is endemi t the Amerian Suthwest in additin t Mexi, Central Ameria, and Suth Ameria. 3. B. Lung avities are typial f the hrni frm f idiidmysis and nt the aute pneumni frm. Mst infeted patients are asymptmati, and disseminated disease is rare. Mst patients with the pneumni frm have spntaneus reslutin f their infetin. 4. B. Mst patients with disseminated idiidmysis have multiple tiny ndules, typially 5 t 10 mm in diameter. Multifal lung nslidatin, diffuse grund-glass paity, and septi pulmnary infarts are nt typial features f disseminated idiidmysis.
Comment Differential Diagnosis The psteranterir (PA) radigraph (Fig. 93.1) shws an vid mass just lateral t the left hilum. Cavitatin is mre nspiuus n mputed tmgraphy (CT) (Fig. 93.2). Nte the thik wall f the avity but the rather smth interir. Sme satellite ndules are als present. Malignant neplasm and infetin shuld be strngly nsidered. Granulmatsis with plyangiitis an als present as a avitary lesin but mre ften lesins are multiple.
Discussion Cidiidmysis infetin is aused by inhalatin f spres f Coccidioides immitis r Coccidioides posadasii, sil inhabitants endemi t desert areas. Althugh mst patients are asymptmati fllwing expsure, sme experiene a mild, ulike illness. Radigraphi ndings vary depending n the stage f infetin. Fllwing initial inhalatin f the spres, there is a lal pneumni respnse, whih is haraterized radigraphially as an area f nslidatin. Suh nslidatin usually invlves less than an entire lbe, is ften lated in the lwer lbes, and usually reslves spntaneusly withut therapy. Chrni pulmnary idiidmysis is haraterized radigraphially by slitary r multiple pulmnary ndules and avities.
Fair Game
Suh avities, as shwn in this ase, an have variable wall thikness and are usually radilgially indistinguishable frm ther auses f avitary lesins. In a minrity (10% t 15%) f patients, idiidmysis is assiated with harateristi thin walled (grape skin) avities. Suh avities an rapidly hange in size, presumably due t a hek-valve mmuniatin with the brnhial tree. Disseminated idiidmysis is rare; it appears radigraphially as multiple ndules. The ndules usually range in size frm 5 mm t 1 m in diameter, but smaller miliary ndules are bserved in sme ases. The urse f disseminated idiidmysis is variable: it may be hrni and insidius r rapidly fatal. The infetin is usually fatal in patients wh are immunmprmised. Like ther infetins, idiidmysis an ause a false-psitive result n urdexygluse–psitrn emissin tmgraphy (FDG-PET) studies. This an ur in the aute r hrni phase f infetin. Mst patients require n treatment. Antifungal therapy may be used in patients whse disease fails t lear spntaneusly. REFERENCES
Lindell RM, Hartman TE:. Fungal infetins. In: Müller NL, Silva CI, eds. Imaging of the Chest. Philadelphia: Saunders; 2008:362–365. Kunin JR, Flrs L, Hamid A, Fuss C, Sauer D, Walker CM. Thrai endemi fungi in the United States: imprtane f patient latin. Radiographics. 2021 Mar–Apr;41(2):380–398. Thrai Radilgy: The Requisites, 3rd ed., pp. 289–309.
CASE 94 Hydrostatic Pulmonary Edema 1. A and C. Hydrstati pulmnary edema (edema sendary t ngestive heart failure r vlume verlad) is the mst mmn ause f interlbular septal thikening. Smth interlbular septal thikening is nt a typial feature f usual interstitial pneumnia, whih is haraterized by irregular lines (retiulatin), reeting interstitial brsis. Smth interlbular septal thikening is nt a typial nding f saridsis. Hwever, small ndules may be present alng the interlbular septa in patients with saridsis. 2. A. Kerley B lines n the hest radigraph rrespnd t thikened interlbular septa n CT. Other hest radigraphi ndings inlude thikening f the interlbar ssures that is due t edema in r thikening f the subpleural interstitium n bth sides f the ssure. Brnhvasular bundle thikening n the hest radigraph rrespnds t edema in r thikening f the peribrnhvasular interstitium. Fluffy lung nslidatin des nt rrespnd t interlbular septal thikening n CT. 3. A. HRCT requires the use f thin llimatin (≤1.5 mm). Prne imaging may be used in sme patients as part f an interstitial lung disease prtl. Multiplanar refrmatins an be generated frm vlumetri CT imaging. Hwever, they are nt a required mpnent f HRCT. A high spatial frequeny renstrutin kernel prvides edge sharpening, better delineating the small brnhpulmnary strutures n HRCT. 4. B. Repeat hest imaging, either with a radigraph r CT, fllwing diuresis an easily nrm whether septal thikening is the result f hydrstati edema. FDG-PET might shw uptake thrughut the lung if the septal thikening reets diffuse lymphangiti arinmatsis. Hwever, the results are nnspei. Transbrnhial bipsy and surgial bipsies are f urse invasive. They may be a gd ptin if septal thikening persists despite diuresis, and the linial presentatin suggests hrni interstitial lung disease.
203
Comment Differential Diagnosis The primary differential diagnsis fr smth interlbular septal thikening is pulmnary edema and lymphangiti arinmatsis. Edema tends t be symmetri and gravitatinally dependent, whereas lymphangiti arinmatsis mre ften is randmly distributed. Pleural effusins an be present with either nditin. Other auses f septal thikening inlude pulmnary venus bstrutin, strage diseases and amylidsis, as well as lymphati and lymphprliferative disrders. Septal thikening an als be a sendary nding in patients with infetin and pulmnary hemrrhage.
Discussion Althugh the diagnsis f ngestive heart failure is usually made n the basis f typial linial and radigraphi ndings, asinally patients with unsuspeted ngestive heart failure are imaged with HRCT f the hest in searh f a ause f dyspnea. Hydrstati edema is als an asinal inidental nding in patients wh are being sanned fr ther purpses. Thus, it is imprtant t be aware f the typial HRCT features f hydrstati edema. On CT f patients with hydrstati edema (Fig. 94.1), a mbinatin f grund-glass paity, smthly thikened septal lines, peribrnhvasular interstitial thikening, inreased vasular aliber, and thikened ssures may be present. Small pleural effusins, ften n the right, are als ften present. Signs f brsis suh as hneymbing, tratin brnhietasis, and arhitetural distrtin are absent. Interestingly, patients with aute ngestive heart failure have als been reprted t have asinally enlarged mediastinal lymph ndes whih are edematus and haziness f the mediastinal fat als aused by edema. Crrelatin between imaging ndings and linial data is usually sufient t nrm the diagnsis. When the diagnsis is in dubt linially, a fllw-up study after diuresis is asinally helpful t nrm reslutin f abnrmalities and t exlude hrni interstitial lung disease. REFERENCES
Strt ML, Kee ST, Glden JA, Webb WR. Hydrstati pulmnary edema: high-reslutin CT ndings. AJR Am J Roentgenol. 1995;165:817–820. Thrai Radilgy: The Requisites, 3rd ed., pp. 226–237.
CASE 95 Internal Mammary Lymph Node Enlargement 1. B and C. Lymphma and breast arinma are mmn auses f internal mammary lymphadenpathy. Arti artatin an ause enlargement f the internal mammary (thrai) vessels, resulting in a lbulated retrsternal paity. Hwever, the arta is nrmal in this patient, and arti artatin wuld nt aunt fr anterir mediastinal abnrmality. Althugh lymphadenpathy is a mmn manifestatin f tuberulsis, invlvement f the internal mammary lymph ndes is unmmn in the absene f disseminated disease. 2. B. Ipsilateral axillary lymph ndes are the mst mmn site f lymph nde metastases in patients with breast arinma. Internal mammary lymph ndes are a mmn site f lymph nde metastases in breast aner, but they are nt the mst mmn. Hilar and mediastinal lymph ndes are less-mmn sites f lymph nde metastases frm breast arinma. 3. B. Internal mammary lymph ndes are apparent n the lateral hest radigraph nly when they are quite large.
204
SECTION IV
Answers
4. C. Primary mediastinal lymphma, whih is nned t the anterir mediastinum, is mst mmnly diffuse large B ell lymphma (als referred t as Primary mediastinal B cell lymphoma). This is a relatively reently desribed tumr whih an lsely resemble Hdgkin disease. Classial Hdgkin lymphma an ur in the anterir mediastinum but is less mmn. Flliular lymphma less mmnly is islated t the anterir mediastinum. Small lymphyti lymphma is usually systemi affeting the spleen and multiple lymph nde statins thrughut the bdy.
Comment
is mmn, but hemptysis nly urs in up t 10% f patients. Air emblism and bipsy trak seeding with tumr ells are extremely rare mpliatins. 4. C. Use f a utting needle fr re needle bipsy imprves the diagnsti yield f transthrai needle bipsy (TTNB) fr a spei benign diagnsis. Inreasing vlume f aspirate will nt likely imprve the diagnsti yield f TTNB fr a benign lesin. On-site ytpathlgy an be useful fr imprving the diagnsti yield f ne needle aspiratin fr malignant lesins. Hwever, establishing a benign diagnsis based n ne-needle aspiratin alne an be very difult. Use f CT urspy des nt imprve the diagnsti yield f TTNB fr benign lesins.
Differential Diagnosis
Comment
The lateral radigraph (Fig. 95.1) shws a lbulated paity lling the retrsternal lear spae. Cntrast-enhaned mputed tmgraphy (CT) image (Fig. 95.2) shws bilateral internal mammary lymphadenpathy and a sft tissue mass in the anterir mediastinum. Althugh breast arinma metastases are the mst mmn ause f internal mammary lymphadenpathy, lymphma wuld be mre mmn in this male patient.
The CT image (Fig. 96.1) shws a TTNB predure. The CT image nrms the intralesinal latin f the bipsy needle. Regarding planning a TTNB predure, a prebipsy CT san shuld be btained. The shrtest, mst vertial rute shuld be hsen, and the path f the needle shuld avid interlbar ssures, pulmnary vessels, bullae, and areas f severe emphysema. TTNB is a relatively safe and aurate predure fr btaining bipsy speimens f lung ndules and masses. The sensitivity fr malignant ndules is greater than 90%, and the auray fr differentiating amng varius ell types f lung aner is rughly 80%. A majr limitatin f TTNB using ne-needle aspiratin is a relatively lw sensitivity (10%–40%) fr making a spei benign diagnsis. Hwever, this an be signiantly imprved by using re needle bipsy devies. Suh devies prvide histlgi speimens, whih imprve the auray f diagnsing benign entities suh as granulmas, hamartmas, and rganizing pneumnia. It is imprtant t remember that a bipsy fr negative malignany is nt diagnsti unless a spei benign diagnsis has been established. Indeed, abut 30% f lesins with nnspei negative bipsy results ultimately prve t be malignant. Thus, when faed with nnspei negative bipsy result, ne shuld nsider repeating the bipsy with a re needle bipsy. Alternatively, with n-site ytpathlgy, FNA an be perfrmed thrugh a guide needle, and if the FNA speimen is insufient, re needle bipsy an be frmed at the same time. With the inreasing imprtane f mleular markers fr aner diagnsis and treatment, re needle bipsy bemes mre imprtant fr tissue sampling. Cre needle bipsy is als remmended fr bipsy f lesins with a suspeted diagnsis f lymphma t prvide sufient tissue fr lassiatin f lymphma.
Discussion Enlarged internal mammary lymph ndes are generally nt visible n hest radigraphs until they are nsiderably enlarged. On a psteranterir (PA) hest radigraph f a patient with enlarged internal mammary ndes, a fal parasternal paity, whih is usually seen at the level f the rst three interstal spaes and less mmnly at the furth r fth level, may be present. On a lateral radigraph, a lbulated retrsternal paity may be present, as in this ase. A lbulated retrsternal paity may als be present in patients with dilated internal mammary vessels. Fr example, artatin f the arta is assiated with llateral internal mammary arteries, and superir vena ava bstrutin is assiated with llateral internal mammary veins. The frmer is assiated with a harateristi appearane f the arta and asinal rib nthing, and the latter is usually assiated with a large mass in the right paratraheal regin. REFERENCES
Sharma A, Fidias P, Hayman LA, etal. Patterns f lymphadenpathy in thrai malignanies. Radiographics. 2004;24:419–434. Thrai Radilgy: The Requisites, 3rd ed., pp. 97–136.
CASE 96 CT-Guided Transthoracic Needle Biopsy Procedure 1. B. Emphysema is the greatest risk fatr fr develping pneumthrax fllwing transthrai needle bipsy f the lung. Apprximately 50% f patients with emphysema will develp pneumthrax fllwing bipsy in ntrast t apprximately 20% t 30% f patients withut emphysema. Older age and lwer lbe latin d nt signiantly affet pneumthrax rate. Bipsy f subpleural lesins is less likely t result in pneumthrax beause the needle may nt pass thrugh aerated lung. 2. D. The sensitivity f transthrai needle bipsy fr malignant lesins is greater than 90%. There is n signiant differene between re needle bipsy and ne needle aspiratin in mst aners. Hwever nn-slid ndules have a higher yield with re needle bipsies. 3. B. Up t 30% f patients underging transthrai needle bipsy will develp a pneumthrax. Mst pneumthraes reslve withut treatment. Mild perilesinal hemrrhage
REFERENCES
Cham MD, Lane ME, Henshke CI, Yankelevitz DF. Lung bipsy: speial tehniques. Semin Respir Crit Care Med. 2008;Aug;29(4):335–349. Thrai Radilgy: The Requisites, 3rd ed., pp. 460–472.
CASE 97 Bronchial Atresia 1. A and C. Pulmnary arinids an grw in a brnhial lumen and result in muid impatin and a brnhele (a irumsribed dilatin f a brnhus). Typially, an endbrnhial mass expanding the airway is visible. Brnhial atresia an result in muid impatin and brnhele frmatin. Cnstritive brnhilitis affets the small airways and results in diffuse r hetergeneus lw-attenuatin lung frm air trapping. This is present in the upper lbe in this ase, but nstritive brnhilitis is nt assiated with a brnhele. Lung absesses are typially rund and ntain liquid with
Fair Game
r withut gas. A lung absess wuld nt explain the adjaent hyperlueny and the walls f absesses are usually thik. 2. D. Air an enter lung distal t an bstrutin thrugh the anals f Lambert and pres f Khn. Brnhpleural stula results in air entering the pleural spae frm adjaent, usually inamed, r nerti, lung. An anmalus brnhus usually des nt prvide aeratin t lung distal t a muele. 3. A. The left upper lbe, espeially the apipsterir segment, is the mst mmn site f brnhial atresia. Hwever, brnhial atresia an affet any lbe and rarely an be multisegmental r multilbar. 4. C. Mst patients with brnhial atresia require n treatment. Resetin is rarely indiated. There is n rle fr endbrnhial therapy.
Comment Differential Diagnosis Axial and rnal (Figs. 97.1 and 97.2) mputed tmgraphy (CT) images shw a tubular struture in the left upper lbe ntaining an air-uid level. The part f the lung supplied by this brnhus is hyperluent due t air trapping. This appearane is mst nsistent with brnhial atresia. Endbrnhial neplasms an als ause brnhele frmatin. The differential diagnsis f brnhial atresia (BA) inludes ther auses f muid impatin inluding allergi brnhpulmnary aspergillsis (ABPA), benign and malignant endbrnhial tumrs (arinid, r rarely lung aner), ysti brnhietasis related t previus infetin, and tuberulus brnhstensis.
Discussion BA is a rare and benign nditin haraterized by fal atresia f a segmental, subsegmental, r lbar brnhus. The left upper lbe, espeially the apipsterir segment, is mst mmnly invlved. The airways distal t the pint f atresia are nrmal but the bstrutin t prximal drainage auses aumulatin f muus distal t the pint f atresia, with resultant muid impatin and frmatin f a brnhele. An imprtant assiated feature is hyperinatin f the distal lung parenhyma thrugh llateral air drift and air trapping. BA manifests radilgially as a rund, vid, r branhing struture (brnhele lled with muus) assiated with distal hyperinatin, but it may be verlked n the hest radigraph. Althugh its imaging features are harateristi, BA is mmnly misdiagnsed, ften mistaken fr an arterivenus malfrmatin. Beause mst ases f BA d nt require surgial resetin, regnitin f its harateristi imaging features is essential fr nservative management. REFERENCES
Kinsella D, Sissns G, Williams MP. The radilgial imaging f brnhial atresia. Br J Radiol. 1992;65:681–685. Biyyam D, Chapman T, Fergusn M, etal. Cngenital lung abnrmalities: embrylgi features, prenatal diagnsis, and pstnatal radilgi-pathlgi rrelatin. Radiographics. 2010;30:1721–1738. Thrai Radilgy: The Requisites, 3rd ed., pp. 193–209.
CASE 98 Hypersensitivity Pneumonitis 1. C and D. The differential diagnsis fr grund-glass attenuatin entrilbular ndules is primarily hypersensitivity pneumnitis (HP) and respiratry brnhilitis. HP is muh mre likely given the extent f ndules and the patient’s symptms. Saridsis and silisis are haraterized by the presene f well-dened ndules
205
in a mid and upper lung predminantly. The ndules are usually slid and lated in a perilymphati distributin. 2. B. Bth the pulmnary artery and brnhile are in the entral re f the pulmnary lbule. The pulmnary veins are nt lated in the entral re f the pulmnary lbule but in the interlbular septa. Pulmnary lymphatis are lated primarily in the interlbular septa as well as in the brnhvasular bundles. 3. D. Chrni esinphili pneumnia is nt assiated with igarette smking and ften urs in asthmati patients. Respiratry brnhilitis, pulmnary Langerhans ell histiytsis, and desquamative interstitial pneumnia ur primarily in igarette smkers. 4. D. Hypersensitivity pneumnitis results primarily frm a ell-mediated delayed-type hypersensitivity reatin (type IV). IgE and yttxi antibdies d nt play a signiant rle in the pathgenesis f hypersensitivity pneumnitis. Immune mplexes play a smaller rle in the pathgenesis f HP.
Comment Differential Diagnosis Cmputed tmgraphy (CT) images (Figs. 98.1 and 98.2) shw diffuse entrilbular grund-glass attenuatin ndules with sme areas f nuent grund-glass paity. A few lw attenuatin lbules are present, suggesting a mpnent f air trapping. The leading diagnsti nsideratin fr a CT with this appearane is hypersensitivity pneumnitis. Respiratry brnhilitis shuld be nsidered in a persn wh smkes, althugh the extent f grund-glass ndules is typially muh less. Oasinally, infetin, espeially Pneumocystis jirovecii an have a similar appearane but nuent grund glass paities predminate rather than grund glass ndules.
Discussion The presene f entrilbular grund-glass ndules n mputed tmgraphy (CT) shuld prmpt nsideratin f the diagnsis f nnbrti HP. Affeted patients typially present with dyspnea and hrni ugh, and the linial piture may be nfused with ther frms f diffuse lung disease. Often, the radilgist is the rst member f the health are team t suggest the pssibility f HP, and the linial team an then investigate the patient’s envirnmental and upatinal expsures t seek a ausative antigen. Differential diagnsti nsideratins fr entrilbular grund-glass ndules shuld inlude respiratry brnhilitis, a disease that affets igarette smkers, and, in the prper linial setting, infetin, inluding Pneumocystis jirovecii pneumnia, althugh this manifestatin is less mmn than thers. Centrilbular ndules in hypersensitivity pneumnitis usually are prly dened and may be assiated with pathy areas f grund-glass attenuatin. The ndings tend t predminate in the upper lung znes, and air trapping may be present n expiratry CT imaging. Treatment f subaute hypersensitivity pneumnitis nsists f remving the ffending antigen frm the patient’s envirnment and ften presribing rtisterids. REFERENCES
Hypersensitivity Pneumnitis: A Cmprehensive Review. J Investig Allergl Clin Immunl. 2015;25(4):237–250 Thrai Radilgy: The Requisites, 3rd ed., pp. 355–376.
CASE 99 Solitary Pulmonary Nodule: Lung Cancer Screening 1. A, B, and C. A small nnalied lung ndule uld represent primary lung arinma, a metastasis, r a nnalied
206
SECTION IV
Answers
granulma. Althugh infetin (typially granulmatus, i.e., tuberulsis [TB] r fungal) an present as multiple r a slitary ndules, primary baterial infetin des nt usually present as a well-dened small ndule. 2. C. A 7 mm slid ndule n a baseline sreening san wuld be lassied as Lung-RADS 3 (prbably benign) ategry. Lung-RADS 1 inludes n ndules r denitely benign ndules. Lung-RADS 2 inludes slid ndules at baseline